SB: Biology 1

Réussis tes devoirs et examens dès maintenant avec Quizwiz!

BONUS BIOLOGY Short Answer What is the typical reported initial threshold value, in millivolts, of a human peripheral nerve axon?

-45 (ACCEPT: -50 TO -40)

BIOLOGY Short Answer: How many human ova are produced by a full round of meiosis from a single primary oocyte?

1

Identify all of the following four hormones that are released by the pituitary gland: 1) growth hormone; 2) prolactin; 3) cortisol; 4) epinephrine [e-puh-NE-frin].

1 AND 2 (ACCEPT: GROWTH HORMONE AND PROLACTIN)

Identify all of the following four molecules that are known to act as second messengers: 1) cyclic AMP; 2) diacylglycerol [dy-as-il-GLY-se-rohl]; 3) glycine [GLYseen]; 4) serotonin.

1 AND 2 (CYCLIC AMP AND DIACYLGLYCEROL)

Identify all of the following three statements that are true regarding lysosomes: 1) Lysosomal enzymes are active at acidic pH; 2) Lysosomes are implicated in inclusion-cell disease; 3) Lysosomes are formed from late endosomes.

1, 2 AND 3

BONUS BIOLOGY Short Answer: For every carbon dioxide molecule fixed in the Calvin-Benson cycle, how many NADPH molecules are utilized?

2

Identify all of the following four choices that would NOT be visible under a standard laboratory light microscope when visualizing a plant cell: 1) nucleus; 2) mRNA [M-R-N-A]; 3) cell wall 4) chloroplast.

2 (ACCEPT: mRNA)

BIOLOGY Short Answer: In angiosperm seed development, what fuses with the second sperm nucleus to form the endosperm?

2 POLAR NUCLEI (ACCEPT: 2 OR BOTH POLAR NUCLEI or POLAR NUCLEI) (DO NOT ACCEPT: POLAR NUCLEUS)

Arrange the following three steps of cell respiration from lowest to highest in terms of net NADH molecules produced per one molecule of glucose entering respiration: 1) glycolysis [gly-KOL-uh-sis], 2) electron transport chain, 3) Krebs cycle.

2, 1, 3 (ACCEPT: ELECTRON TRANSPORT CHAIN, GLYCOLYSIS, KREBS CYCLE)

How many chromosomes does a human gamete contain?

23

Ascientist has pea plants that are homozygous for purple flowers and plants that are homozygous for white flowers. Given that the purple flower trait is dominant, if the plants are crossed and then the resulting F1 generation is allowed to self-pollinate, what percentage of the resulting F2 plants will have white flowers?

25

Identify all of the following three neurotransmitters that are considered to have only excitatory actions: 1) acetylcholine [uh-seet-l-KOH-leen], 2) GABA, 3) glutamate.

3 (ACCEPT: GLUTAMATE

BIOLOGY Short Answer What is the most common name for the terminus of a tRNA molecule to which amino acids are attached?

3-PRIME TERMINUS (ACCEPT: CCA 3-PIME END or 3-PRIME END or 3-PRIME)

How many molecules of ATP are needed to synthesize two glucose molecules in the Calvin cycle?

36 ATP

Bonus: Place the following four structures in order from inside to outside for stems with a vascular cylinder: 1) Pericycle; 2) Cortex; 3) Epidermis; 4) Pith.

4, 1, 2, 3

BONUS BIOLOGY Short Answer If a germ cell of a genus of grass has 48 chromosomes at metaphase II, how many chromosomes will a germ cell of this grass have in anaphase-II?

96

Which neurotransmitter is most commonly responsible for action potentials at the animal neuromuscular [noo-roh-MUHS-kyuh-ler] junction?

ACETYLCHOLINE

BONUS BIOLOGY Short Answer: Name all of the following 3 transport systems that directly require energy to operate: active transport; facilitated diffusion; sodium potassium pump

ACTIVE TRANSPORT; SODIUM POTASSIUM PUMP (ACCEPT: ALL BUT FACILITATED DIFFUSION)

BONUS BIOLOGY Short Answer: Most corticosteroids are synthesized in what specific part of the human body?

ADRENAL CORTEX (ACCEPT: ADRENAL GLANDS)

BIOLOGY Short Answer: By words or numbers, name all of the following 3 functions that meiosis accomplishes: 1) reducing the copies of alleles in the ovum and sperm 2) reducing the numbers of chromosomes in the ovum and sperm 3) increasing genetic diversity through chromosomal exchange events

ALL

BONUS BIOLOGY Short Answer: Name all of the following 3 groups of organisms that generate ATP by chemiosmosis (read as: chem-ee-oz-MOE-sis): Cyanobacteria; plant cells; animal cells

ALL

BONUS BIOLOGY Short Answer Name all of the following 4 organisms in which transposons (read as: trans-POE-zons) are found: humans; maize; fruit fly; bacteria

ALL

BIOLOGY Short Answer: What are the common names for the 2 functional groups that combine to form a peptide bond?

AMINO; CARBOXYL (ACCEPT: AMINE AND CARBOXYLIC ACID)

BONUS BIOLOGY Multiple Choice In one form of epigenetics called genomic imprinting, DNA is silenced by what mechanism? W) Methylation X) Acetylation Y) Phosphorylation Z) Ubiquitylation

ANSWER W) METHYLATION

BIOLOGY Multiple Choice The drug colchicine [KOL-chuh-seen] inhibits polymerization of tubulin in microtubule assembly. Which of the following cell processes would be most severely impacted by colchicine activity? W) DNA replication X) Mitosis Y) Translation Z) Cellular respiration

ANSWER X) MITOSIS

BONUS BIOLOGY Multiple Choice What is the reason why many modern crop-producing plants, such as wheat, have much larger cell volumes than the ancestral wild plants from which they were derived? W) Plants can tolerate trisomy X) Plants can tolerate polyploidy Y) Plants have been genetically engineered Z) Plants tolerate gene duplications

ANSWER X) PLANTS CAN TOLERATE POLYPLOIDY

[bonus] Short Answer The oxidized form of nicotinamide adenine dinucleotide has what specific charge?

ANSWER: +1 (ACCEPT: 1 PLUS or ONE POSITIVE)

BIOLOGY Short Answer If there are two alleles for the gene encoding size, and if the frequency of the big size allele, B [upper case B], in a population of warthogs is 0.4, then what is the frequency of the small size allele, b [lower case b], assuming Hardy Weinberg equilibrium?

ANSWER: 0.6

BONUS Biology - Short Answer Identify all of the following three hormones that are released by the posterior pituitary gland: 1) Oxytocin; 2) Calcitonin; 3) Luteinizing hormone

ANSWER: 1

Biology - Short Answer Identify all of the following three organelles that are surrounded by a double membrane: 1) Nucleus; 2) Lysosome; 3) Ribosome.

ANSWER: 1

BIOLOGY Short Answer How many human ova are produced by a full round of meiosis from a single primary oocyte?

ANSWER: 1 (Solution: mammals produce 1 ovum and 3 polar bodies per primary oocyte)

BONUS Biology - Short Answer Identify all of the following three bile components that are secreted by hepatocytes: 1) Bile salts; 2) Bilirubin; 3) Bicarbonate.

ANSWER: 1 AND 2

BONUS Biology - Short Answer Identify all of the following three bile components that are secreted by hepatocytes: 1) Bile salts; 2) Bilirubin; 3) Bicarbonate.

ANSWER: 1 AND 2

BONUS Biology - Short Answer Identify all of the following three statements that are true regarding fats: 1) Fats are a more efficient form of energy storage than proteins; 2) Fats play a role in cell structure; 3) Fats are a less efficient form of energy storage than carbohydrates.

ANSWER: 1 AND 2

BONUS Biology - Short Answer Identify all of the following three choices that are pieces of evidence that lend credence to the endosymbiotic theory: 1) Chloroplasts contain their own genomes; 2) Mitochondria contain bacteria-like ribosomes; 3) All mitochondrial proteins are synthesized in the nucleus.

ANSWER: 1 AND 2

Biology - Short Answer Identify all of the following three compounds that one would find in an animal cell: 1) D-glucose; 2) L-alanine; 3) L-trehalose.

ANSWER: 1 AND 2

BONUS Biology - Short Answer Identify all of the following three choices that are properties of exoskeletons: 1) Better protection to internal organs; 2) Unlimited growth potential; 3) High resistance to bending.

ANSWER: 1 AND 3

BONUS Biology - Short Answer Identify all of the following three choices that are true of fairy rings: 1) The oldest portion of the fungus is at the center of the fairy ring; 2) The center of the fairy ring is absorbing the most nutrients per unit time relative to the rest of the ring; 3) The outer edge of the fairy ring is where one could find basidiocarps.

ANSWER: 1 AND 3

BONUS Biology - Short Answer Identify all of the following three choices that are properties of exoskeletons: 1) Better protection to internal organs; 2) Unlimited growth potential; 3) High resistance to bending.

ANSWER: 1 AND 3

BONUS Biology - Short Answer Identify all of the following three choices that are true of fairy rings: 1) The oldest portion of the fungus is at the center of the fairy ring; 2) The center of the fairy ring is absorbing the most nutrients per unit time relative to the rest of the ring; 3) The outer edge of the fairy ring is where one could find basidiocarps.

ANSWER: 1 AND 3

Biology - Short Answer Identify all of the following three choices that represent correct reasons why the eradication of polio is possible: 1) Humans are the only reservoir of polio; 2) Transmission of the polio virus is very difficult; 3) Vaccination generates herd immunity.

ANSWER: 1 AND 3

Biology - Short Answer Identify all of the following three statements that are true regarding nutrition: 1) Leptin suppresses appetite; 2) Ghrelin [greh-l ihn]suppresses appetite; 3) A rise in blood sugar stimulates the beta cells of the pancreas.

ANSWER: 1 AND 3

Biology - Short Answer Identify all of the following three statements that are true regarding nutrition: 1) Leptin suppresses appetite; 2) Ghrelin [greh-lihn] suppresses appetite; 3) A rise in blood sugar stimulates the beta cells of the pancreas.

ANSWER: 1 AND 3

[bonus] Short Answer By number or name, identify all of the following 4 areas of plants where you would be MOST likely to see meristematic tissue: 1) longitudinal section of a terminal bud 2) cross-section of the pith of a sunflower stem 3) transverse section of a lateral root tip 4) cross-section of central parenchymal cells of a corn stem

ANSWER: 1 AND 3 (ACCEPT: 1 - LONGITUDINAL SECTION OF A TERMINAL BUD AND 3 - TRANSVERSE SECTION OF A LATERAL ROOT TIP)

BONUS Biology - Short Answer Identify all of the following three statements that are TRUE regarding E. coli: 1) Gram negative; 2) Forms spores; 3) Rod-shaped.

ANSWER: 1 AND 3 (ACCEPT: GRAM NEGATIVE; ROD SHAPED)

BONUS Biology - Short Answer Identify all of the following four embryonic brain regions that develop from the forebrain: 1) Telencephalon; 2) Mesencephalon; 3) Metencephalon; 4) Diencephalon.

ANSWER: 1 AND 4

BONUS Biology - Short Answer Identify all of the following four embryonic brain regions that develop from the forebrain: 1) Telencephalon; 2) Mesencephalon; 3) Metencephalon; 4) Diencephalon.

ANSWER: 1 AND 4

(General Science) [bonus] Short Answer By number or name, identify all of the following blood measures that would suggest a woman is deficient in iron: 1) low hematocrit; 2) larger than average size of erythrocytes; 3) high levels of TSH; 4) increased transferrin level

ANSWER: 1 AND 4 (ACCEPT: 1 - LOW HEMATOCRIT; 4 - INCREASED TRANSFERRIN LEVEL

[bonus] Short Answer By number or name, identify all of the following 4 processes that contribute directly to genetic variation in humans: 1) random fertilization of ova with sperm; 2) crossing over; 3) independent assortment; 4) mitosis

ANSWER: 1, 2 AND 3 (ACCEPT: ALL BUT #4)

BONUS Biology - Short Answer Identify all of the following three species that are part of the phylum Mollusca: 1) Blue-ringed octopus; 2) Giant clam; 3) Oyster.

ANSWER: 1, 2, 3

Biology - Short Answer Identify all of the following three statements that are true of fixed action patterns: 1) They are highly instinctive behaviors; 2) Once begun, they always continue until completion; 3) They can be triggered by inappropriate stimuli.

ANSWER: 1, 2, 3

Biology - Short Answer Identify all of the following three conditions that negatively impact short term memory: 1) Post traumatic stress disorder; 2) Aphasia [ah-FAY-sjah]; 3) Alzheimer's.

ANSWER: 1, 2, 3 (ACCEPT: ALL OF THEM)

BONUS BIOLOGY Short Answer Identify all of the following five structures which can be found on paramecia [par-uh-MEE-see-uh]: 1) oral groove, 2) cilia, 3) contractile vacuoles, 4) flagella, 5) trichocysts [TRIK-uh-sists].

ANSWER: 1, 2, 3, 5 (ACCEPT: ORAL GROOVE, CILIA, CONTRACTILE VACUOLES, TRICHOCYSTS)

Biology - Short Answer Identify all of the following four cellular attachments that can be found in animal cells: 1) Tight junction; 2) Desmosome; 3) Plasmodesmata [plazz-moh-dezMAH-tah]; 4) Adherens junction.

ANSWER: 1, 2, AND 4

BONUS BIOLOGY Short Answer Of the following four choices, identify all that are involved in plant primary growth: 1) axillary bud meristem, 2) cork cambium, 3) apical meristem, 4) vascular cambium.

ANSWER: 1, 3 (ACCEPT: AXILLARY BUD MERISTEM, APICAL MERISTEM)

BONUS BIOLOGY Short Answer Name all of the following five organisms that lack vessel elements as conducting cells: 1) Norway spruce, 2) Dandelion, 3) Eastern white pine, 4) White fir, 5) Kousa dogwood.

ANSWER: 1, 3, 4 (ACCEPT: NORWAY SPRUCE, EASTERN WHITE PINE, WHITE FIR)

BONUS BIOLOGY Short Answer Identify all of the following five choices that are diploid: 1) spermatogonium [spur-mat-uh-GOH-nee-uhm], 2) spermatids, 3) secondary spermatocyte [spuhr-MAT-uh-syt], 4) primary spermatocyte, 5) mature sperm cells.

ANSWER: 1, 4 (ACCEPT: SPERMATOGONIUM, PRIMARY SPERMATOCTYE)

BONUS BIOLOGY Short Answer By number, order the following five cell layers in a non-woody eudicot [yoo-DY-kot] stem from most lateral to most medial: 1) epidermis [ep-i-DER-mis], 2) vascular cambium [KAM-bee-uhm], 3) pith, 4) primary phloem [FLOH-em], 5) primary xylem [ZY-luhm].

ANSWER: 1, 4, 2, 5, 3

BONUS BIOLOGY Short Answer With regard to the maturation of an erythrocyte [ih-RITH-ruh-syt], arrange the following four choices in correct sequence starting with the cell type after the proerythroblast [proh-i-RITH-ruh-blast] and ending with the mature erythrocyte: 1) Basophilic [bay-suh-FIL-ik] erythroblast, 2) Reticulocyte [ri-TIK-yoo-luh-syt], 3) Orthochromatic [ohrthuh-kroh-MAT-ik] normoblast, 4) Polychromatophilic [poh-lee-kroh-muh-tuh-fil-ik] erythroblast.

ANSWER: 1, 4, 3, 2 (ACCEPT: BASOPHILIC ERYTHROBLAST, POLYCHROMATOPHILIC ERYTHROBLAST, ORTHOCHROMATIC NORMOBLAST, RETICULOCYTE)

BONUS BIOLOGY Short Answer If a plant that is heterozygous for each of three independentlym assorting traits is allowed to self-fertilize, what proportion of the offspring would be heterozygous for all three traits?

ANSWER: 1/8

BONUS GENERAL SCIENCE Short Answer Give the respective SI multipliers for the following prefixes: exa; peta; and giga

ANSWER: 10^18; 10^15; 10^9

[bonus] Short Answer If 2n is the number of chromosomes and 2s is the DNA content of a diploid human cell in G1 of the cell cycle, what are the n and s values, respectively, of a human spermatogonia at the end of telophase one of meiosis?

ANSWER: 1N AND 2S

BIOLOGY Short Answer How many embryonic seed leaves does a bean seedling have?

ANSWER: 2

BONUS BIOLOGY Short Answer For every carbon dioxide molecule fixed in the Calvin-Benson cycle, how many NADPH molecules are utilized?

ANSWER: 2

BONUS BIOLOGY Short Answer An F1 individual plant is heterozygous for each of four traits and is allowed to self-fertilize. The plant produces 128 offspring. How many of the offspring would you expect to have the genotype AaBBccDd [big A little a big B big B little c little c big D little d]?

ANSWER: 2

Biology - Short Answer Identify all of the following three choices that represent dangers to biodiversity: 1) Allowing fallow farmland to regrow with weeds; 2) Importing an Australian moth to deal with pests on farmland; 3) Relocating mountain lions that have moved into human habitats.

ANSWER: 2

Biology - Short Answer Identify all of the following three structures that are examples of convergent evolution: 1) Whale flippers and human hands; 2) Butterfly wings and bird wings; 3) Spider pedipalps and shrimp antennae.

ANSWER: 2

Short Answer: Organisms that have radial symmetry and belong to the Radiata typically have how many germ layers?

ANSWER: 2

BONUS BIOLOGY Short Answer Identify all of the following four cranial nerves that have a purely somatic motor function: 1) Trigeminal [try-JEM-uh-nl], 2) Trochlear [TROK-lee-er], 3) Hypoglossal, 4) Glossopharyngeal [glos-oh-fuh-RIN-jee-uhl].

ANSWER: 2 AND 3 (ACCEPT: TROCHLEAR, HYPOGLOSSAL)

BIOLOGY Short Answer In angiosperm seed development, what fuses with the second sperm nucleus to form the endosperm?

ANSWER: 2 POLAR NUCLEI (ACCEPT: 2 OR BOTH POLAR NUCLEI or POLAR NUCLEI) (DO NOT ACCEPT: POLAR NUCLEUS)

BONUS \BIOLOGY Short Answer Polymerase [POL-uh-muh-rays] chain reaction (PCR) is a widely used method to rapidly clone pieces of DNA. Place the following three steps of PCR in order for a single amplification cycle: 1) Annealing, 2) Denaturation, 3) Elongation.

ANSWER: 2, 1, 3 (ACCEPT: DENATURATION, ANNEALING, ELONGATION)

BONUS BIOLOGY Short Answer Order the following three human oxygen-binding proteins from highest to lowest affinity for oxygen: 1) fetal hemoglobin, 2) myoglobin [my-uh-GLOH-bin], 3) maternal hemoglobin.

ANSWER: 2, 1, 3 (ACCEPT: MYOGLOBIN, FETAL HEMOGLOBIN, MATERNAL HEMOGLOBIN)

BONUS Biology - Short Answer Order the following four organisms from least to greatest in terms of the number of genes in their genomes: 1) S. cerevisiae [sehr-u h-v is-ee-ay]; 2) E. coli; 3) D. melanogaster; 4) H. sapiens.

ANSWER: 2, 1, 3, 4

BONUS BIOLOGY Short Answer By number, identify all of the following four phrases that would NOT be indicative of a healthy stream ecosystem: 1) woody riparian zone that is equal to or greater than stream width; 2) abundant algal mats in stream channel; 3) stream channel with abundant large rocks and old logs; 4) banks unstable with easily disturbed loose soil.

ANSWER: 2, 4

BONUS BIOLOGY Short Answer By number, identify all of the following five choices that are effects of stimulation of the sympathetic divisions of the autonomic nervous systems: 1) constricts eye pupil; 2) relaxes bronchi [BRONG-kee]; 3) stimulates saliva secretion; 4) inhibits urination; 5) stimulates norepinephrine [nohr-ep-uh-NEF-rin] release.

ANSWER: 2, 4, 5

BIOLOGY Short Answer The extracellular matrix protein, collagen, is composed of intertwined helices. How many protein helices come together to form collagen?

ANSWER: 3

BONUS BIOLOGY Short Answer In the standard genetic code, how many of the 64 codons signal for stop of translation?

ANSWER: 3

[bonus] Short Answer If the flower color red is dominant and white is recessive, what will be the expected phenotypic ratio of red to white flowers in the first generation of a cross between parent plants both heterozygous for flower color?

ANSWER: 3 TO 1

BONUS BIOLOGY Short Answer DNA replication involves many different steps. By number, arrange the following four steps in order from start to finish: 1) DNA polymerase [POL-uh-muhrays] synthesizes a new DNA strand, 2) DNA ligase [LY-gays] repairs nicks in the phosphodiester [fos-foh-dy-ES-tuhr] backbone, 3) replication bubble opens at origin, 4) primase builds the primer.

ANSWER: 3, 4, 1, 2

BONUS BIOLOGY Short Answer Arrange the following four groups from the taxa containing the largest number of known living species to the group containing the least: 1) Amphibians, 2) Mammals, 3) Fishes, 4) Birds.

ANSWER: 3, 4, 1, 2 (ACCEPT: FISHES, BIRDS, AMPHIBIANS, MAMMALS)

BONUS BIOLOGY Short Answer By number, indentify all of the following five characteristics that apply to the iris plant: 1) floral parts in multiples of four or five, 2) root system with a central tap root, 3) parallel venation on leaves, 4) vascular bundles in stem arranged in a ring, 5) one cotyledon [kat-uhl-EED-uhn].

ANSWER: 3, 5

BIOLOGY Short Answer What is the most common name for the terminus of a tRNA molecule to which amino acids are attached?

ANSWER: 3-PRIME TERMINUS (ACCEPT: CCA 3-PIME END or 3-PRIME END or 3-PRIME)

BONUS BIOLOGY Short Answer How many pairs of chromosomes does a fruit fly have?

ANSWER: 4

Short Answer During meiosis, how many chromatids are in a chromosomal tetrad?

ANSWER: 4

BONUS BIOLOGY Short Answer By number, place the following steps necessary for cloning a gene into bacteria from genomic DNA in order from first to last: 1) Cut the target gene and the cloning vector with restriction enzymes; 2) Transform bacteria with the plasmid; 3) Ligate the target gene and the cloning vector with DNA ligase; 4) Amplify the target gene using PCR.

ANSWER: 4, 1, 3, 2

BONUS BIOLOGY Short Answer Arrange proximally to distally the following four components of a bird's contour feather: 1) superior umbilicus [uhm-BIL-i-kuhs], 2) rachis [RAY-kis], 3) calamus, 4) inferior umbilicus.

ANSWER: 4, 3, 1, 2 (ACCEPT: INFERIOR UMBILICUS, CALAMUS, SUPERIOR UMBILICUS, RACHIS)

BONUS BIOLOGY Short Answer By number, order the following five stages of animal development from earliest to latest: 1) gastrulation, 2) organogenesis, 3) fertilization, 4) gametogenesis, 5) cleavage.

ANSWER: 4, 3, 5, 1, 2

BONUS BIOLOGY Short Answer For a DNA template that reads 3' [3 prime] TAC AAA ATC 5' [5 prime], give the sequence of the RNA transcribed from it with appropriate polarity.

ANSWER: 5' AUG UUU UAG 3'

BONUS BIOLOGY Short Answer Arrange the following six layers found in a one-year old woody sapling from most lateral to most medial: 1) Cortex, 2) Phloem [FLOH-em] fibers, 3) Pith, 4) Secondary phloem, 5) Periderm, 6) Secondary xylem [ZY-luhm].

ANSWER: 5, 1, 2, 4, 6, 3 (ACCEPT: PERIDERM, CORTEX, PHLOEM FIBERS, SECONDARY PHLOEM, SECONDARY XYLEM, PITH)

BONUS BIOLOGY Short Answer In adult humans, how many bones articulate with the occipital bone?

ANSWER: 6

[bonus] Short Answer For a dihybrid cross between parent plants that are each heterozygous for both traits, if there is complete dominance and independent assortment, what is the total fraction of the offspring that will have a recessive phenotype for either trait and the dominant for the other?

ANSWER: 6/16 (ACCEPT: 3/8) (Solution: 3/16 + 3/16)

Short Answer How many codons in the entire genetic code call for amino acids?

ANSWER: 61

Short Answer: How many walking legs are found in arachnids?

ANSWER: 8

BONUS BIOLOGY Short Answer If a germ cell of a genus of grass has 48 chromosomes at metaphase two, how many chromosomes will a germ cell of this grass have in anaphase-two?

ANSWER: 96

BONUS BIOLOGY Short Answer What is the name of the plant hormone, produced in older leaves, root caps, and stems, which promotes bud and seed dormancy?

ANSWER: ABSCISIC ACID (ACCEPT: ABSCISIN II AND DORMIN)

BONUS BIOLOGY Short Answer What is the technical name of the process, usually occurring in the fall, through which deciduous plants systematically shed their leaves? A thin layer of cells may be produced during this process, thus further sealing the site of detachment.

ANSWER: ABSCISSION

TOSS-UP Biology - Short Answer What group of light-absorbing compounds, including chlorophyll b and carotenoids, are found in photosynthetic organisms and work with chlorophyll a to expand the action spectrum of the organism?

ANSWER: ACCESSORY PIGMENTS

BONUS BIOLOGY Short Answer Name all of the following 3 transport systems that directly require energy to operate: active transport; facilitated diffusion; sodium potassium pump

ANSWER: ACTIVE TRANSPORT; SODIUM POTASSIUM PUMP (ACCEPT: ALL BUT FACILITATED DIFFUSION)

BONUS Biology - Short Answer Cadherins [k uh-deer-ins] are components of what type of cell junction?

ANSWER: ADHERENS JUNCTIONS

BONUS BIOLOGY Short Answer Most corticosteroids are synthesized in what specific part of the human body?

ANSWER: ADRENAL CORTEX (ACCEPT: ADRENAL GLANDS)

[BONUS] Short Answer: Most corticosteroids are synthesized in what specific part of the human body?

ANSWER: ADRENAL CORTEX (ACCEPT: ADRENAL GLANDS)

BIOLOGY Short Answer Name the genus of mosquito that most frequently serves as the vector for the debilitating dengue [DENG-gay] fever.

ANSWER: AEDES

BIOLOGY Short Answer What is the commercially important gelatinous [juh-LAT-n-uhs] substance that is produced by Gelidium amansii?

ANSWER: AGAR

BIOLOGY Short Answer What is the common name of the seabird that has the largest wingspan of any living bird, up to 11.5 feet?

ANSWER: ALBATROSS (ACCEPT: WANDERING ALBATROSS)

BIOLOGY Short Answer To prepare a tincture of iodine the iodine is mixed in what general type of solvent?

ANSWER: ALCOHOL

BIOLOGY Short Answer One group of carbohydrates is known as polyhydroxylated aldehydes. By what other term is this group described?

ANSWER: ALDOSE

BIOLOGY Short Answer By words or numbers, name all of the following 3 functions that meiosis accomplishes: 1) reducing the copies of alleles in the ovum and sperm 2) reducing the numbers of chromosomes in the ovum and sperm 3) increasing genetic diversity through chromosomal exchange events

ANSWER: ALL

BONUS BIOLOGY Short Answer Name all of the following 3 groups of organisms that generate ATP by chemiosmosis (read as: chem-ee-oz-MOE-sis): Cyanobacteria; plant cells; animal cells

ANSWER: ALL

[bonus] Short Answer Name all of the following 4 physiological inputs that affect the homeostatic actions of the hypothalamus in humans: limbic system; olfactory system; retina; reticular formation

ANSWER: ALL

BONUS BIOLOGY Short Answer What is the process in which a protein inhibits the function of a second protein by binding to the regulatory site of that second protein, thus altering its functionality?

ANSWER: ALLOSTERIC REGULATION (ACCEPT: ALLOSTERIC INHIBITION)

BONUS BIOLOGY Short Answer In fruit flies, one gene called fruitless is responsible for male courtship behavior. The gene is transcribed in both males and females, but due to differences in intron usage, is only translated as a full-length protein in males. A genetic XX female will attempt to court other females if she expresses the male form of the fruitless gene. What molecular process would have to be accurately executed to assure that the male form of the gene is expressed only in males?

ANSWER: ALTERNATIVE SPLICING (ACCEPT: ALTERNATIVE RNA PROCESSING OR SPLICING)

BIOLOGY Short Answer What are the common names for the 2 functional groups that combine to form a peptide bond?

ANSWER: AMINO; CARBOXYL (ACCEPT: AMINE AND CARBOXYLIC ACID)

BONUS Biology - Short Answer Fish extrude nitrogenous waste in what toxic form?

ANSWER: AMMONIA

BIOLOGY Short Answer Insect symbionts are represented in all EXCEPT which of the following three groups: 1) bacteria, 2) fungi, or 3) animals?

ANSWER: ANIMALS (ACCEPT: 3)

BONUS 14) BIOLOGY Short Answer Name the structure to which the following terms refer: stylate [STY-layt], lamellate [luh-MEL-ayt], filiform [FIL-uh-fohrm], clavate [KLAY-vayt] and plumose [PLOO-mohs].

ANSWER: ANTENNA

BIOLOGY Short Answer What is the MOST common anatomical synonym for ventral, when locating a part of the human body in relation to another part?

ANSWER: ANTERIOR

BONUS Biology Short Answer: What is the most common term for the substance recognized as foreign and able to elicit antibody production

ANSWER: ANTIGEN

BONUS BIOLOGY Short Answer What person, in 1676, reported to the Royal Society of London the first known observations of protozoans?

ANSWER: ANTON VAN LEEUWENHOEK (ACCEPT: LEEUWENHOEK)

BIOLOGY Short Answer What is the term for the process of programmed cell death that may occur in multicellular organisms?

ANSWER: APOPTOSIS

Biology - Short Answer What is the process by which cells that have reached the end of their functional life span undergo programmed cell death?

ANSWER: APOPTOSIS

BONUS BIOLOGY Short Answer What scientific term describes strategies, such as black widow coloration, used by some animals to alert or warn other animals of their presence and potential danger?

ANSWER: APOSEMATISM (ACCEPT: APOSEMATIC)

BONUS Biology Short Answer Name all of the following 4 plants that produce seeds with a fleshy covering that are eaten by animals and dispersed in their droppings: apple; orchid; cherry; sphagnum moss

ANSWER: APPLE; CHERRY

BONUS BIOLOGY Short Answer Name all of the following 4 plants that produce seeds with a fleshy covering that are eaten by animals and dispersed in their droppings: apple; orchid; cherry; sphagnum moss

ANSWER: APPLE; CHERRY

BONUS Biology - Short Answer What generic group of transport proteins facilitates the diffusion of water across membranes?

ANSWER: AQUAPORIN

BIOLOGY Short Answer What disease occurs when blood vessel walls harden and blood passage is decreased or fully blocked?

ANSWER: ARTERIOSCLEROSIS (ACCEPT MYOCONDITIS)

BIOLOGY Short Answer What is the scientific name of the phylum characterized by the presence of jointed appendages?

ANSWER: ARTHROPODA (DO NOT ACCEPT: ARTHROPODS)

BONUS BIOLOGY Short Answer Name an allosteric [al-uh-STER-ik] inhibitor of pyruvate [pyROO-vayt] kinase.

ANSWER: ATP (ACCEPT: ADENOSINE TRIPHOSPHATE, ALANINE OR ACETYL-CoA)

[bonus] Short Answer Order the following 4 storage forms of energy from the most readily available to humans to the most difficult to mobilize: adipose stores; ATP; glycogen; glucose

ANSWER: ATP; GLUCOSE; GLYCOGEN; ADIPOSE STORES

BIOLOGY Short Answer What is the most common term for the method Louis Pasteur used to create the first rabies vaccine?

ANSWER: ATTENUATION

Short Answer What base triplet is the start codon for methionine (read as: methEYE-oh-neen)?

ANSWER: AUG

BONUS BIOLOGY Short Answer What plant hormone is primarily responsible for apical dominance in angiosperms?

ANSWER: AUXIN (ACCEPT: INDOLE-ACETIC ACID or IAA)

BIOLOGY Short Answer What is the scientific term given to the shape of the E. coli bacterium?

ANSWER: BACILLUS (ACCEPT: ROD or BACILLI or BACILLIFORM)

Biology Short Answer: What is the scientific term given to the shape of the E. coli bacterium?

ANSWER: BACILLUS (ACCEPT: ROD or BACILLI or BACILLIFORM)

BIOLOGY Short Answer Give the genus and species name for the first bacterium shown to be the cause of a disease:

ANSWER: BACILLUS ANTHRACIS

BONUS 2) Biology - Short Answer What is the term used to describe a virus that attacks bacteria?

ANSWER: BACTERIOPHAGE (ACCEPT: PHAGE)

BIOLOGY Short Answer What is the unit in which the lengths of gene-coding DNA sequences are measured?

ANSWER: BASE PAIR (ACCEPT: BP)

BONUS BIOLOGY Short Answer What is the name of the principle that describes the asymmetric fitness pressures between males and females that can lead to sexual selection and evolution?

ANSWER: BATEMAN'S

BONUS BIOLOGY Short Answer What is the most common term for the form of protection when a non-poisonous frog has the same appearance as a poisonous frog?

ANSWER: BATESIAN MIMICRY (ACCEPT: MIMICRY or WARNING MIMICRY) (DO NOT ACCEPT: MULLERIAN MIMICRY)

BIOLOGY Short Answer Moths have evolved high-frequency hearing as a defense against which organisms?

ANSWER: BATS

BIOLOGY Short Answer The following four terms would apply to what group of organisms: primaries, axillars, coverts, scapulars?

ANSWER: BIRDS

[Bonus] Short Answer: Name all of the following 4 choices that are considered part of the connective tissue: blood; adipocytes (read as: ADD-ih-poe-sites); muscles; nerves

ANSWER: BLOOD; ADIPOCYTES

Short Answer Which one of the following 4 animal groups is LEAST closely related to the others: brachiopods; gastropods; cephalopods; bivalves

ANSWER: BRACHIOPODS

BIOLOGY Short Answer In mammals, B cells are produced in the bone marrow. What cloacal [kloh-AY-kuhl] organ in birds produces these cells and is the source of their name?

ANSWER: BURSA OF FABRICIUS (ACCEPT: BURSA)

BONUS BIOLOGY Short Answer In Labrador retrievers, coat color is determined by two sets of genes on different loci [LOH-sy]. At one locus, the upper case B allele [uh-LEEL] codes for black coat color, and the lower case b allele codes for brown coat pigment. At the second locus, the upper case E allele codes for the presence of a transporter to carry pigment to the hair shaft and the lower case e allele codes for the absence of the transporter. In the presence of transporter, the dog is black or brown; in the absence of transporter, the dog is yellow. What is the genotype of a yellow dog that is heterozygous for the brown allele?

ANSWER: Bbee (ACCEPT: bBee)

BIOLOGY Short Answer Intestinal absorption of what vitamin is most directly dependent on intrinsic factor?

ANSWER: B₁₂

BIOLOGY Short Answer In muscle cells, the sarcoplasmic reticulum [sahr-kuh-PLAHZ-mik ri-TIK-yuh-luhm] is an important organelle that permits the muscle cell to contract. It acts to store a particular chemical until it is electrically excited. What chemical is stored in the sarcoplasmic reticulum?

ANSWER: CALCIUM (ACCEPT: CALCIUM ION OR CA+2)

[Bonus] Short Answer: What specific ion is most central to muscle contraction in striated human muscle?

ANSWER: CALCIUM (ACCEPT: CALCIUM ^(2+) or Ca^(2+))

BIOLOGY Short Answer What is the primary calcium-binding protein in eukaryotic cells, that is about 148 amino acids long, and has a wide variety of functions often through kinases and phosphatases (read as: foss-fah-TAY-sis)?

ANSWER: CALMODULIN

BIOLOGY Short Answer In what part of photosynthesis, named after its discoverers, is NADPH used to produce 3-carbon sugar phosphate molecules?

ANSWER: CALVIN CYCLE (ACCEPT: CALVIN-BENSON CYCLE or CALVIN-BENSONBASSHAM CYCLE)

Biology - Short Answer What photosynthetic pathway, named after its discoverers, is also known as the light-independent pathway?

ANSWER: CALVIN-BENSON CYCLE (ACCEPT: CALVIN CYCLE)

BONUS BIOLOGY Short Answer What are two specific types of glycocalyx [gly-koh-KAY-liks] in bacteria?

ANSWER: CAPSULE, SLIME LAYER

BIOLOGY Short Answer If one end of a single polypeptide chain has an amino group exposed, what functional group is typically exposed at the opposite end?

ANSWER: CARBOXYL

BIOLOGY Short Answer The stigma, style, and ovary make up what part of a plant?

ANSWER: CARPEL (ACCEPT: PISTIL)

BONUS BIOLOGY Short Answer What is the most common biochemical term for the degradative phases of metabolism?

ANSWER: CATABOLISM

BONUS Biology - Short Answer What group of monoamine hormones is derived from tyrosine?

ANSWER: CATECHOLAMINE

Biology - Short Answer What group of monoamine hormones is derived from tyrosine?

ANSWER: CATECHOLAMINE

TOSS-UP11 ) Biology - Short Answer Some herbivores, such as rabbits and horses, possess the ability to digest cellulose with the aid of bacteria located in a pouch at the beginning of the large intestine. What is the name of this pouch?

ANSWER: CECUM

[bonus] Short Answer In the Amanita mushroom, what is the name of the cell in which karyogamy and meiosis occur and what is the product of meiosis, respectively?

ANSWER: CELL = BASIDIUM; PRODUCT = BASIDIOSPORE

BONUS BIOLOGY Short Answer Beta-lactams, the class of antibiotics to which penicillin belongs, interfere with bacterial synthesis of what cellular structure in bacteria?

ANSWER: CELL WALL

Short Answer: What is typically the largest organelle of a mature plant cell and can comprise up to 80% of the cell's volume?

ANSWER: CENTRAL VACUOLE (ACCEPT: VACUOLE)

BONUS Biology - Short Answer What structures, composed of nine sets of triplet microtubules arranged in a ring, play a role in eukaryotic cell division, but are not present in plant cells?

ANSWER: CENTRIOLES

[BONUS] Short Answer: Name all of the following 4 cellular structures that are NOT typically found in diploid higher plant cells: centrioles; flagella; lysosomes; plasmodesmata

ANSWER: CENTRIOLES; FLAGELLA; LYSOSOMES (Solution: flagella are found in certain plants sperm cells; centrioles in some algae)

BIOLOGY Short Answer What biome is characterized by dense evergreen shrubs and small trees, usually in the Mediterranean region of the world?

ANSWER: CHAPARRAL (ACCEPT SHRUBLANDS, SCRUB, BRUSH)

BONUS Biology - Short Answer What specific cell type in the stomach is responsible for production of pepsinogen?

ANSWER: CHIEF CELLS

BIOLOGY Short Answer What characteristic of male crickets do organisms of the genus Ormia use in order to find the crickets and lay their eggs on and around them?

ANSWER: CHIRPING (ACCEPT: SINGING, CALLING)

[bonus] Short Answer Name all of the following 4 choices that are complex carbohydrates: chitin (read as: KIE-tin); amylase; cellulose; glycogen

ANSWER: CHITIN; CELLULOSE; GLYCOGEN

Biology - Short Answer The deficiency of magnesium causes what disorder in plants, characterized by the leaves turning pale yellow?

ANSWER: CHLOROSIS

BIOLOGY Short Answer What is the common name for the 27-carbon lipid whose synthesis is in part regulated by HMG CoA (read as: H, M, G, koh, A) reductase and is the basic starting point for estrogen synthesis?

ANSWER: CHOLESTEROL

[bonus] Short Answer Name all of the following 4 molecules that are typically NOT found in plant cell plasma membranes: phospholipids; integral proteins; cholesterol; cellulose

ANSWER: CHOLESTEROL; CELLULOSE (Solution: cellulose is in cell wall, not cell plasma membrane)

[bonus] Short Answer Name all of the following 3 metabolic pathways where substrate-level phosphorylation occurs: citric acid cycle; glycolysis; electron transport system

ANSWER: CITRIC ACID CYCLE; GLYCOLYSIS

[BONUS] Short Answer: Name all of the following 4 organisms that are bivalves: clam; oyster; mussel; snail

ANSWER: CLAM; OYSTER; MUSSEL

BONUS BIOLOGY Short Answer Name all of the following five insect orders that have adults with chewing mouthparts: 1) Hemiptera [hi-MIP-ter-uh], 2) Strepsiptera [strep-sip-TER-uh], 3) Homoptera, 4) Lepidoptera [lep-i-DOP-ter-uh], 5) Coleoptera [koh-lee-OP-ter-uh].

ANSWER: COLEOPTERA (ACCEPT: 5)

BONUS BIOLOGY Short Answer Which one of the basic 3 plant cell types has irregularly thickened cell walls, retains its cytoplasm, and functions in structural support?

ANSWER: COLLENCHYMA

BIOLOGY Short Answer Recently, the number of American bee colonies has been seriously diminished due to sudden, massive, unexplained die-offs. The agent of this epidemic has not yet been conclusively identified. What is the name of this disease?

ANSWER: COLONY COLLAPSE DISORDER (ACCEPT: CCD)

BIOLOGY Short Answer On a phylogenetic [fy-LOJ-uh-neek] tree, two species are grouped in the same clade. What is represented by the branch point that joins these two species?

ANSWER: COMMON ANCESTOR

BONUS Biology - Short Answer What type of microscopy uses a spatial pinhole and point illumination to eliminate out-of-focus light and is commonly used to visualize specimens thicker than the focal depth of the lens?

ANSWER: CONFOCAL MICROSCOPY

BONUS BIOLOGY Short Answer What term, derived from the Greek words for feces and rock, applies to fossilized animal dung?

ANSWER: COPROLITE

BIOLOGY Short Answer Name the circulatory adaptation found in the legs of ducks and geese that allows them to swim in water that is near freezing and yet not become hypothermic.

ANSWER: COUNTERCURRENT EXCHANGER (ACCEPT: COUNTERCURRENT HEAT EXCHANGE, COUNTERCURRENT)

BIOLOGY Short Answer Name the taxonomic order in which the gharial [GUHR-ee-uhl] or gavial [GAY-vee-uhl], Gavialis gangeticus [ga-vee-AL-is gan-JET-uh-kuhs], is found.

ANSWER: CROCODILIA

Short Answer In plants, water molecules are generally protected from water loss to the outside environment by what lipid-based structure covering the epidermis?

ANSWER: CUTICLE

BONUS BIOLOGY Short Answer Name all of the following 4 plants that produce seeds: cycads; ferns; mosses; conifers

ANSWER: CYCADS; CONIFERS

BIOLOGY Short Answer What is the most common name for the first membrane-associated molecule discovered to function in a wide variety of cells in what is commonly referred to as a second messenger?

ANSWER: CYCLIC AMP (ACCEPT: cAMP)

BONUS Biology - Short Answer What common second messenger is released after epinephrine binds to liver GPCRs and was the subject of the 1994 Nobel Prize?

ANSWER: CYCLIC AMP (ACCEPT: cAMP)

BONUS 7) BIOLOGY Short Answer Which amino acid participates in the formation of disulfide bonds?

ANSWER: CYSTEINE

BIOLOGY Short Answer What is the most common autosomal recessive lethal disease in the U.S. among Caucasians?

ANSWER: CYSTIC FIBROSIS

BONUS BIOLOGY Short Answer What recessive, life-threatening disorder affects the cells that produce mucus, sweat, and digestive juices, rendering secretions thick and sticky?

ANSWER: CYSTIC FIBROSIS (ALSO ACCEPT CF OR MUCOVISCIDOSIS)

BIOLOGY Short Answer What is the MOST common term for the division of a somatic cell into 2 nucleated cells following of mitosis?

ANSWER: CYTOKINESIS

Short Answer: What is the MOST common term for the division of a somatic cell into 2 nucleated cells following of mitosis?

ANSWER: CYTOKINESIS

BIOLOGY Short Answer What class of hormone, often produced in a plant's roots, induces cell division in plant roots and shoots?

ANSWER: CYTOKININ

BONUS BIOLOGY Short Answer What are the names for the pyrimidine bases found in DNA?

ANSWER: CYTOSINE; THYMINE

BONUS BIOLOGY Short Answer Name all of the following 4 plants that have their seeds typically dispersed by wind: dandelion; milkweed; maize; lupine

ANSWER: DANDELION; MILKWEED

BONUS BIOLOGY Short Answer Name all of the following 4 plants that have their seeds typically dispersed by wind: dandelion; milkweed; maize; lupine

ANSWER: DANDELION; MILKWEED

(General Science) [bonus] Short Answer Name all of the following that are primarily bacterial diseases: diphtheria; herpes zoster; strep throat; rubella; yellow fever

ANSWER: DIPHTHERIA; STREP THROAT

BIOLOGY Short Answer The fossil record of European black bears provides evidence of adaptive selection. In a population of black bears, identify the specific type of adaptive selection which would allow the average size of European black bears to increase after periods of glaciation but later decrease in size as interglacial periods warmed.

ANSWER: DIRECTIONAL (DO NOT ACCEPT: DIVERSIFYING)

Short Answer Bacterial transduction is primarily caused by the transference of what specific biological substance?

ANSWER: DNA

BIOLOGY Short Answer What enzyme in E. coli functions as the primary proof-reading molecule during replication?

ANSWER: DNA POLYMERASE (ACCEPT: DNA POLYMERASE 3)

BONUS BIOLOGY Short Answer What is the most common name of the protein that synthesizes DNA?

ANSWER: DNA POLYMERASE (ACCEPT: POLYMERASE)

(general science) Short Answer What neurotransmitter is deficient in Parkinson's disease?

ANSWER: DOPAMINE

Biology - Short Answer Thomas Hunt Morgan did his inheritance experimentation on red-eyed and white-eyed mutants of which organism?

ANSWER: DROSOPHILA MELANOGASTER (ACCEPT: FRUIT FLY)

BONUS BIOLOGY Short Answer What is the common name of one mammal species that is not viviparous [vy-VIP-ayr-uhs]?

ANSWER: DUCK BILLED PLATYPUS OR SPINY ANTEATERS (ACCEPT: ECHIDNAS)

BONUS BIOLOGY Short Answer What section of the small intestine is the first to receive gastric contents?

ANSWER: DUODENUM

[BONUS] Short Answer: What is the anatomical term for the specific section of the small intestine into which the pancreatic duct delivers its digestive enzymes?

ANSWER: DUODENUM

BIOLOGY Short Answer What is the term for the process in which a reptile sheds its outer layer of skin, or an arthropod [AHR-thruh-pod] or crustacean [kruh-STAY-shuhn] sheds its outer shell?

ANSWER: ECDYSIS (ACCEPT: MOLTING)

[BONUS] Short Answer: Name all of the following 4 organisms that would be considered to have radial symmetry: flatworms; cestodes; Echinoderms; sea urchins

ANSWER: ECHINODERMS; SEA URCHINS

BIOLOGY Short Answer What is the name for the boundary between two or more ecological communities?

ANSWER: ECOTONE

[Bonus] Short Answer From the OUTSIDE to the INSIDE, name the 3 embryonic germ layers in mammals:

ANSWER: ECTODERM; MESODERM; ENDODERM

[Bonus] Short Answer Name all of the following 3 metabolic processes which directly require oxygen: glycolysis; alcohol fermentation; electron transport system

ANSWER: ELECTRON TRANSPORT SYSTEM

Biology - Short Answer Recombinant DNA can be introduced to eukaryotic cells by briefly shocking them. What is this technique called?

ANSWER: ELECTROPORATION (DO NOT ACCEPT: TRANSFECTION, TRANSFORMATION)

BIOLOGY Short Answer What is the common name of a living species in the order Proboscidea [pro-bos-SID-ee-uh]?

ANSWER: ELEPHANTS (ACCEPT: AFRICAN ELEPHANT OR ASIAN ELEPHANT)

BONUS BIOLOGY Short Answer Name the stage of translation during which peptide bond formation occurs.

ANSWER: ELONGATION

BIOLOGY Short Answer What is the term for the hypothesized process by which eukaryotes [yoo-KAR-ee-ohts] evolved a photosynthetic [foh-tuh-SIN-thuh-tik] organelle?

ANSWER: ENDOSYMBIOSIS

BIOLOGY Short Answer What is the term for the flap of elastic cartilage that prevents food from entering a human's trachea when swallowing?

ANSWER: EPIGLOTTIS

Short Answer Animal tissues are generally divided into four main types. If three of them are, connective, nervous, and muscle, what is the fourth tissue?

ANSWER: EPITHELIAL (ACCEPT: EPITHELIUM)

[bonus] Short Answer Name all of the following 5 products that are NOT produced by the adult human liver: clotting factors; glycogen; erythropoietin; bile; corticotropin (read as: coretick-oh-TRO-pin)

ANSWER: ERYTHROPOIETIN AND CORTICOTROPIN (Solution: erythropoietin = kidney; corticotropin = pituitary)

[BONUS] Short Answer: Name all of the following 5 organs in the human body that are NOT typically considered part of the respiratory system: nose; bronchi; pharynx; esophagus; cervix

ANSWER: ESOPHAGUS; CERVIX

Biology - Short Answer Fruit ripening is promoted by which plant hormone?

ANSWER: ETHYLENE (ACCEPT: ETHENE)

BIOLOGY Short Answer What is the common name of the unicellular organism that has the following characteristics: 1) it has chloroplasts and can make its own food; 2) it can also ingest food from its environment; 3) it usually lives in ponds or puddles; 4) it moves by a flagellum; 5) it has an eyespot that detects light.

ANSWER: EUGLENA

BIOLOGY Short Answer What is the genus name of the most thoroughly studied bacterium?

ANSWER: Escherichia

[Bonus] Short Answer Which one of the following 4 plant groups is phenotypically and phylogenetically LEAST related to the others: ferns; liverworts; hornworts; mosses

ANSWER: FERNS

Short Answer: What is the most common term for the specific type of cartilage that forms the disks between human vertebrae?

ANSWER: FIBROCARTILAGE

BONUS Biology - Short Answer Bacteria use what cellular structure for motility?

ANSWER: FLAGELLUM

BIOLOGY Short Answer What is the common name of the group of marine spiny-finned fish of the order Heterosomata, all of which, when adult, swim along the sea floor on one side of the body, which is highly compressed and has both eyes on the uppermost side?

ANSWER: FLATFISH

BONUS BIOLOGY Short Answer What is the term for an inanimate object that is capable of transmitting infectious disease from one individual to another?

ANSWER: FOMITE

BONUS Biology - Short Answer What law of muscle contraction states that the stroke volume of the heart increases in response to an increase in the volume of blood filling the heart?

ANSWER: FRANK-STARLING LAW (ACCEPT: STARLING'S LAW, FRANK-STARLING'S LAW OF THE HEART, STARLING'S LAW OF THE HEART)

BONUS Biology - Short Answer What law of muscle contraction states that the stroke volume of the heart increases in response to an increase in the volume of blood filling the heart?

ANSWER: FRANK-STARLING LAW (ACCEPT: STARLING'S LAW, FRANK-STARLING'S LAW OF THE HEART, STARLING'S LAW OF THE HEART)

BONUS BIOLOGY Short Answer Estrogen and progesterone [proh-JES-tuh-rohn] are most directly regulated by which two hormones?

ANSWER: FSH AND LH (ACCEPT: FOLLICLE-STIMULATING AND LUTEINIZING)

BIOLOGY Short Answer Which taxonomic kingdom of organisms has cell walls of chitin [KY-tin]?

ANSWER: FUNGI

BONUS BIOLOGY Short Answer Cells that become quiescent enter G-zero from what stage of the cell cycle?

ANSWER: G-1

BIOLOGY Short Answer From the FIRST to the LAST, what are the stages into which interphase is most commonly divided?

ANSWER: G-1; S; G-2 (ACCEPT: GAP ONE, DNA SYNTHESIS, GAP 2)

BIOLOGY Short Answer During which phase of interphase does expression of proteins needed to prepare for DNA replication increase?

ANSWER: G1

BIOLOGY Short Answer When food reaches the stomach in humans, secretion of hydrochloric acid is stimulated by which hormone?

ANSWER: GASTRIN

BIOLOGY Short Answer Yersinia, Drosophila and Homo are all what taxonomical category?

ANSWER: GENUS (ACCEPT: GENERA)

BONUS BIOLOGY Short Answer Name all of the following 5 plants that are dicots: wheat; geranium; garlic; bamboo; rose

ANSWER: GERANIUM; ROSE

TOSS-UP5) Biology - Short Answer The nervous system consists of neurons and what other type of cell that is responsible for nourishing neurons?

ANSWER: GLIAL [GLEE-AL]CELL (ACCEPT: GLIA)

Biology - Short Answer What is the name of the specific sugar that animal cells use as input into glycolysis?

ANSWER: GLUCOSE

BONUS BIOLOGY Short Answer From what monosaccharide is cellulose primarily composed?

ANSWER: GLUCOSE (ACCEPT: Β-D-GLUCOPYRANOSYL or D-GLUCOSE)

[BONUS] Short Answer: What eukaryotic cell organelle primarily functions to modify, sort, and package products for exocytosis?

ANSWER: GOLGI (ACCEPT: GOLGI APPARATUS, COMPLEX or STACK)

BONUS BIOLOGY Short Answer Name all of the following 4 choices that are characteristic of E. coli: gram negative; an enteric organism; non-motile; bacillus )

ANSWER: GRAM NEGATIVE; AN ENTERIC ORGANISM; BACILLUS (ACCEPT: ALL BUT NON-MOTILE

BIOLOGY Short Answer What is the most common color for the calyx (read as: KAL-IX) of angiosperms?

ANSWER: GREEN

(general science) [bonus] Short Answer Lichens are a symbiotic relationship of what 2 general types of organisms?

ANSWER: GREEN ALGAE AND FUNGI (ACCEPT: ALGAE OR CYANOBACTERIA FOR GREEN ALGAE)

(General Science) [bonus] Short Answer A relatively new scourge to the citrus growers in Florida, what is the common name of the citrus disease caused by a species of bacteria called C. liberibacter and spread by an introduced non-native insect?

ANSWER: GREENING (ACCEPT: CITRUS GREENING)

BONUS BIOLOGY Short Answer You are walking at night on a sandy beach in southern California during a high tide that followed the last full moon. What fish will you most likely see mating in the sand on the beach during your walk?

ANSWER: GRUNION (ACCEPT: LEURESTHES)

BONUS BIOLOGY Short Answer What specific cells and how many of them border each stomal pore?

ANSWER: GUARD CELLS; 2

BONUS BIOLOGY Short Answer What specific cells and how many of them border each stomal pore?

ANSWER: GUARD CELLS; 2

BIOLOGY Short Answer What is the term for the type of plant that is adapted to living in a saline environment?

ANSWER: HALOPHYTE

BONUS Biology - Short Answer What model in population genetics predicts that allele frequencies will stay constant from generation to generation, assuming that the population remains free of other evolutionary influences?

ANSWER: HARDY-WEINBERG PRINCIPLE

BONUS BIOLOGY Short Answer Valproic acid and trapoxin are inhibitors of what class of enzymes that regulate gene expression by altering chromatin accessibility?

ANSWER: HDAC (ACCEPT: HISTONE DEACETYLASES; DO NOT ACCEPT: HAT)

BIOLOGY Short Answer The sinoatrial (read as: sine-oh-AYE-tree-al) node is located in what human organ?

ANSWER: HEART

TOSS-UP 2) Biology - Short Answer What type of leukocytes, characterized by their CD4 receptors, secrete cytokines [sigh-toe-kines] that activate nearby B and T cells?

ANSWER: HELPER T CELLS

[bonus] Short Answer Name all of the following 4 human blood cells that are considered lymphocytes: helper T cells; CD8 cells; CD4 cells; plasma cells

ANSWER: HELPER T CELLS; CD8 CELLS; CD4 CELLS; PLASMA CELLS (ACCEPT: ALL)

Short Answer Black water fever is a complication of malarial infection. The term black water comes from what oxidized product in the urine of infected individuals?

ANSWER: HEMOGLOBIN (ACCEPT: MET-HEMOGLOBIN)

BIOLOGY Short Answer In what taxonomic phylum can you find an indeterminate species in the genus Marchantia [mar-CHANT-ee-uh]?

ANSWER: HEPATOPHYTA

BONUS 7) BIOLOGY Short Answer An athlete who normally has plenty of endurance is admitted to the emergency room with symptoms of sickle cell anemia, which include pain crisis in multiple limbs from lack of oxygen as a result of a track competition at high altitude. Suspecting sickle cell trait, doctors decide to screen the athlete's beta-globin gene for sickle-cell mutations. What is the general genetic term that would best describe the athlete's beta-globin genotype of the alleles being tested?

ANSWER: HETEROZYGOUS

BONUS BIOLOGY Short Answer What is the common name for the native annual plant with the following description: It has stems that coil around a support for climbing, the leaves are alternate compound with 3 leaflets, and it has a thin-walled fruit that the Chippewa traditionally used as a food source and digestive aid.

ANSWER: HOGPEANUT (ACCEPT: WILD PEANUT) (DO NOT ACCEPT: PEANUT)

BONUS BIOLOGY Short Answer By name or number, list all of the following four pollinators that are members of the order Hymenoptera (read as: hy-meh-NOHP-teh-rah): 1) ruby-throated hummingbird, 2) monarch butterfly, 3) green bottle fly, 4) honey bee.

ANSWER: HONEY BEE (ACCEPT: 4)

BONUS BIOLOGY Short Answer Give the common name of a family of large Asian birds that has the following unique nesting characteristics: When a female is ready to lay eggs, she enters a hole in a tree and seals off the entrance, leaving just a narrow opening. Once sealed inside, she becomes dependent on the male, who passes food through the hole while she incubates and raises the chicks.

ANSWER: HORNBILLS

BIOLOGY Short Answer In segmented animals, a specific family of genes is particularly important for directing the orientation and placement of structures and segments. These genes encode transcription factors that are able to bind a regulatory region of DNA called the homeobox. Name this gene family.

ANSWER: HOX

BONUS BIOLOGY Short Answer Name all of the following 4 animals for which L-ascorbate is an essential nutrient: humans; dogs; cats; horses

ANSWER: HUMANS

Biology - Short Answer Insects that have two pairs of membranous wings, mobile heads, and chewing or sucking mouthparts, such as ants, bees, and wasps, are classified as part of what order?

ANSWER: HYMENOPTERA

BIOLOGY Short Answer Damage to which human brain region results in anomalous circadian [uh-NOM-uh-luhs sur-kay-dee-uhn] rhythms?

ANSWER: HYPOTHALAMUS

Short Answer What specific area of the human diencephalon is responsible for producing factors that regulate the release of hormones of the pituitary?

ANSWER: HYPOTHALAMUS

BIOLOGY Short Answer What class of immunoglobulins is most directly responsible for hay fever and common allergies in humans?

ANSWER: IGE (ACCEPT: E or IMMUNOGLOBULIN E)

BONUS BIOLOGY Short Answer What are the five major steps in Holozoic nutrition?

ANSWER: INGESTION, DIGESTION, ABSORPTION, ASSIMILATION, EGESTION

BIOLOGY Short Answer Motion of the basilar membrane stimulates what cell type that initiates the transmission of a signal to the brain?

ANSWER: INNER HAIR (ALSO ACCEPT STEREOCILIA OR HAIR CELLS)

BIOLOGY Short Answer What modified base in the anticodon's wobble position can base pair with uracil [YOOR-uh-sil], cytosine [SY-tuh-seen] or adenine [AD-n-een] in the third position of an mRNA codon?

ANSWER: INOSINE

BIOLOGY Short Answer In eukaryotes [yoo-KAR-ee-ohts], the spliceosome [SPLY-see-uhsohm] is an RNA-protein complex important for RNA processing. What does this complex remove?

ANSWER: INTRONS

Biology - Short Answer What chemical element, that occurs in various forms, is used as a mordant in Gram testing, is used as a method of detecting sugar in Lugol's solution, and is required for proper thyroid function?

ANSWER: IODINE

Biology - Short Answer Cytochrome proteins contain prosthetic groups that possess what metal center?

ANSWER: IRON

[bonus] Short Answer Name all of the following 4 organisms that are human ectoparasites: Plasmodium ovale; Ascaris; Trichinella spiralis (read as: Trick-ih-nel-la spear-rah-liss); Ixodes (read as: icks-OH-dees)

ANSWER: IXODES

BIOLOGY Short Answer What specific class of antibodies is most commonly responsible for degranulation of basophils or mast cells?

ANSWER: IgE (ACCEPT: IMMUNOLGLOBULIN E)

BONUS BIOLOGY Short Answer What is the most plentiful class of immunoglobulins in human plasma?

ANSWER: IgG (ACCEPT: G or IMMUNOGLOBULIN G)

Biology - Short Answer What pentameric antibody is generally the first to be secreted during primary immune response and is responsible for promoting agglutination reactions?

ANSWER: IgM (ACCEPT: IMMUNOGLOBULIN M)

BONUS5) Biology - Short Answer What type of population selection would occur in an environment that is very close to its carrying capacity?

ANSWER: K-SELECTION

BIOLOGY Short Answer Who is credited with creating the bionomial system for naming species?

ANSWER: KARL VON LINNE OR CAROLUS LINNEAUS

Biology - Short Answer What is the general name for enzymes that catalyse reactions that involve the transfer of a phosphate group from ATP?

ANSWER: KINASES

BIOLOGY Short Answer What human aneuploid [an-yoo-PLOID] syndrome is characterized by an XXY chromosome composition and sexual immaturity?

ANSWER: KLINEFELTER

[BONUS] Short Answer: What is the common name for the joint that is formed by the articulation of the tibia, the fibula, and the femur?

ANSWER: KNEE

BONUS BIOLOGY Short Answer What is the common name of the large species of lizard found in Indonesia whose unusual size is attributed to island gigantism?

ANSWER: KOMODO DRAGON

BIOLOGY Short Answer The spines of cacti are a modification of what plant structure?

ANSWER: LEAF (ACCEPT: LEAVES)

Biology - Short Answer Testosterone and other androgens are produced by what specialized cells that populate the seminiferous tubules?

ANSWER: LEYDIG CELLS

BONUS BIOLOGY Short Answer During the early stages of the Human Genome Project, fragments of human DNA were placed into vectors called yeast artificial chromosomes (YACs). What replication enzyme would be needed in the test tube to connect the two DNA fragments together, creating recombinant DNA?

ANSWER: LIGASE (ACCEPT: DNA LIGASE)

BONUS BIOLOGY Short Answer What is the term for the organic substance that consists of a dendritic network polymer of phenylpropene units and binds the cells, fibers, and vessels that constitute wood?

ANSWER: LIGNIN

Short Answer What biological substance is the SECOND MOST abundant organic compound on Earth after cellulose, and is especially abundant in sclereids, xylem fibers and tracheids?

ANSWER: LIGNIN

[Bonus] Short Answer Name all of the following 3 classes of biological molecules whose immediate breakdown product can generate energy by feeding into metabolism at the level of pyruvate or acetyl Co-A: lipids; proteins; carbohydrates

ANSWER: LIPIDS; PROTEINS; CARBOHYDRATES (ACCEPT: ALL)

BONUS BIOLOGY Short Answer What human organ produces the majority of immune complement proteins?

ANSWER: LIVER

BONUS22) Biology - Short Answer Microphylls are found in what group of plants?

ANSWER: LYCOPHYTES

BONUS Biology - Short Answer What organelle is defective in inclusion-cell disease, also known as I-cell disease?

ANSWER: LYSOSOME

TOSS-UP22) Biology - Short Answer What cellular organelles are derived from the Golgi apparatus and contain hydrolytic [high-d roh-l it-ic] digestive enzymes

ANSWER: LYSOSOMES

BONUS Biology - Short Answer In C4 photosynthesis, carbon dioxide is fixed to phosphoenolpyruvate [fos-fo h-ee-nol-pie-r oo-vate] instead of RuBisCo, which forms oxaloacetate. Oxaloacetate is then converted into what organic compound before it is transported to the bundle-sheath cells?

ANSWER: MALATE (ACCEPT: MALIC ACID)

BIOLOGY Short Answer Into what specific mitochondrial compartment are hydrogen ions pumped out during chemiosmosis (read as: chem-ee-oz-MOE-sis)?

ANSWER: MATRIX

BONUS BIOLOGY Short Answer Name all of the following 4 diseases that are caused by viruses: measles; AIDS; tuberculosis; leprosy

ANSWER: MEASLES; AIDS

BIOLOGY Short Answer Cutaneous [kyoo-ATY-nee-uhs] sensation to the palm of the hand and thumb is provided primarily by which nerve?

ANSWER: MEDIAN

BONUS Biology - Short Answer What specific type of plant tissue is composed of undifferentiated cells and is found in locations undergoing growth?

ANSWER: MERISTEMATIC TISSUE

(general science) [Bonus] Short Answer What is the medical term for the type of lung cancer specifically associated with asbestosis?

ANSWER: MESOTHELIOMA

BONUS BIOLOGY Short Answer In the production of mammalian proteins, the first amino acid in the polypeptide sequence is almost always the same amino acid. What is this amino acid that is prevalent in almost all proteins?

ANSWER: METHIONINE (ACCEPT: MET AND M)

Biology - Short Answer What cytoskeletal component is composed of actin?

ANSWER: MICROFILAMENTS (ACCEPT: CYTOSKELETAL FILAMENTS, THIN FILAMENTS)

Biology - Short Answer Dynein [DYE-nee-in] and kinesin [kin-EE-sin] are proteins that bind to what cytoskeletal features?

ANSWER: MICROTUBULES

Biology - Short Answer What cellular substructures, characterized by their dynamic instability, go through growing phases known as "rescue" and shrinking phases known as "catastrophe?"

ANSWER: MICROTUBULES

BONUS BIOLOGY Short Answer What is the layer typically found between 2 plant cell walls of adjoining plant cells which is composed of pectin?

ANSWER: MIDDLE LAMELLA

Biology - Short Answer What 1953 experiment, designed to test the Oparin-Haldane hypothesis, sought to emulate the conditions of early Earth in a laboratory?

ANSWER: MILLER-UREY EXPERIMENT

BIOLOGY Short Answer In eukaryotic cells, the citric acid cycle takes place in what cellular organelle?

ANSWER: MITOCHONDRIA

BONUS BIOLOGY Short Answer In what specific part of a eukaryotic cell will pyruvate be converted into acetyl Co-enzyme A?

ANSWER: MITOCHONDRION (ACCEPT: MITOCHONDRIA)

BIOLOGY Short Answer Barbara McClintock studied variation in maize kernel coloration. She found that different patterns of coloration were correlated with chromosome structural changes. She was awarded the 1983 Nobel Prize in physiology or medicine for her discovery. What were the genetic elements she discovered?

ANSWER: MOBILE GENETIC ELEMENTS (ACCEPT: TRANSPOSONS, JUMPING GENES OR MOBILE CONTROLLING ELEMENTS, TRANSPOSABLE ELEMENTS)

BIOLOGY Short Answer What cell in the peripheral blood is the immediate precursor of the macrophage?

ANSWER: MONOCYTE

BONUS BIOLOGY Short Answer By name or number, identify all of the following four organisms that are nonvascular plants: 1) moss, 2) liverwort, 3) St. John's wort, 4) hornwort.

ANSWER: MOSS, LIVERWORT, AND HORNWORT (ACCEPT: 1, 2, AND 4)

[Bonus] (General Science) Short Answer Name all of the following 4 diseases that are NOT caused by bacteria: cholera; bubonic plague; syphilis; mumps

ANSWER: MUMPS

[BONUS] Short Answer: A sarcomere is a basic unit in what specific tissue of mammals?

ANSWER: MUSCLE

BONUS BIOLOGY Short Answer By name or number, identify all of the following four biological processes that rely on the participation of calcium ions: 1) oxygen transport, 2) sweat production, 3) digestion, 4) muscular contraction

ANSWER: MUSCULAR CONTRACTION (ACCEPT: 4)

Biology - Short Answer What are the mating structures of basidiomycetes [bah-s id-ee-o h-mye-seets]called?

ANSWER: MUSHROOMS (ACCEPT: BASIDIOCARPS)

BIOLOGY Short Answer What is the main respiratory pigment of skeletal muscle cells?

ANSWER: MYOGLOBIN

BIOLOGY Short Answer Indicate the name of the common and treatable condition that occurs when a person's eyeball is longer than normal and the lens focuses distant objects in the front of the retina.

ANSWER: MYOPIA (ACCEPT: NEARSIGHTEDNESS)

BONUS BIOLOGY Short Answer Which actin-associated motor protein generates the force for muscle contraction?

ANSWER: MYOSIN

BIOLOGY Short Answer What reduced coenzyme donates its electrons to complex-one in the electron transport system?

ANSWER: NADH

[bonus] Short Answer What is the average yield of ATP for every NADH and for every FADH₂, respectively, that pass through the electron transport system?

ANSWER: NADH = 3; FADH₂ = 2

Biology - Short Answer What genetic analysis tool, freely available from the National Center for Biotechnology Information, allows visitors to compare DNA sequences for similarities?

ANSWER: NCBI BLAST (ACCEPT: BLAST)

Biology - Short Answer Roundworms are part of what invertebrate phylum?

ANSWER: NEMATODA

BONUS BIOLOGY Short Answer Beadle and Tatum primarily used what organism to show that mutations in a single gene caused disruptions in metabolic pathways because of the lack of a single enzyme?

ANSWER: NEUROSPORA (ACCEPT: BREAD MOLD or NEUROSPORA CRASSA)

TOSS-UP9) Biology - Short Answer GABA [gab-a], glycine, and acetylcholine are all examples of what group of signaling molecules?

ANSWER: NEUROTRANSMITTERS

BONUS BIOLOGY Short Answer Name all of the following 4 cells that are leukocytes: neutrophil; basophil; melanocyte; macrophage

ANSWER: NEUTROPHIL; BASOPHIL

(General Science) [bonus] Short Answer What is the common name for vitamin B3 that is a central component of coenzymes NAD and NADP+ ?

ANSWER: NIACIN (ACCEPT: NICOTINIC ACID)

BONUS BIOLOGY Short Answer The acronym NAD stands for what phrase?

ANSWER: NICOTINAMIDE ADENINE DINUCLEOTIDE

BIOLOGY Short Answer Name all of the following 3 choices that are chemoautotrophs: Nitrosomonas (read as: nitro-so-MOAN-us) Paramecium; Volvox

ANSWER: NITROSOMONAS

TOSS-UP5) Biology - Short Answer The nucleus is enveloped by a net-like array of protein filaments that maintain the shape of the nucleus. What is this array known as?

ANSWER: NUCLEAR LAMINA

BIOLOGY Short Answer What is the most common name for the histone octamer used in the coiling and packing of DNA and regulation of gene expression?

ANSWER: NUCLEOSOMES

BONUS BIOLOGY Short Answer Name all of the following 4 organelles that are bounded by a double membrane: nucleus; mitochondrion; lysosome; chloroplast

ANSWER: NUCLEUS; MITOCHONDRION; CHLOROPLAST (ACCEPT: ALL BUT LYSOSOME)

[bonus] Short Answer Name all of the following organelles that are bounded by a double membrane: nucleus; mitochondrion; chloroplast; lysosome

ANSWER: NUCLEUS; MITOCHONDRION; CHLOROPLAST (ACCEPT: ALL BUT LYSOSOME)

BONUS BIOLOGY Short Answer What are the four main human blood types?

ANSWER: O, A, B, AND AB (accept in any order)

BIOLOGY Short Answer What is the name of the process in which the female gametes are formed in an animal?

ANSWER: OOGENESIS

BIOLOGY Short Answer What cell type is MOST directly responsible for secreting the bone matrix in developing humans?

ANSWER: OSTEOBLASTS (DO NOT ACCEPT: OSTEOCYTE)

BONUS BIOLOGY Short Answer What specific human bone cell is antagonistic to osteoblasts and is known to cause apoptosis (read as: A-pop-TOE-sis) of osetoblasts in vitro?

ANSWER: OSTEOCLASTS

BONUS BIOLOGY Short Answer In cladistics [kluh-DIS-tiks], what is the term for a group of organisms used as a standard for comparison in constructing a phylogenetic [fy-luh-JEN-uhtik] tree?

ANSWER: OUTGROUP

BIOLOGY Short Answer Where is intercellular fluid normally found?

ANSWER: OUTSIDE OF CELLS (ACCEPT: IN BETWEEN CELLS OR IN THE INTERSTITIUM)

BIOLOGY Short Answer Which gas was in short supply in the Earth's atmosphere prior to the evolution of the citric acid cycle?

ANSWER: OXYGEN

Short Answer What is the final electron acceptor for electrons passing down the electron transport system?

ANSWER: OXYGEN (ACCEPT: MOLECULAR OXYGEN or O₂)

BONUS BIOLOGY Short Answer During photorespiration, the plant enzyme RuBisCO (read as: rueBIS-ko) will bind what molecular substance rather than carbon dioxide?

ANSWER: OXYGEN (ACCEPT: O2)

BONUS BIOLOGY Short Answer What protein, that takes its name from its apparent molecular weight, can arrest cells in the G1-S checkpoint, initiate apoptosis (read as: aye-pop-TOE-sis) and, when deactivated, is implicated in a wide variety of cancers?

ANSWER: P53

BIOLOGY Short Answer What gene, sometimes called the guardian of the genome, has a central role in cell cycle checkpoints and is the most frequently mutated gene in human cancers?

ANSWER: P53 (ACCEPT: TUMOR SUPPRESSOR P53, PROTEIN 53, TUMOR PROTEIN 53, TUMOR ANTIGEN P53, TRP53, TRANSFORMATION RELATED PROTEIN 53)

BIOLOGY Short Answer From what basic plant cell type are the majority of cells in a corm (read as: KORM) composed?

ANSWER: PARENCHYMA

BIOLOGY Short Answer Name the plant tissue beneath the epidermis that makes up the cells that surround a stoma.

ANSWER: PARENCHYMA

BONUS Biology - Short Answer What specific cell type in the stomach is responsible for production of hydrochloric acid in response to stimulation by gastrin?

ANSWER: PARIETAL CELLS

TOSS-UP 8) Biology - Short Answer When a phylogenetic tree has the fewest number of evolutionary events possible, it is said to embody what principle of simplicity?

ANSWER: PARSIMONY

BONUS BIOLOGY Short Answer By name or number, identify all of the following five animals that are even-toed ungulates: 1) peccaries, 2) camels, 3) goats, 4) bison, 5) horses.

ANSWER: PECCARIES, CAMELS, AND GOATS (ACCEPT: 1, 2, AND 3)

Short Answer The pincers of scorpions are a modification of what arachnid appendage?

ANSWER: PEDIPALPS

Short Answer: What is the botanical term for the stem of a group of flowers?

ANSWER: PEDUNCLE

BONUS BIOLOGY Short Answer Name all of the following 4 substances that would typically be considered exocrine products: insulin; pepsin; hemoglobin; epinephrine

ANSWER: PEPSIN

BONUS BIOLOGY Short Answer Amino acids are joined to one another in linear polymers that fold to form proteins. What is the name of the chemical bond that joins the amino terminus of one amino acid to the carboxyl terminus of the next amino acid in the sequence?

ANSWER: PEPTIDE BOND (ACCEPT: PEPTIDE, AMINE BOND)

Biology - Short Answer Penicillin inhibits the cross-linking of which structural molecule in bacteria?

ANSWER: PEPTIDOGLYCAN

BIOLOGY Short Answer Bonus What is the biological term most often used for the act of a cell engulfing a particle by extending its pseudopodia around the particle?

ANSWER: PHAGOCYTOSIS

BIOLOGY Short Answer What is the most common term used in genetics to describe the observable physical characteristics of an organism caused by the expression of a gene or set of genes?

ANSWER: PHENOTYPE

Biology - Short Answer What term is used to describe the composite of an organism's observable characteristics and traits?

ANSWER: PHENOTYPE

BONUS BIOLOGY Short Answer Sieve cells and sieve tube members are types of what specific conductive plant cell type?

ANSWER: PHLOEM

BONUS BIOLOGY Short Answer What is the general biological term for organisms that synthesize organic molecules from inorganic molecules using light as an energy source?

ANSWER: PHOTOAUTOTROPHS (ACCEPT: PHOTOTROPHS)

BONUS BIOLOGY Short Answer Many angiosperms flower at the same time every year, even though they may have started growing at different times. Their flowering is a response to the changing length of day and night. What is the name of this phenomenon?

ANSWER: PHOTOPERIODISM (ACCEPT: PHOTOPERIODICITY)

BIOLOGY Short Answer What wasteful process occurs when rubisco acts as an oxygenase [OK-si-juh-nays] instead of a carboxylase [kahr-BOK-suh-layz] during the Calvin cycle, decreasing the efficiency of photosynthesis and producing carbon dioxide?

ANSWER: PHOTORESPIRATION

BIOLOGY Short Answer What specific biological process is believed to have produced most of the diatomic oxygen in the Earth's atmosphere?

ANSWER: PHOTOSYNTHESIS

BONUS BIOLOGY Short Answer Name all of the following 3 processes that do NOT occur in E. coli: glycolysis; photosynthesis; glycogenolysis (read as: GLYKO-jen-oh-LY-sis)

ANSWER: PHOTOSYNTHESIS

[BONUS] Short Answer: Most of the molecular oxygen in Earth's atmosphere came from what biological process?

ANSWER: PHOTOSYNTHESIS

BONUS BIOLOGY Short Answer Order the following 5 taxonomic group with the MOST members to the one with the FEWEST members: genus; order; phylum; class; family

ANSWER: PHYLUM; CLASS; ORDER; FAMILY; GENUS

BONUS BIOLOGY Short Answer Using the traditional system of classification, when organisms belong to the same class, name all of the following 4 taxonomic categories to which they must also belong: order; family; phylum; kingdom

ANSWER: PHYLUM; KINGDOM

BONUS BIOLOGY Short Answer In an area originally devoid of life, what is the name for the first community to become established?

ANSWER: PIONEER (DO NOT ACCEPT: FOUNDER)

Short Answer What part of the human brain releases the most corticotropin?

ANSWER: PITUITARY

BONUS BIOLOGY Short Answer Name all of the following 4 cells that typically possess mitochondria: erythrocytes; plasma cells; lymphocytes; neurons

ANSWER: PLASMA CELLS; LYMPHOCYTES; NEURONS (ACCEPT: ALL BUT ERYTHROCYTES)

BONUS BIOLOGY Short Answer Name the two types of cell junctions that connect the cytoplasm of adjacent cells

ANSWER: PLASMODESMATA (ACCEPT: PLASMODESMA) AND GAP JUNCTION (ACCEPT: COMMUNICATING JUNCTION)

BONUS BIOLOGY Short Answer What is the common name of the woody shrub or small tree with the following three characteristics: 1) 7 to 13 leaflets per pinnate leaf, 2) grows in very wet or flooded soils, and 3) contains a resin that causes skin and mucus membrane irritation to humans?

ANSWER: POISON SUMAC

[bonus] Short Answer Order the following 4 choices from the SMALLEST to the LARGEST, assuming average size for each: erythrocyte; mitochondrion; polio virus; macrophage

ANSWER: POLIO VIRUS; MITOCHONDRION; ERYTHROCYTE; MACROPHAGE

BIOLOGY Short Answer What is the most common name for the micro-gametophyte in flowering plants?

ANSWER: POLLEN (ACCEPT: POLLEN GRAINS)

BIOLOGY Short Answer If a trace amount of blood was discovered at a crime scene, what technique could be used to amplify the amount of DNA present in this sample, thereby allowing scientists to determine the individual's blood type?

ANSWER: POLYMERASE CHAIN REACTION (ACCEPT: PCR)

BONUS BIOLOGY Short Answer What is the general name for integral membrane proteins that directly couple the energy of ATP hydrolysis to the movement of molecules against their concentration gradient?

ANSWER: PRIMARY ACTIVE TRANSPORTERS (ALSO ACCEPT: DIRECT ACTIVE TRANSPORT; DO NOT ACCEPT: ACTIVE TRANSPORTERS, SECONDARY ACTIVE TRANSPORTERS)

BIOLOGY Short Answer What is the one type of infectious agent that contains no nucleic acid?

ANSWER: PRION

Short Answer The corpus luteum in humans primarily secretes what hormone?

ANSWER: PROGESTERONE

BIOLOGY Short Answer Of the 20 amino acids encoded by the universal genetic code, only one is technically an imino acid, as the R-group is covalently bonded to the amino terminus to form a nitrogen-containing ring. Unless it is modified, this amino acid also tends to disrupt secondary structures such as alpha-helices and beta sheets if it is found in the middle of them. What is this amino acid?

ANSWER: PROLINE

Biology - Short Answer What type of DNA sequence, an example of which is the TATA box, is required for transcription factor binding and the transcription of DNA to RNA?

ANSWER: PROMOTER

BIOLOGY Short Answer During what phase of meiosis-one do homologous chromosomes exchange genetic information?

ANSWER: PROPHASE-ONE (ACCEPT: PROPHASE)

BIOLOGY Short Answer During what phase of meiosis-one do homologous chromosomes exchange genetic information?

ANSWER: PROPHASE-ONE (ACCEPT: PROPHASE)

[bonus] Short Answer If you wanted to radioactively label protein and DNA to tell which one was present in a certain place, what 2 isotopes would you most likely use for the protein and the DNA, respectively?

ANSWER: PROTEIN = S-35; DNA = P-32 (ACCEPT: SULFUR-35 AND PHOSPHORUS-32) (DO NOT ACCEPT: SULFUR AND PHOSPHOROUS)

BONUS Biology - Short Answer What phylum of gram-negative bacteria includes Rhizobia, Nitrosomonas, and E. coli?

ANSWER: PROTEOBACTERIA

BONUS Biology - Short Answer What phylum of gram-negative bacteria includes Rhizobia, Nitrosomonas, and E. coli?

ANSWER: PROTEOBACTERIA

BIOLOGY Short Answer During cellular respiration, what molecule is produced in the cytoplasm and transported to the mitochondria [my-tuh-KON-dree-uh], where it is converted into acetyl [uh-SET-l] CoA?

ANSWER: PYRUVATE

BIOLOGY Short Answer What is the highest level of protein structure present in collagen, a fibrous protein with three discrete helical polypeptides [HEL-i-kuhl pol-ee-PEP-tyds] supercoiled into a triple helix?

ANSWER: QUATERNARY

BIOLOGY Short Answer What is the name of the phenomenon whereby microorganisms detect and respond to the microbial population density in their immediate surroundings?

ANSWER: QUORUM SENSING

[Bonus] Short Answer Name all of the following 4 choices that are compound fruits: tomato; raspberry; grape; pineapple

ANSWER: RASPBERRY AND PINEAPPLE

BONUS Biology - Short Answer What type of enzyme, produced as part of bacterial immunity against viruses, recognizes palindromic sites on foreign DNA and cleaves them?

ANSWER: RESTRICTION ENZYME

BONUS Biology - Short Answer What autoimmune disease results in damage to cartilage and the bones of joints, resulting in painful inflammation?

ANSWER: RHEUMATOID ARTHRITIS

BONUS BIOLOGY Short Answer Name all of the following 4 choices that are typically considered human autoimmune dysfunctions: Rheumatoid arthritis; Lupus; shingles; Down's syndrome

ANSWER: RHEUMATOID ARTHRITIS; LUPUS

BONUS Biology - Short Answer What botany term is used to describe horizontal shoots that grow just below the surface?

ANSWER: RHIZOMES

BONUS BIOLOGY Short Answer From what vitamin is the electron carrier FAD (read as: F, A, D) derived?

ANSWER: RIBOFLAVIN (ACCEPT: VITAMIN B2)

BIOLOGY Short Answer What is the common name for the units in DNA to which phosphates are covalently linked?

ANSWER: RIBOSE SUGARS (ACCEPT: PENTOSE SUGARS or SUGARS or DEOXYRIBOSE SUGARS)

BONUS BIOLOGY Short Answer Order the following 4 objects found in the human body from the SMALLEST to the LARGEST: mitochondrion; ribosome; erythrocyte; average monocyte

ANSWER: RIBOSOME; MITOCHONDRION; ERYTHROCYTE; AVERAGE MONOCYTE

BONUS BIOLOGY Short Answer In what chamber of the human heart is the sinoatrial (read as: sineoh-AYE-tree-al) node located?

ANSWER: RIGHT ATRIUM

Biology - Short Answer In the human heart, what chamber contains the SA node?

ANSWER: RIGHT ATRIUM

BIOLOGY Short Answer From what chamber of the human heart does blood carried by the left pulmonary artery leave?

ANSWER: RIGHT VENTRICLE

BONUS Biology - Short Answer In the process of peptide synthesis by ribosomes, what specific macromolecule catalyzes the amide bond formation?

ANSWER: RNA (ACCEPT: RIBONUCLEIC ACID; DO NOT ACCEPT: RIBOZYME)

BONUS 8) Biology - Short Answer What phenomenon involves the injection of double-stranded RNAs turning off the expression of genes with the same sequence as the RNA?

ANSWER: RNA INTERFERENCE

BONUS BIOLOGY Short Answer What specific molecule binds immediately upstream of the structural genes in the lac (read as: LACK) operon and functions to transcribe these genes?

ANSWER: RNA POLYMERASE

BONUS BIOLOGY Short Answer In the late 1800's, who was the first person to isolate and grow numerous pathogenic Monerans?

ANSWER: ROBERT KOCH

Short Answer In which of the following 4 phases of cell cycle would a chemical that causes mispairing of bases during replication do the most damage: G-1; G-2; S; M

ANSWER: S

BONUS BIOLOGY Short Answer What is the common biological name for the product of human sublingual glands?

ANSWER: SALIVA

[BONUS] Short Answer: What is the MOST common term for the general type of nourishment characteristic of mushrooms where the organism derives its nutrients by absorbing the biological products of another organism?

ANSWER: SAPROPHYTIC (ACCEPT: SAPROBE or SAPROPHAGE or SAPROPHYTE)

Biology - Short Answer What is the term for malignant tumors of the connective tissues?

ANSWER: SARCOMAS

BIOLOGY Short Answer Which one of basic plant cell types has a primary and secondary cell wall that is dead when mature?

ANSWER: SCLERENCHYMA

BIOLOGY Short Answer Biologists usually refer to the head of a tapeworm by what name?

ANSWER: SCOLEX

BIOLOGY Short Answer Proteins have different levels of structure. Which level of structure is identified with hydrogen bonds at regular intervals that produce alpha helices and betapleated sheets?

ANSWER: SECONDARY

BONUS BIOLOGY Short Answer Choosing from primary through quaternary, what level of protein structure is conferred by a beta pleated sheet?

ANSWER: SECONDARY

BONUS BIOLOGY Short Answer Order the following 3 choices from the EARLIEST stage to the LATEST stage of ovarian follicle development, and identify which of the structures is typically the LARGEST: secondary follicle; corpus luteum; corpus albicans

ANSWER: SECONDARY FOLLICLE; CORPUS LUTEUM; CORPUS ALBICANS; LARGEST = CORPUS LUTEUM

BONUS BIOLOGY Short Answer Order the following 3 choices from the EARLIEST stage to the LATEST stage of ovarian follicle development, and identify which of the structures is typically the LARGEST: secondary follicle; corpus luteum; corpus albicans

ANSWER: SECONDARY FOLLICLE; CORPUS LUTEUM; CORPUS ALBICANS; LARGEST = CORPUS LUTEUM

BIOLOGY Short Answer In what specific part of the testes does spermatogenesis occur in mammals?

ANSWER: SEMINIFEROUS TUBULES

BONUS BIOLOGY Short Answer DNA replication requires that the DNA that has been pulled apart stays single-stranded until the polymerase is able to use it as a template. What is the name of the protein that keeps the DNA single-stranded during replication?

ANSWER: SINGLE STRAND BINDING PROTEIN (ACCEPT: SSB OR SINGLE STRANDED BINDING PROTEIN)

BIOLOGY Short Answer Who discovered the antibiotic nature of penicillin in 1928?

ANSWER: SIR ALEXANDER FLEMING (ACCEPT: ALEXANDER FLEMING, FLEMING)

BIOLOGY Short Answer What is the only infectious human disease to be declared completely eradicated from nature?

ANSWER: SMALLPOX

BONUS BIOLOGY Short Answer What 2 ions are most directly involved with the conduction of a nerve cell message down an axon?

ANSWER: SODIUM AND POTASSIUM (ACCEPT: Na+ AND K+ )

[Bonus] Short Answer: In mammals, the action potential of a neuron is most dependent on the transmembrane movement of what 2 ions?

ANSWER: SODIUM AND POTASSIUM (ACCEPT: Na+ and K+ )

BIOLOGY Short Answer If a person suffers an injury and is no longer able to walk but the heart was not affected, which peripheral nervous system component is most likely to have been damaged?

ANSWER: SOMATIC (ACCEPT: SOMATOMOTOR SYSTEM)

(General Science) Short Answer What technique, used by the U.S. Navy in military exercises and suspected of harming many marine species, is the focus of a recent agreement to not run these exercises within 40 kilometers of the new Hawaiian Islands Marine Reserve?

ANSWER: SONAR (ACCEPT: MIN-RANGE SONAR)

BONUS BIOLOGY Short Answer What is the name for the technique in which DNA is digested using restriction enzymes, separated using gel electrophoresis, transferred to a filter, treated with an alkaline solution, and radioactively probed to search for specific DNA sequences?

ANSWER: SOUTHERN BLOT (DO NOT ACCEPT: BLOT, NORTHERN BLOT OR WESTERN BLOT)

Short Answer: What is the scientific name for the openings in the grasshopper body through which air enters?

ANSWER: SPIRACLES

BONUS BIOLOGY Short Answer What name is used both for the breathing aperture [AP-er-cher] on arthropods [AHR-thruh-pods] and the nasal opening of whales?

ANSWER: SPIRACLES (ACCEPT: SPIRACULUM)

BONUS BIOLOGY Short Answer In photosynthesis II (read as: two), what is light energy most notably used to initially accomplish?

ANSWER: SPLITTING OF WATER

[bonus] Short Answer Name all of the following 4 organisms that are cephalopods (read as: SEFF-al-low-pods): squid; mussel; chambered nautilus; slug

ANSWER: SQUID; CHAMBERED NAUTILUS

BONUS BIOLOGY Short Answer Indicate, respectively, the term to describe the stage in which two genetically different nuclei occur in the same cell, and the principal kingdom in which the stage occurs as a normal part of the organism's life cycle.

ANSWER: STAGE = HETEROKARYOTIC; KINGDOM = FUNGI

Short Answer What specific part of any member of the Anthophyta produces pollen?

ANSWER: STAMEN (ACCEPT: ANTHER)

BONUS BIOLOGY Short Answer What is the full binomial name of the staph bacteria which is resistant to beta-lactams?

ANSWER: STAPHYLOCOCCUS AUREUS

BONUS Biology - Short Answer Cholesterol and testosterone are examples of what type of lipid that contains four cycloalkane [sigh-kloh-a l-cane] rings and is vital to homeostasis

ANSWER: STEROID

Short Answer In photosystem one, when NADP+ forms NADPH with the catalytic help of NADP+ reductase, from what specific place in the chloroplast are the hydrogen ions derived?

ANSWER: STROMA

BONUS Biology - Short Answer What protein structure forms between homologous chromosomes during meiosis and is thought to mediate chromosomal pairing?

ANSWER: SYNAPTONEMAL COMPLEX

BONUS Biology - Short Answer What protein structure forms between homologous chromosomes during meiosis and is thought to mediate chromosomal pairing?

ANSWER: SYNAPTONEMAL COMPLEX

BONUS BIOLOGY Short Answer What are the 2 basic categories of lymphocyte cells?

ANSWER: T AND B (ACCEPT: B AND T CELLS)

BIOLOGY Short Answer What is the common name for the product of human lachrymal (read as: LACK-reh-mal) glands?

ANSWER: TEARS

BIOLOGY Short Answer What is the common name for the product of human lachrymal (read as: LACK-reh-mal) glands?

ANSWER: TEARS

BONUS Biology - Short Answer In 1961, it was demonstrated that fibroblast cells can only divide about 50 times in culture, at which point the cell cycle halts before DNA replication. The reason for this limit on cell division, also called the Hayflick limit, is due to the loss of what chromosomal caps?

ANSWER: TELOMERES

BONUS BIOLOGY Short Answer Bacteria depend on cell membrane lipids to provide structure and stability. What physical environmental factor is most likely to cause these membranes to lose stability?

ANSWER: TEMPERATURE

[bonus] Short Answer Name all of the following 4 plant structures that are typically modified leaves: corms; tendrils; cacti spines; bracts

ANSWER: TENDRILS; CACTI SPINES; BRACTS

(General Science) Short Answer What is the medical name for the disease caused by Clostridium bacteria that is prevalent in soils and commonly named lockjaw?

ANSWER: TETANUS (ACCEPT: TETANY)

BIOLOGY Short Answer In an autologous tissue transplant, from whom does the person receive the tissue?

ANSWER: THEMSELVES

BONUS BIOLOGY Short Answer What human gland, which is mostly lymphoid tissue, is critical to the normal development of cell mediated immunity and, if removed after young adulthood, seems to have no ill effects on the immune system?

ANSWER: THYMUS

BONUS BIOLOGY Short Answer What human gland or organ produces tri-iodothyronine (read as: TRY- I-OH-doh-THY-row-neen)?

ANSWER: THYROID

BONUS BIOLOGY Short Answer Name all of the following 4 hormones that are formed primarily from the amino acid tyrosine: glucagon; thyroxine; epinephrine; erythropoietin (read as: EE-rithroe-POE-ih-tin)

ANSWER: THYROXINE; EPINEPHRINE

BONUS9) Biology - Short Answer What type of intercellular junction is used to bind intestinal epithelia so that fluids will not seep between them?

ANSWER: TIGHT JUNCTION

Biology - Short Answer In the late 1800s, Martinus Beijerinck investigated what specific infectious agent, which he discovered could not be cultured on nutrient media in test tubes or petri dishes?

ANSWER: TOBACCO MOSAIC VIRUS

BONUS BIOLOGY Short Answer Name all of the following 4 diseases that are caused by protists: botulism; rabies; typhoid fever; toxoplasmosis

ANSWER: TOXOPLASMOSIS

BIOLOGY Short Answer What is the process by which bacterial genes can be transferred from one bacterium to another via bacteriophages

ANSWER: TRANSDUCTION

BIOLOGY Short Answer What specific type of microscopy passes a beam of electrons through an ultra-thin specimen that is usually stained with an electron opaque substance, producing an image which can be captured by a charged coupled device?

ANSWER: TRANSMISSION ELECTRON MICROSCOPY (ACCEPT: TEM) (DO NOT ACCEPT: SCANNING ELECTRON MICROSCOPY OR SEM)

BIOLOGY Short Answer What is the most common scientific term for plant water loss through stomata

ANSWER: TRANSPIRATION

BONUS BIOLOGY Short Answer What adjective describes hormones that cause secretion of other hormones?

ANSWER: TROPIC

BIOLOGY Short Answer Name the protein that polymerizes [puh-LIM-uh-ryz-es] to form microtubules.

ANSWER: TUBULIN

BIOLOGY Short Answer What is the specific type of protein from which spindle fibers are composed?

ANSWER: TUBULIN (ACCEPT: ALPHA AND/OR BETA TUBULIN)

TOSS-UP 7) GENERAL SCIENCE Short Answer What major biome occurs only in the Northern Hemisphere and is composed of both Low Arctic and High Arctic areas?

ANSWER: TUNDRA

[BONUS] Short Answer: Name all of the following 5 animal groups that are NOT echinoderms: sea urchins; sand dollars; tunicates; rotifers; brittle stars

ANSWER: TUNICATES; ROTIFERS

BIOLOGY Short Answer By name or number, identify all of the following four animals that are ectotherms: 1) turtles, 2) goldfish, 3) otters, 4) beetles.

ANSWER: TURTLES, GOLDFISH, AND BEETLES (ACCEPT: 1, 2, AND 4)

BONUS Biology - Short Answer Insects and reptiles excrete nitrogenous [nigh-traw-jen-uhs] waste in what nontoxic form?

ANSWER: URIC ACID

(general science) [bonus] Short Answer Gout in humans is the result of the precipitation of what substance in joints?

ANSWER: URIC ACID (ACCEPT: LITHIC ACID or TRIKETOPURINE)

BONUS Biology Short Answer What is the anticodon on a tRNA for AAA (read as: A, A, A)?

ANSWER: UUU

BONUS BIOLOGY Short Answer What is the anticodon on a tRNA for AAA (read as: A, A, A)?

ANSWER: UUU

BONUS Biology - Short Answer What nerve is known as the "wandering" nerve, as it travels down into the chest and abdomen from the throat and larynx, where it helps control the cardiovascular and gastrointestinal systems?

ANSWER: VAGUS NERVE

BONUS Biology - Short Answer What largely parasympathetic nerve innervates the heart and is responsible for slowing down heart rate and lowering ventricular contractility when in a "rest and digest" state?

ANSWER: VAGUS NERVE (ACCEPT: CN X, CRANIAL NERVE TEN)

BONUS BIOLOGY Short Answer What is the term for the pattern of arrangement of embryonic leaves within the bud?

ANSWER: VERNATION

BONUS BIOLOGY Short Answer Name the fat soluble vitamin that is a component of visual pigments in vertebrates.

ANSWER: VITAMIN A (ACCEPT: RETINOL)

Biology - Short Answer Complete beta oxidation of odd-numbered saturated fatty acids requires what vitamin that is not required for the beta oxidation of even-numbered saturated fatty acids?

ANSWER: VITAMIN B-12 (Accept: COBALAMIN)

BIOLOGY Short Answer Name the water soluble vitamin found in nuts, grains, and meats that functions in vertebrates as a component of the coenzyme NAD⁺ .

ANSWER: VITAMIN B3 (ACCEPT: NIACIN, NICOTINIC ACID, NICOTINAMIDE, VITAMIN PP; DO NOT ACCEPT: VITAMIN B7, BIOTIN)

BIOLOGY Multiple Choice At which of the following pH levels will human pepsin be most active: W) 2 X) 4 Y) 6 Z) 8

ANSWER: W) 2

Biology - Multiple Choice At which of the following pH values would pepsin likely be most active? W) 2 X) 4 Y) 6 Z) 8

ANSWER: W) 2

TOSS-UP8) Biology - Multiple Choice At which of the following pH values would pepsin likely be most active? W) 2 X) 4 Y) 6 Z) 8

ANSWER: W) 2

BIOLOGY Multiple Choice Lysosomes [LY-suh-sohms] are membrane-enclosed organelles in a cell containing an array of enzymes. Which of the following is closest to the pH of these vesicles? W) 5 X) 6 Y) 7 Z) 8

ANSWER: W) 5

BONUS BIOLOGY Multiple Choice Which of the following is the RNA complementary sequence of the following sequence: 5'CCGCGA 3' (read as: 5-prime, CCGCGS, 3-prime) W) 5' UCGCGG 3' (read as: 5-prime, UCGCGG, 3-prime) X) 5' GGCGCU 3' Y) 5' GGCGCT 3' Z) 5' TCGCGG 3'

ANSWER: W) 5' UCGCGG 3'

BIOLOGY Multiple Choice If a single nucleotide is inserted into the middle of the protein coding sequence of a gene, which of the following is the likely outcome? W) A frameshift mutation that leads to multiple missense and/or nonsense mutations X) A single codon will have a missense mutation Y) One codon will have a missense or nonsense mutation, and the another nucleotide [NOOklee-uh-tyd] will be deleted Z) A frameshift mutation that is silent

ANSWER: W) A FRAMESHIFT MUTATION THAT LEADS TO MULTIPLE MISSENSE AND/OR NONSENSE MUTATIONS

BONUS BIOLOGY Multiple Choice Which of the following situations might cause disruptive selection to occur in a population of seed-eating finches on an island? W) A hurricane that wipes out plants that produce medium-sized seeds X) A hurricane that wipes out plants that produce small and large seeds Y) A hurricane that wipes out plants that produce small seeds Z) A hurricane that reduces equally plants that produce all sizes of seeds

ANSWER: W) A HURRICANE THAT WIPES OUT PLANTS THAT PRODUCE MEDIUM-SIZED SEEDS

(general science) Multiple Choice Which of the following is the BEST description of an ecological niche: W) a situation in which a species is well adapted to a specific set of environmental conditions X) the biomass load of a given environment Y) a pollutant that becomes concentrated in organisms as it moves up through the food chain Z) all the environmental conditions, plants, and animals of an isolated area

ANSWER: W) A SITUATION IN WHICH A SPECIES IS WELL ADAPTED TO A SPECIFIC SET OF ENVIRONMENTAL CONDITIONS

(General Science) Multiple Choice Which of the following is MOST commonly used for removing many organic compounds in water in municipal water treatment plants: W) activated charcoal X) precipitation reactions Y) reverse osmosis Z) dredging

ANSWER: W) ACTIVATED CHARCOAL

BIOLOGY Multiple Choice Which of the following is NOT an example of a current population that shows genetic drift due to the bottleneck effect? W) African elephant X) Northern elephant seals Y) Whooping crane Z) Cheetah

ANSWER: W) AFRICAN ELEPHANT

BIOLOGY Multiple Choice Which of the following is most directly responsible for the colloid osmotic pressure in humans that acts to retain fluid in the vascular system: W) albumin X) sodium ions Y) calcium ions Z) potassium ions

ANSWER: W) ALBUMIN

BONUS BIOLOGY Multiple Choice Which of the following is NOT an example of energy coupling? W) An ion releases energy as it travels through an ion channel down its concentration gradient X) A kinase hydrolyzes ATP for energy to catalyze a reaction Y) A transport molecule uses energy from one molecule going down its concentration gradient to power transport of another molecule up its gradient Z) As protons move down their concentration gradient, ATP synthase gathers energy to catalyze the synthesis of ATP

ANSWER: W) AN ION RELEASES ENERGY AS IT TRAVELS THROUGH AN ION CHANNEL DOWN ITS CONCENTRATION GRADIENT

BIOLOGY Multiple Choice Which of the following tissues is most directly responsible for the growth in height of a tree: W) apical meristem X) shoot terminus Y) terminal cambial Z) cambium terminus

ANSWER: W) APICAL MERISTEM

BIOLOGY Multiple Choice Which of the following tissues is most directly responsible for the growth in height of a tree: W) apical meristem X) shoot terminus Y) terminal cambial Z) cambium terminus

ANSWER: W) APICAL MERISTEM

BONUS BIOLOGY Multiple Choice Which of the following habitat and life history combinations would select for the production of a large brood size? W) Arctic, precocial young [pri-KOH-shuhl] X) Arctic, altricial young [al-TRISH-uhl] Y) Tropics, precocial young Z) Tropics, altricial young

ANSWER: W) ARCTIC, PRECOCIAL YOUNG

(General Science) Multiple Choice Which of the following is an intestinal worm which grows to about 25 centimeters in length and is transferred from person to person by way of contaminated soil: W) Ascaris (read as: as-KAH-ris) X) guinea worm Y) shistosomiasis (read as: SHISS-toe-so-MY-ah-sis) Z) lymphatic filariasis (read as: fil-AH-ree-AH-sis)

ANSWER: W) ASCARIS

BIOLOGY Multiple Choice Which of the following is NOT an example of substrate-level phosphorylation in cellular respiration? W) ATP synthase phosphorylates ADP during chemiosmosis X) GDP is phosphorylated as succinyl [SUHK-suh-nil] and CoA is converted to succinate [SUHK-suh-nayt] Y) Phosphofructokinase phosphorylates ADP Z) Pyruvate kinase phosphorylates ADP

ANSWER: W) ATP SYNTHASE PHOSPHORYLATES ADP DURING CHEMIOSMOSIS

(General Science) Multiple Choice Which of the following is the most common and accurate term for when a person's immune system generates antibodies against its own body's cells: W) autoimmune dysfunction X) homeostatic inflammation Y) heteroimmune reaction Z) anaphylactic reaction

ANSWER: W) AUTOIMMUNE DYSFUNCTION

BONUS BIOLOGY Multiple Choice Indicate where action potentials are generated in a myelinated [MY-uh-luh-nay-tid] post-synaptic cell. W) Axon hillock X) Dendrites Y) Neuron nucleus Z) Schwann cell

ANSWER: W) AXON HILLOCK

BIOLOGY Multiple Choice Which of the following venomous snakes produces mostly neurotoxins? W) Black mamba X) Eastern diamondback rattlesnake Y) Cottonmouth Z) Copperhead

ANSWER: W) BLACK MAMBA

BONUS BIOLOGY Multiple Choice What is the term for deposition of solid matter composed of mineral substances and salts, formed in any portion of the human body, such as kidney stones? W) Calculus X) Caries Y) Carbuncle Z) Cachexia [kuh-KEK-see-uh]

ANSWER: W) CALCULUS

BONUS BIOLOGY Multiple Choice When using a light microscope to view a stained cross section of a dorsal root ganglion, which of the following structures are you most likely to see? W) Cell bodies of sensory neurons X) Axons of sensory neurons Y) Axons of motor neurons Z) Cell bodies of motor neurons

ANSWER: W) CELL BODIES OF SENSORY NEURONS

Biology - Multiple Choice Which of the following is most similar in structure to the basal body of a eukaryotic flagellum [fluh-j el-u m]? W) Centriole X) Cilium Y) Bacterial flagellum Z) Intermediate filament

ANSWER: W) CENTRIOLE

BIOLOGY Multiple Choice Nematocysts (read as: NEHM-aht-oh-sists) are typically found in which of the following groups? W) Cnidarians (read as: ny-DEHR-ee-ahns (NO "C")) X) Flatworms Y) Roundworms Z) Echinoderms (read as: i-KY-noh-dehrms)

ANSWER: W) CNIDARIANS

BONUS BIOLOGY Multiple Choice Mineral salts participate in enzymatic activity as which of the following? W) Co-factors X) Activators Y) Stabilizers Z) Catalysts

ANSWER: W) CO-FACTORS

BIOLOGY Multiple Choice Which of the following marine fish groups reproduces ovoviviparously [oh-voh-vy-vip-ER-uhs-lee]? W) Coelocanth [SEE-luh-kanth] X) Ratfish Y) Ray Z) Lamprey

ANSWER: W) COELOCANTH

BIOLOGY Which of the following is a major benefit of membrane-bound cells and organelles? W) Compartmentalization of biochemical reactions X) Protection from UV radiation Y) Thermal isolation Z) Membranes conduct electrical signals

ANSWER: W) COMPARTMENTALIZATION OF BIOCHEMICAL REACTIONS

BIOLOGY Multiple Choice Which of the following is a major benefit of membrane-bound cells and organelles? W) Compartmentalization of biochemical reactions X) Protection from UV radiation Y) Thermal isolation Z) Membranes conduct electrical signals

ANSWER: W) COMPARTMENTALIZATION OF BIOCHEMICAL REACTIONS

Biology - Multiple Choice Which of the following correctly describes chromatin? W) Condensed DNA and associated proteins X) Just condensed DNA Y) DNA and associated carbohydrates Z) Loosely coiled DNA and associated proteins

ANSWER: W) CONDENSED DNA AND ASSOCIATED PROTEINS

BONUS BIOLOGY Multiple Choice Which of the following plant hormones is NOT correctly matched with hormone effect? W) Cytokinin [sy-tuh-KY-nin]: promotes seed dormancy X) Abscisic acid: promotes stomata closing Y) Ethylene [ETH-uh-leen]: promotes fruit ripening Z) Auxin [AWK-sin]: gravitropism

ANSWER: W) CYTOKININ: PROMOTES SEED DORMANCY

BONUS Biology - Multiple Choice The terms frustule [fruh-s tool], epitheca [epi-thek-u h], and silica apply to which of the following organisms? W) Diatoms X) Cocolithophors Y) Dinoflagellates Z) Foraminifera

ANSWER: W) DIATOMS

BONUS BIOLOGY Multiple Choice Assume you would like to isolate a specific membrane protein from a cell. How could you best remove the protein from the membrane? W) Disrupt the lipid bilayer with detergents X) Grind the cells using a pestle Y) Stain the target protein with fluorescing antibodies Z) Centrifuge the cells to separate the lipid bilayer from other cellular components

ANSWER: W) DISRUPT THE LIPID BILAYER WITH DETERGENTS

BIOLOGY Multiple Choice For ions or polar molecules to pass across a plasma membrane in passive transport, the molecules must move in which of the following ways? W) Down an electrochemical gradient X) Up an electrochemical gradient Y) Through a pump Z) With a chaperone molecule

ANSWER: W) DOWN AN ELECTROCHEMICAL GRADIENT

BIOLOGY Multiple Choice Which of the following BEST describes how annelids use their hydrostatic skeleton for locomotion? W) Each body segment is walled off, allowing body fluids to be compressed or expanded by its muscles X) Each body segment has small holes allowing only small amounts of liquid to move at one time Y) All body segments are connected, and the muscles contract and release to move the fluid around the worm's body, creating aqueous motion Z) Anterior body segments are connected but walled off from posterior segments. Muscles control body fluid movement bilaterally

ANSWER: W) EACH BODY SEGMENT IS WALLED OFF, ALLOWING BODY FLUIDS TO BE COMPRESSED OR EXPANDED BY ITS MUSCLES

BONUS BIOLOGY Multiple Choice Which of the following mammal species is usually considered diurnal [dy-UR-nl]? W) Eastern fox squirrel X) Black-footed ferret Y) Virginia opossum Z) Raccoon

ANSWER: W) EASTERN FOX SQUIRREL

BIOLOGY Multiple Choice Which of the following organ systems is responsible for the secretion of hormones? W) Endocrine X) Reproductive Y) Circulatory Z) Immune

ANSWER: W) ENDOCRINE

BIOLOGY Multiple Choice Which of the following is an example of catabolism in the cell? W) Energy is extracted from the fat in avocado X) New DNA is synthesized Y) Glutamic acid is made from alpha-ketoglutarate [kee-toh-GLOO-tay-rayt] in the citric acid cycle Z) Dihydroxyacetone [dy-hy-drok-see-AS-uh-tohn] phosphate from glycolysis [gly-KOL-uhsis] is converted to a precursor for lipid synthesis

ANSWER: W) ENERGY IS EXTRACTED FROM THE FAT IN AVOCADO

BIOLOGY Multiple Choice Human epidermis is mostly composed of which of the following basic animal tissues types: W) epithelial X) connective Y) nervous Z) muscle

ANSWER: W) EPITHELIAL

Biology - Multiple Choice Which of the following types of cells has the longest lifespan in circulation? W) Erythrocyte [uh-ree-throh-sight] X) Platelet Y) Monocyte Z) Neutrophil [nyoo-troh-fil]

ANSWER: W) ERYTHROCYTE

BIOLOGY Multiple Choice Approximately how often is the entire stomach lining in a human replaced? W) Every three days X) Every five days Y) Every week Z) Every month

ANSWER: W) EVERY THREE DAYS

BIOLOGY Multiple Choice Which of the following portions of a gene could ultimately be translated to protein? W) Exon X) Terminator Y) Intron Z) Promoter

ANSWER: W) EXON

Multiple Choice: The cloaca (read as: klo-AY-ka) is a reproductive structure of: W) female bird X) male shark Y) female cestode Z) male lobster

ANSWER: W) FEMALE BIRD

BIOLOGY Multiple Choice Which of the following would BEST describe the quaternary structure of collagen: W) fibrous X) globular Y) planar Z) transport

ANSWER: W) FIBROUS

Biology Multiple Choice: Which of the following would BEST describe the quaternary structure of collagen: W) fibrous X) globular Y) planar Z) transport

ANSWER: W) FIBROUS

BIOLOGY Multiple Choice Which of the following bases wobbles on a transfer RNA anticodon? W) First base X) Second base Y) Third base Z) First base and third base

ANSWER: W) FIRST BASE

BONUS BIOLOGY Multiple Choice Which of the following statements is NOT correct regarding short day plants? W) Flowering occurs when night length is less than critical length X) Continuity of darkness is crucial; flowering will not occur if darkness is interrupted by a momentary light flash Y) Short day plants are synonymous with long night plants Z) Photoperiod can be detected via phytochromes

ANSWER: W) FLOWERING OCCURS WHEN NIGHT LENGTH IS LESS THAN CRITICAL LENGTH

(General Science) Multiple Choice: Which of the following is MOST closely related to the mosquito: W) fly X) wasp Y) human body louse Z) dragonfly

ANSWER: W) FLY

Biology - Multiple Choice Which of the following are examples of asexual reproduction? W) Fragmentation, budding, and binary fission X) Parthenocarpic [pahr-then-o h-car-pic] fruit development and budding Y) Binary fission and zygote formation Z) Fragmentation and sporogenesis

ANSWER: W) FRAGMENTATION, BUDDING, AND BINARY FISSION

BONUS BIOLOGY Multiple Choice Loss of the retinoblastoma gene product, Rb, causes cancer by affecting which of the following portions of the cell cycle? W) G1/S checkpoint X) S/G2 checkpoint Y) G2/M checkpoint Z) M/G1 checkpoint

ANSWER: W) G1/S CHECKPOINT

BONUS BIOLOGY Multiple Choice While hiking in Arizona, you come over a rise, hear a rattling sound and see a western diamondback rattlesnake coiled and ready to strike. Indicate which of the following physiological responses would NOT occur in you? W) Gluconeogenesis would occur in the liver X) Blood vessels in the gut and skin would constrict Y) Heart rate and blood pressure would increase Z) Adipose tissue would release fatty acids to the blood stream

ANSWER: W) GLUCONEOGENESIS WOULD OCCUR IN THE LIVER

BONUS BIOLOGY Multiple Choice Which of the following does not occur in the tertiary structure of a protein? W) Glycosidic linkages X) Disulfide bridges Y) Hydrophobic interactions Z) Ionic bonds

ANSWER: W) GLYCOSIDIC LINKAGES

Biology - Multiple Choice There are three checkpoints during normal cell division. In what stages in the life of a cell are these checkpoints found? W) Growth phase I, Growth Phase II, and mitosis X) Growth phase I, S phase, and mitosis Y) S phase, telophase, and anaphase Z) S phase, growth phase II, and mitosis

ANSWER: W) GROWTH PHASE I, GROWTH PHASE II, AND MITOSIS

Biology - Multiple Choice Which of the following nucleobases is NOT a pyrimidine [pur-RIM-ih-deen]? W) Guanine X) Cytosine Y) Thymine Z) Uracil

ANSWER: W) GUANINE

BIOLOGY Multiple Choice Which of the following is NOT synthesized by the human liver? W) Hemoglobin X) Fibrinogen (read as; fy-BRI-noh-jehn) Y) Albumin Z) Lipoprotein

ANSWER: W) HEMOGLOBIN

[bonus] Multiple Choice Which of the following regulate development in animals, plants and fungi, and were first discovered in Drosophila: W) homeobox (read as: HOME-ee-oh-box) genes X) p53 Y) shock genes Z) oncogenes

ANSWER: W) HOMEOBOX GENES

BIOLOGY Multiple Choice Which of the following terms is used to describe a structure that is similar in different species of organisms, because the species share a common ancestor that was the source of the structure? W) Homologous [huh-MOL-uh-guhs] X) Convergent Y) Analogous [uh-NAL-uh-guhs] Z) Divergent

ANSWER: W) HOMOLOGOUS

Biology - Multiple Choice Which of the following tissues would contain a chondrocyte? W) Hyaline cartilage X) Areolar connective tissue Y) Pseudostratified ciliated columnar epithelium Z) Compact bone

ANSWER: W) HYALINE CARTILAGE

BIOLOGY Multiple Choice Which of the following is the target tissue of the hormone leptin where it binds with LepRB (read as: L, E, P, R, B) receptors: W) hypothalamus X) pancreas Y) adipose tissue Z) kidney

ANSWER: W) HYPOTHALAMUS

[bonus] Multiple Choice Which of the following is a primary consequence of histone acetylation: W) increased access of transcriptional factors to DNA X) decreased access of transcriptional factors to DNA Y) increased rate of cells entering mitosis Z) G-1 block

ANSWER: W) INCREASED ACCESS OF TRANSCRIPTIONAL FACTORS TO DNA

BIOLOGY Multiple Choice From which of the following does the seed coat of angiosperm seeds typically develop: W) integuments X) polar nuclei Y) nucleus Z) cotyledons

ANSWER: W) INTEGUMENTS

Biology - Multiple Choice Which of the following phases is typically the longest in a normal cell's life cycle? W) Interphase X) Metaphase Y) Prophase Z) Anaphase

ANSWER: W) INTERPHASE

BONUS BIOLOGY Multiple Choice Which of the following parts of a nerve cell acts as a resistor? W) Ion channels X) Cytoplasm Y) Plasma membrane Z) Mitochondria [my-tuh-KON-dree-uh]

ANSWER: W) ION CHANNELS

BONUS 13) BIOLOGY Multiple Choice Which of the following best explains the property of TAQ polymerase [tak pol-UH-muh-rays] that makes it ideal for use in PCR reactions? W) It is heat-stable X) It is resistant to mechanical perturbation Y) It can remain stable at a variety of pH levels Z) It can be used in low concentrations

ANSWER: W) IT IS HEAT-STABLE

BONUS BIOLOGY Multiple Choice Which of the following is NOT true of DNA replication: W) it is initiated by a short DNA primer X) the growing strand is polymerized in a 5-prime to 3-prime direction Y) DNA polymerase helps form the phosphodiesterase (read as: FOSS-foe-die-ESS-ter-ase) bonds Z) synthesis of the lagging strand is discontinuous

ANSWER: W) IT IS INITIATED BY A SHORT DNA PRIMER

BIOLOGY Multiple Choice Which of the following best describes why the xylem [ZY-luhm] is an important plant tissue? W) It transports water and minerals from the roots to the leaves X) It transports water and minerals from the leaves to the roots Y) It is the best part of the plant to eat Z) It gives a plant its strength to stand straight

ANSWER: W) IT TRANSPORTS WATER AND MINERALS FROM THE ROOTS TO THE LEAVES

Biology - Multiple Choice In which of the following organisms would one most likely find nematocysts? W) Jellyfish X) Octopus Y) Sponge Z) Sea cucumber

ANSWER: W) JELLYFISH

Biology - Multiple Choice In which of the following organisms would one most likely find nematocysts? W) Jellyfish X) Octopus Y) Sponge Z) Sea cucumber

ANSWER: W) JELLYFISH

BONUS BIOLOGY Multiple Choice Which of the following is NOT true regarding juvenile hormone (JH) in the silkworm moth as it pertains to the hormonal control of insect metamorphosis? W) JH is produced by the prothoracic [proh-tha-RAS-ik] gland X) Levels of JH are higher in first instar larvae [LAHR-vee] relative to levels in fifth instar larvae Y) Pupae [PYOO-pee] have little to no JH Z) The interplay of ecdysone [EK-duh-sohn] and JH is important to the molting process

ANSWER: W) JH IS PRODUCED BY THE PROTHORACIC GLAND

BONUS BIOLOGY Multiple Choice Which of the following reactions is incorrectly matched to its enzyme? W) Kinase: catalyzes the removal of a phosphate group from a molecule X) Isomerase [eye-SOM-uh-rays]: catalyzes the rearrangement of bonds within a single molecule Y) Synthase: condenses two smaller molecules together Z) Protease [PROH-tee-ays]: breaks down proteins by hydrolyzing bonds between amino acids

ANSWER: W) KINASE: CATALYZES THE REMOVAL OF A PHOSPHATE GROUP FROM A MOLECULE

BIOLOGY Multiple Choice Which of the following is MOST characteristic of human blast cells in on observation with light microscopy: W) large nuclear to cytoplasmic ratio X) small nucleolus and mostly heterochromatin in the nucleus Y) lack of cytoplasmic granules and small overall size compared to mature cells Z) absence of nucleolus (read as: NEW-clee-oh-luss) and endoplasmic reticulum

ANSWER: W) LARGE NUCLEAR TO CYTOPLASMIC RATIO

BIOLOGY In DNA replication, which of the following molecules is synthesized in the same direction that the replication fork is moving? W) Leading strand X) Okazaki fragment Y) Primer Z) Lagging strand

ANSWER: W) LEADING STRAND

BIOLOGY Multiple Choice Which of the following animal species is usually NOT considered gregarious [gri-GAIR-ee-uhs]? W) Leopards X) Harbor seals Y) Brown-headed cowbirds Z) Nile crocodiles

ANSWER: W) LEOPARDS

BONUS BIOLOGY Multiple Choice While on holiday in Mexico, you note a local vendor selling arrow plant seed pods that jump due the presence of an immature insect that is violently throwing itself from side to side in the seed pod. The insect responsible for this would be a member of what insect order? W) Lepidoptera [lep-i-DOP-ter-uh] X) Diptera Y) Coleoptera [koh-lee-OP-ter-uh] Z) Hymenoptera [hy-muh-NOP-ter-uh]

ANSWER: W) LEPIDOPTERA

BONUS BIOLOGY Multiple Choice Which of the following branches of science deals with study of the life and phenomena of inland standing waters, especially lakes and ponds? W) Limnology [lim-NOL-uh-jee] X) Hydrology Y) Ichthyology [ik-thee-OL-uh-jee] Z) Ecology

ANSWER: W) LIMNOLOGY

BIOLOGY Multiple Choice Which of the following types of genes violates Mendel's principle of independent assortment? W) Linked X) Codominant Y) Dominant Z) Epistatic

ANSWER: W) LINKED

BONUS BIOLOGY Multiple Choice In humans, which of the following structures is NOT derived from myeloid [MY-uh-loid] stem cells? W) Lymphocytes [LIM-fuh-syts] X) Eosinophils [ee-uh-SIN-uh-fils] Y) Basophils [BAY-suh-fils] Z) Platelets [PLAYT-lits]

ANSWER: W) LYMPHOCYTES

BIOLOGY Multiple Choice Which of the following organelles within the eukaryotic [yooKAR-ee-oh-tik] cell functions as the digestive system of the cell, serving both to degrade material taken up from outside the cell and to digest obsolete components of the cell itself? W) Lysosome X) Golgi apparatus Y) Smooth endoplasmic reticulum [EN-duh-plaz-mik ri-TIK-yuh-luhm] Z) Rough endoplasmic reticulum

ANSWER: W) LYSOSOME

GENERAL SCIENCE Multiple Choice Bioluminescence is a phenomenon most common in which of the following environments: W) marine X) fresh water Y) land Z) atmosphere

ANSWER: W) MARINE

BIOLOGY Multiple Choice Which of the following is the most common term for an area of primitive cells in plants where active cell division occurs: W) meristem X) ground tissue Y) primary cells Z) germinal tissue

ANSWER: W) MERISTEM

BIOLOGY Multiple Choice Which of the following choices is NOT a source of new genetic combinations? W) Mitosis [my-TOH-sis] X) Fertilization Y) Mutation Z) Meiosis [meye-OH-sis]

ANSWER: W) MITOSIS

BONUS BIOLOGY Multiple Choice Photosynthesis requires water as a reactant. Oxygen atoms from the water end up in which of the following product molecules? W) Molecular oxygen X) Water Y) Glucose Z) ATP

ANSWER: W) MOLECULAR OXYGEN

BIOLOGY Multiple Choice Cephalopods belong to which of the following groups? W) Mollusks X) Annelids Y) Roundworms Z) Arthropods

ANSWER: W) MOLLUSKS

Biology - Multiple Choice Which of the following is NOT true regarding aquatic biomes? W) Much of the photosynthesis occurring in a coral reef is performed by corals X) Intertidal zones are characterized by high levels of oxygen and nutrients Y) Oceanic pelagic zones are the largest biome on Earth Z) Phytoplankton are the dominant photosynthesizers in pelagic zones

ANSWER: W) MUCH OF THE PHOTOSYNTHESIS OCCURRING IN A CORAL REEF IS PERFORMED BY CORALS

BONUS BIOLOGY Multiple Choice The mechanism of action of the antiviral drugs zanamivir [zaNUHM-i-veer] and oseltamivir [oh-sel-TUHM-i-veer] is to inhibit what virally encoded enzymes expressed on the surface of influenza virions [VY-ree-ons]? W) Neuraminidase [nyoor-uh-MIN-uh-days] X) Hemagglutinin [he-muh-GLU-ti-nin] Y) Nonspecific protease Z) Reverse transcriptase

ANSWER: W) NEURAMINIDASE

BONUS BIOLOGY Multiple Choice Which of the following statements is NOT true about otters? W) Newborn otters are taken care of by their fathers only X) Their dens are called holts or couches Y) They feed on fish, shellfish, amphibians and birds Z) A male otter is called a dog

ANSWER: W) NEWBORN OTTERS ARE TAKEN CARE OF BY THEIR FATHERS ONLY

BONUS Biology - Multiple Choice Atropine is a powerful toxin that is gathered from which of the following groups of plants? W) Nightshades X) Orchids Y) Horsetails Z) Yews

ANSWER: W) NIGHTSHADES

Multiple Choice Which of the following occurs during synapsis of meiosis: W) non-sister chromatids of homologous chromosomes cross over X) sister chromatids of homologous chromosomes cross over Y) non-sister chromatids of non-homologous chromosomes cross over Z) sister chromatids of non-homologous chromosomes cross over

ANSWER: W) NON-SISTER CHROMATIDS OF HOMOLOGOUS CHROMOSOMES CROSS OVER

BIOLOGY Multiple Choice In which of the following locations would a spliceosome [SPLYsee-uh-sohm] be found? W) Nucleus [NOO-klee-uhs] X) Cytoplasm Y) Plasma membrane Z) Endosome

ANSWER: W) NUCLEUS

BONUS Biology - Multiple Choice Through what anatomical feature does water exit a sponge? W) Osculum X) Ostium Y) Atrium Z) Coelom [SEE-lum]

ANSWER: W) OSCULUM

BIOLOGY Multiple Choice Which of the following is NOT a common method by which ion channels are gated? W) Osmolarity X) Voltage Y) Mechanical Z) Ligand

ANSWER: W) OSMOLARITY

Multiple Choice: Blood albumin is most directly involved with: W) osmoregulation X) hormone regulation Y) bile production Z) acid base regulation

ANSWER: W) OSMOREGULATION

BONUS BIOLOGY Multiple Choice When a gene is duplicated within an organism to generate a second copy of that gene, what term in genomics would describe the relationship between the genes? W) Paralogous X) Orthologous Y) Syntologous Z) Heterologous

ANSWER: W) PARALOGOUS

BONUS Biology - Multiple Choice Which of the following correctly describes the difference between peptoids, or poly-N-substituted glycines, and proteins? W) Peptoids have less constrained secondary structure than proteins X) Peptoids cannot possess tertiary structure, while proteins can Y) Peptoids form more stable alpha helices than proteins Z) Peptoids form more stable beta sheets than proteins

ANSWER: W) PEPTOIDS HAVE LESS CONSTRAINED SECONDARY STRUCTURE THAN PROTEINS

BIOLOGY Multiple Choice Which of the following processes results in a net reduction in the amount of atmospheric carbon dioxide? W) Photosynthesis X) Fossil fuel use Y) Respiration Z) Decomposition

ANSWER: W) PHOTOSYNTHESIS

BIOLOGY Multiple Choice Which of the following bird species feeds their young crop milk for the first few days after hatching? W) Pigeons X) Gulls Y) Ostriches Z) Hawks

ANSWER: W) PIGEONS

BONUS BIOLOGY Multiple Choice Which of the following is NOT a cellular structure that functions to increase surface area and maximize the performance of cellular processes? W) Pili [pee-LEE] X) Microvilli [my-kroh-VIL-eye] Y) Cisternae [si-STUR-nee] Z) Cristae [KRIS-tuh]

ANSWER: W) PILI

BONUS BIOLOGY Multiple Choice Leghemoglobin [leg-HEE-muh-gloh-bin] functions as an oxygen binding protein in which of the following? W) Plant roots X) Bacteria Y) Human legs Z) Lactating mothers

ANSWER: W) PLANT ROOTS

BONUS BIOLOGY Multiple Choice Which disease-causing agent is NOT correctly matched to the disease it causes? W) Plasmodium falciparum (read as: plahz-MŌE-dee-uhm fahl-SI-pah-ruhm) - Dengue hemorrhagic fever X) Rickettsia rickettsii (read as: ri-KEHT-see-ah ri-KEHT-see) - Rocky Mountain spotted fever Y) Borrelia burgdorferi (read as: boh-REHL-ee-ah boorg-DOHR-feh-ree) - Lyme disease Z) Trypanosoma brucei (read as: try-pah-noh-SŌE-mah BROOH-sey) - Human African sleeping sickness

ANSWER: W) PLASMODIUM FALCIPARUM - DENGUE HEMORRHAGIC FEVER

BIOLOGY Multiple Choice The absence of which of the following transcript components would decrease eukaryotic translational initiation? W) Poly(A) tail X) Intron Y) Stop codon Z) Internal exon

ANSWER: W) POLY(A) TAIL

BONUS Biology - Multiple Choice Which of the following could occur due to a mutation in a Hox gene in a human embryo? W) Polydactyly [poly-d ak-til-ee] X) Sickle cell anemia Y) Phenylketonuria [fee-nil-k ee-toe-n oo-ree-u h] Z) Chronic myelogenous [my-u hl-a w-jen-us] leukemia

ANSWER: W) POLYDACTYLY

BONUS BIOLOGY Multiple Choice The cotyledon [kot-uhl-EED-uhn] is which of the following? W) Primary embryonic leaf X) Primary embryonic petal Y) Secondary embryonic leaf Z) Secondary embryonic petal

ANSWER: W) PRIMARY EMBRYONIC LEAF

BIOLOGY Multiple Choice Which of the following types of biological molecules is digested by trypsin: W) proteins X) nucleic acids Y) carbohydrates Z) lipids

ANSWER: W) PROTEINS

[bonus] Multiple Choice What is the first place in the breakdown of glucose, starting in glycolysis and through the Krebs cycle, where CO2 is released: W) pyruvate yielding Acetyl Co-A X) 3-phosphoglycerate (read as: 3-foss-foe-GLISS-err-ate) yielding 2-phosphoglycerate Y) isocitrate yielding alpha ketogluterate Z) oxaloacetate (read as: ox-AL-oh-ACETATE) yielding citrate

ANSWER: W) PYRUVATE YIELDING ACETYL Co-A

BIOLOGY Multiple Choice The 2011 Nobel Prize in Physiology or Medicine was awarded to three researchers, including Jules Hoffman and Bruce Beutler, for their discoveries leading to the characterization of the Toll-like receptor (TLR) family of molecules. This family of molecules is encoded by genes in a wide variety of eukaryotes, ranging from flies to humans, and plays an important role in which of the following processes? W) Recognizing pathogens in the immune system X) Breaking down lipids in the digestive system Y) Regulating neurotransmitters in the nervous system Z) Enabling meiosis in the reproductive system

ANSWER: W) RECOGNIZING PATHOGENS IN THE IMMUNE SYSTEM

Multiple Choice Which of the following is a primary target of certain HIV drugs, such as AZT, specifically because HIV is a retrovirus: W) reverse transcriptase X) HIV receptors Y) CD 4 cells Z) CD 8 cells

ANSWER: W) REVERSE TRANSCRIPTASE

BONUS BIOLOGY Multiple Choice Which of the following cellular phenomena uses double-stranded RNA with the same or similar sequence to a gene to target that gene for inhibition of transcription and translation? W) RNA interference X) Transposition Y) RNA editing Z) Alternative splicing

ANSWER: W) RNA INTERFERENCE

BONUS BIOLOGY Multiple Choice Which of the following correctly describes the path of a newly synthesized secreted protein? W) Rough endoplasmic reticulum [en-duh-PLAZ-mik ri-TIK-yuh-luhm], vesicles[VES-i-kuhls], Golgi apparatus, vesicles, plasma membrane X) Golgi apparatus, vesicles, rough endoplasmic reticulum, vesicles, plasma membrane Y) Smooth endoplasmic reticulum, vesicles, Golgi apparatus, plasma membrane Z) Golgi apparatus, vesicles, smooth endoplasmic reticulum, plasma membrane

ANSWER: W) ROUGH ENDOPLASMIC RETICULUM, VESICLES, GOLGI APPARATUS, VESICLES, PLASMA MEMBRANE

BONUS BIOLOGY Multiple Choice If a student were to analyze the liquid in the nephrons of humans and the nephridia of earthworms, which of the following pairs of substances might be found in both organisms? W) Salts and urea X) Salts and sugar Y) Sugar and urea Z) Sugar and ammonia

ANSWER: W) SALTS AND UREA

BIOLOGY Multiple Choice In a landmark 1958 study by researchers Meselson and Stahl involving nitrogen isotope labeling, which of the following important DNA replication mechanisms was supported by their data? W) Semiconservative X) Dispersive Y) Conservative Z) Transformative

ANSWER: W) SEMICONSERVATIVE

BONUS BIOLOGY Multiple Choice Which of the following epithelial [ep-uh-THEE-lee-uhl] tissues is NOT correctly matched to its location in the human body? W) Simple cuboidal epithelium [KYOO-boid-uhl ep-uh-THEE-lee-uhm]—Rectal lining X) Simple squamous [SKWAY-muhs] epithelium—Alveoli lining Y) Stratified squamous epithelium—Esophageal [ih-sof-uh-JEE-uhl] lining Z) Pseudostratified ciliated columnar epithelium—Bronchiole lining

ANSWER: W) SIMPLE CUBOIDAL EPITHELIUM—RECTAL LINING

Multiple Choice: The type of muscle that is in the walls of human arteries is: W) smooth X) striated Y) skeletal Z) fast twitch

ANSWER: W) SMOOTH

BIOLOGY Multiple Choice Horseshoe crabs are most closely related to which of the following organisms? W) Spiders X) Millipedes Y) Centipedes Z) Insects

ANSWER: W) SPIDERS

BIOLOGY Multiple Choice Which of the following fish is endemic to the island of Cozumel? W) Splendid toadfish X) Coelacanth [SEE-luh-kanth] Y) Mola mola Z) Orange roughy

ANSWER: W) SPLENDID TOADFISH

BIOLOGY Multiple Choice Which of the following is NOT a seedless plant? W) St. John's wort X) Hornwort Y) Whisk fern Z) Liverwort

ANSWER: W) ST. JOHN'S WORT

BONUS BIOLOGY Multiple Choice Which of the following is NOT a result of parasympathetic nervous system stimulation? W) Stimulation of glucose release by the liver X) Contraction of pupils Y) Stimulation of salivation Z) Contraction of bronchi [BRONG-kee]

ANSWER: W) STIMULATION OF GLUCOSE RELEASE BY THE LIVER

BONUS BIOLOGY Multiple Choice Which of the following is NOT a result of parasympathetic nervous system stimulation? W) Stimulation of glucose release by the liver X) Contraction of pupils Y) Stimulation of salivation Z) Contraction of bronchi [BRONG-kee]

ANSWER: W) STIMULATION OF GLUCOSE RELEASE BY THE LIVER

BIOLOGY Multiple Choice Which of the following is the enzyme that maintains chromosomal stability by protecting chromosome ends? W) Telomerase [tuh-LOM-uh-rays] X) Nuclease [NOO-klee-ays] Y) Terminase Z) Polymerase [POL-uh-muh-rays]

ANSWER: W) TELOMERASE

BONUS BIOLOGY Multiple Choice Your body has approximately one million B cells and ten million different T cells. A gram-negative bacterium enters your body and one of your B cells binds to the bacterium via the BCR recognizing the LPS. Which of the following will happen next? W) The B cell will proliferate X) The virus will enter the B cell Y) Similar T cells will be destroyed Z) The B cell will be destroyed

ANSWER: W) THE B CELL WILL PROLIFERATE

BIOLOGY Multiple Choice Which of the following is an example of a covalent bond or bonds? W) The bond between two adjacent amino acids in a polypeptide chain X) The bond between a tRNA anticodon and an mRNA codon Y) The bonds between the large subunit and the small subunit of the ribosome [RY-buh-sohm] Z) The bonds between the small ribosomal subunit and the mRNA

ANSWER: W) THE BOND BETWEEN TWO ADJACENT AMINO ACIDS IN A POLYPEPTIDE CHAIN

BIOLOGY Multiple Choice The following are examples of proteins that are not functional because of some problem. In which of the following examples does the problem result from damage primarily to the secondary structure of the protein? W) The protein has been exposed to urea, which has disrupted its hydrogen bonds X) A mutation is at the active site of a protein and the protein has lost catalytic activity Y) A mutation is at the dimer [DY-mer] interface of a protein and prevents its dimerization Z) A protein has been exposed to an oily chemical that disrupts its hydrophobic core

ANSWER: W) THE PROTEIN HAS BEEN EXPOSED TO UREA, WHICH HAS DISRUPTED ITS HYDROGEN BONDS

BIOLOGY Multiple Choice Which of the following is NOT a characteristic of diatoms? W) They have a saccharide cell covering Y) They have a frustule [FRUHS-chool] made up of silica X) They contain carotenoids [kuh-ROT-n-oids], as well as chlorophyll a and b Z) Some species produce toxins

ANSWER: W) THEY HAVE A SACCHARIDE CELL COVERING

BIOLOGY Multiple Choice Which of the following is the term for the transfer of genes by viruses from one prokaryote to another: W) transduction X) conjugation Y) transformation Z) relaxation

ANSWER: W) TRANSDUCTION

BIOLOGY Multiple Choice Which of the following is NOT an example of a human disease caused by a recessive allele that fails to produce a viable cellular protein? W) Turner's syndrome X) Tay-sachs [tay-SAKS] Y) Cystic fibrosis Z) Phenylketonuria [fen-l-kee-toh-NOOR-ee-uh]

ANSWER: W) TURNER'S SYNDROME

BONUS BIOLOGY Multiple Choice Which of the following parts of B and T cells bind to antigens? W) Variable region X) Heavy chain Y) Alpha chain only Z) Disulfide bridge

ANSWER: W) VARIABLE REGION

Multiple Choice Which of the following is missing in bryophytes and is an evolutionary advancement of higher plants: W) vascular tissue X) epidermal tissue Y) chloroplasts Z) plasmodesmata (read as: PLAS-mo-dez-MAH-tah)

ANSWER: W) VASCULAR TISSUE

BONUS BIOLOGY Multiple Choice Hardy-Weinberg equilibrium is based on the assumption that allele frequencies in a population are stable. Which of the following conditions would be most likely to contribute to the stability of allele frequencies? W) Very large population X) Migration of individuals into or out of a population Y) Mutations Z) Natural selection

ANSWER: W) VERY LARGE POPULATION

BIOLOGY Multiple Choice Which of the following pairs are the most likely hydrogenbonding partners? W) Water and ammonia X) Water and estrogen Y) Water and carbonate Z) Water and methane

ANSWER: W) WATER AND AMMONIA

BIOLOGY Multiple Choice During the process of protein synthesis, what is the function of the TATA box and on what type of nucleic acid is it found? W) Where DNA replication begins, found on the DNA template X) Where transcription begins, found on the DNA Y) Where translation begins, found on the mRNA Z) Where elongation begins, found on the tRNA

ANSWER: W) WHERE DNA REPLICATION BEGINS, FOUND ON THE DNA TEMPLATE

BIOLOGY Multiple Choice In one form of the disease called macular degeneration, eyesight decreases due to overproduction of new blood vessels (angiogenesis) in the eye. Inhibition of a gene involved in angiogenesis can be achieved by introducing which of the following forms of complementary RNA molecule into the eye? W) siRNA X) Messenger RNA Y) lncRNA Z) Transfer RNA

ANSWER: W) siRNA

BONUS BIOLOGY Short Answer By name or number, give all of the following five choices that are components of bile in vertebrates: 1) water, 2) cholesterol, 3) lymph, 4) bilirubin, 5) amylase.

ANSWER: WATER, CHOLESTEROL, BILIRUBIN (ACCEPT: 1, 2, AND 4)

BONUS Biology - Short Answer What color does the normal human eye see when all cone opsins are stimulated equally?

ANSWER: WHITE

(General Science) [BONUS] Multiple Choice: Which of the following BEST describes the size of a typical animal virus: W) 600 micrometers X) 100 nanometers Y) 1 nanometer Z) 1 angstrom

ANSWER: X) 100 NANOMETERS

BIOLOGY Multiple Choice In which of the following wavelengths does DNA absorb the best: W) 3,900 angstroms X) 260 nanometers Y) 340 nanometers Z) 540 nanometers

ANSWER: X) 260 NANOMETERS

BIOLOGY Multiple Choice What is the predicted size in billions of base pairs of the woollymammoth genome? W) 1 X) 4 Y) 33 Z) 100

ANSWER: X) 4

BONUS BIOLOGY Multiple Choice How many fused rings does the carbon skeleton of steroids have? W) 2 X) 4 Y) 6 Z) 8

ANSWER: X) 4

BONUS BIOLOGY Multiple Choice Which of the following is closest to the diameter of a plasmodesmata (read as: PLAS-mo-dez-MAH-tah): W) 2 angstroms X) 60 nanometers Y) 80 microns Z) 180 microns

ANSWER: X) 60 NANOMETERS (Solution: this is the only logical size considering the average plant cell size and the size of transported molecules)

Multiple Choice In early experiments in the mid-1900's, the source and fate of oxygen involved in photosynthesis was mostly determined through experiments that used: W) animals in closed containers that would die unless a plant was introduced into the container X) a heavy isotope of oxygen as a tracer Y) fluorescent labeling of various enzymes and organic products Z) simple accounting of the atomic mass of all the reactants and products

ANSWER: X) A HEAVY ISOTOPE OF OXYGEN AS A TRACER

BIOLOGY Multiple Choice Which of the following proteins would likely be synthesized by a cytosolic ribosome instead of an ER-bound ribosome? W) Insulin X) Actin Y) Subunits for a potassium channel Z) G protein-coupled receptor

ANSWER: X) ACTIN

[Bonus] Multiple Choice: Which of the following is NOT true in humans: W) trisomy X originates from a nondisjunction X) all enzymes are proteins Y) albumin is produced by the liver Z) the spleen produces red blood cells in the fetus

ANSWER: X) ALL ENZYMES ARE PROTEINS

Biology - Multiple Choice Which of the following organisms probably uses PEP carboxylase to capture carbon dioxide and keep stomata closed during the day? W) Sugar cane X) Aloe vera Y) Corn Z) Sorghum

ANSWER: X) ALOE VERA

BIOLOGY Multiple Choice Which of the following BEST explains why leaves fall off trees in the autumn: W) the leaves die and passively fall off X) an abscission layer forms at the base of the leaf stem causing it to fall off Y) the leaves slowly dry out from the bottom up because of abscisic acid that prevents phloem from delivering nutrients Z) the vascular tissue of the leaves fill up with resin increasing their stiffness and causing them to become brittle

ANSWER: X) AN ABSCISSION LAYER FORMS AT THE BASE OF THE LEAF STEM CAUSING IT TO FALL OFF

BIOLOGY Multiple Choice You suspect that Cylon, an alien, may be the parent of a humanalien hybrid baby. The alien's blood type happens to be homozygous for blood type P for purple, the allele of which is symbolized by IP [eye superscript P]. As with other blood types, the P protein is co-dominant. If the human parent's blood type is A with a homozygous genotype, what blood phenotype would the baby have to be in order to be a confirmed human-alien hybrid baby? W) O X) AP Y) P Z) A

ANSWER: X) AP

BIOLOGY Multiple Choice In which of the following cell processes would caspase activity most likely occur? W) Mitosis X) Apoptosis Y) DNA replication Z) Phagocytosis

ANSWER: X) APOPTOSIS

Biology - Multiple Choice Baker's yeast is an example of what type of fungi? W) Basidiomycete [bah-sid-ee-oh-MY-seet] X) Ascomycete [az-koh-MY-seet] Y) Zygomycete [zye-go-MY-seet] Z) Imperfect fungus

ANSWER: X) ASCOMYCETE

BONUS BIOLOGY Multiple Choice How do bacteria protect their own genomic DNA from being cut by their restriction endonucleases [en-duh-NOO-klee-ays-is]? W) Genomic DNA of a bacterial species does not contain the recognition sequences for its own restriction endonucleases X) Bacteria apply protective methyl groups to DNA at sites recognized by their restriction endonucleases Y) Bacteria have proteins associated with their genomic DNA, preventing the endonucleases from getting close enough to cut Z) Sequestration of chromosomal DNA in the nucleoid region of a bacterial cell protects it from being cut by restriction endonucleases

ANSWER: X) BACTERIA APPLY PROTECTIVE METHYL GROUPS TO DNA AT SITES RECOGNIZED BY THEIR RESTRICTION ENDONUCLEASES

BIOLOGY Multiple Choice Which of the following organisms stores major food reserves as laminarin and oil instead of starch? W) Green algae X) Brown algae Y) Red algae Z) Flowering plants

ANSWER: X) BROWN ALGAE

BIOLOGY Multiple Choice Which of the following molecules do methanogens (read as: methAN-oh-gens) most often use as their final electron acceptor in their generation of energy: W) hydrogen X) carbon dioxide Y) methane Z) oxygen

ANSWER: X) CARBON DIOXIDE

BIOLOGY Multiple Choice Bacteria are classified as Gram positive or Gram negative based on several characteristics, including which of the following? W) Genome size X) Cell wall structure Y) Degree of virulence Z) Reproduction rate

ANSWER: X) CELL WALL STRUCTURE

BONUS BIOLOGY Multiple Choice In a double reciprocal plot of Michaelis-Menton kinetics, what is the effect of a competitive inhibitor: W) change in the vertical axis intercept X) change in the slope only Y) change in the horizontal axis intercept only Z) change in both vertical and horizontal axes intercepts

ANSWER: X) CHANGE IN THE SLOPE ONLY

Multiple Choice: Which of the following MOST closely represents the location of the temporal bone in humans: W) above the eye X) cheek or lateral border of the eye Y) back of skull Z) top of skull

ANSWER: X) CHEEK OR LATERAL BORDER OF THE EYE

BIOLOGY Multiple Choice Which of the following was first proposed by Peter Mitchell as a potential energy ion gradient across a selectively permeable membrane that is responsible for the production of ATP: W) osmosis X) chemiosmosis (read as: chem-ee-oz-MOE-sis): Y) electro-phosphorylation Z) photosynthesis

ANSWER: X) CHEMIOSMOSIS

(General Science)Multiple Choice In a recent breakthrough announcement by the CDC, which of the following diseases has been shown to have both a genetic and physiological component: W) malaria X) chronic fatigue syndrome Y) Tay Sach's disease Z) sleeping sickness

ANSWER: X) CHRONIC FATIGUE SYNDROME

BIOLOGY Multiple Choice Eggs exit a chicken's body through what specific structure: W) rectal plexus X) cloaca (read as: KLO-aye-ka) Y) rostrum Z) spiculum

ANSWER: X) CLOACA

BIOLOGY Multiple Choice Restriction endonucleases [en-duh-noo-klee-AY-sis] are found in bacteria and are thought to provide protection from viruses by cutting viral DNA into small fragments. Molecular biologists have used these enzymes to do which of the following? W) Fight viral infections in eukaryotes [yoo-KAR-ee-ohts] X) Clone specific genes into plasmids for expression by bacteria Y) Restrict the production of telomeres in yeast Z) Amplify desired DNA molecules in a controlled reaction

ANSWER: X) CLONE SPECIFIC GENES INTO PLASMIDS FOR EXPRESSION BY BACTERIA

BIOLOGY Multiple Choice Restriction endonucleases [en-duh-noo-klee-AY-sis] are found in bacteria and are thought to provide protection from viruses by cutting viral DNA into small fragments. Molecular biologists have used these enzymes to do which of the following? W) Fight viral infections in eukaryotes [yoo-KAR-ee-ohts] X) Clone specific genes into plasmids for expression by bacteria Y) Restrict the production of telomeres in yeast Z) Amplify desired DNA molecules in a controlled reaction

ANSWER: X) CLONE SPECIFIC GENES INTO PLASMIDS FOR EXPRESSION BY BACTERIA

(General Science) Multiple Choice As a close relative to the bacteria that cause botulism and gas gangrene, which of the following bacteria are a growing concern in hospitals, killing patients who have been treated with antibiotics: W) Bacillus anthracis (read as: an-THRAY-sis) X) Clostridium difficile Y) Treponema pallidium (read as: Trep-oh-KNEE-mah pal-LID-ee-um) Z) Vibrio cholera

ANSWER: X) CLOSTRIDIUM DIFFICILE

BONUS Biology - Multiple Choice Which of the following is true regarding the phylum Cnidaria [nihd-air-ee-uh]? W) Cnidarians include organisms such as flatworms and tapeworms X) Cnidarians usually have a motile medusa form and a non-motile polyp form Y) Cnidarians form three germ layers during development, one of which forms the coelem Z) Gastropoda is a class within the phylum Cnidaria

ANSWER: X) CNIDARIANS USUALLY HAVE A MOTILE MEDUSA FORM AND A NON-MOTILE POLYP FORM

BONUS BIOLOGY Multiple Choice Which of the following is a group of about 20 proteins that aid in the attraction and phagocytosis of microorganisms by macrophages: W) alpha interferons X) complement system Y) TNF Z) hemopoietins

ANSWER: X) COMPLEMENT SYSTEM

BIOLOGY Multiple Choice A child who excretes black urine is born to two normal parents. The child has a recessive homozygous genotype. What is the most likely explanation for the genotype? W) Spontaneous mutations X) Consanguinity [kon-sang-GWIN-i-tee] Y) Anticipation Z) Environmental exposure

ANSWER: X) CONSANGUINITY

(General Science) [Bonus] Multiple Choice: Which of the following is one of the MAIN ways of preventing Brucellosis in humans: W) avoid person-to-person contact X) consume only pasteurized dairy products Y) avoid flea bites Z) boil all drinking water

ANSWER: X) CONSUME ONLY PASTEURIZED DAIRY PRODUCTS

BONUS Biology - Multiple Choice Which of the following compounds would one use to detect the presence of protein in a polyacrylamide gel? W) Ethidium bromide [eth-ih-dee-um broh-myde] X) Coomassie Blue [coo-mah-see] Y) Rhodamine 2 [row-duh-meen] Z) Fluorescein [flow-ruh-seen]

ANSWER: X) COOMASSIE BLUE

[bonus] Multiple Choice Which of the following is a primary function of histones H2A, H2B, H3 and H4: W) preventing damage to DNA from ionizing radiation X) densely packaging DNA into nucleosomes Y) holding DNA in the double helical configuration Z) directing the packaging of DNA into looped domains of about 300 nanometers

ANSWER: X) DENSELY PACKAGING DNA INTO NUCLEOSOMES

[bonus] Multiple Choice In mammalian kidneys, reabsorption of water occurs mostly in which of the following: W) glomerulus X) descending limb of the loop of Henle (read as: HEN-lee) Y) proximal convoluted tubule Z) Bowman's capsule

ANSWER: X) DESCENDING LIMB OF THE LOOP OF HENLE

Biology - Multiple Choice Which of the following organisms would use the highest percentage of its energy budget for homeostatic regulation? W) Marine hydra X) Desert bird Y) Desert insect Z) Marine jellyfish

ANSWER: X) DESERT BIRD

BIOLOGY Multiple Choice In which of the following organ systems of the human body do chief cells primarily function: W) excretory X) digestive Y) nervous Z) reproductive

ANSWER: X) DIGESTIVE

BIOLOGY Multiple Choice Which of the following is the mechanism of action of most insecticides? W) Blocking oxygen intake X) Disrupting nerve function Y) Preventing digestion Z) Stopping DNA synthesis

ANSWER: X) DISRUPTING NERVE FUNCTION

BONUS BIOLOGY Multiple Choice During DNA replication, which of the following enzymes is mismatched to its function? W) DNA helicase: unwinds DNA strands X) DNA primase: creates DNA primer to start replication Y) DNA polymerase: joins nucleotides together Z) Single strand binding protein: prevents parental DNA strands from re-annealing

ANSWER: X) DNA PRIMASE: CREATES DNA PRIMER TO START REPLICATION

(General Science) [bonus] Multiple Choice Which of the following is the BEST reason why Parkinson's disease cannot be treated by simply administering dopamine into the blood stream: W) dopamine's blood half-life is too short X) dopamine will not cross the blood brain barrier Y) dopamine is the precursor to the real effective transmitter in Parkinson's disease Z) it is not known why dopamine will not work if administered directly into the blood

ANSWER: X) DOPAMINE WILL NOT CROSS THE BLOOD BRAIN BARRIER

BONUS Biology - Multiple Choice Which of the following immune cells is most involved in defense against parasites? W) Neutrophils X) Eosinophils [ee-oh-SIN-oh-filz] Y) Basophils [BAY-zoh-filz] Z) Mast cells

ANSWER: X) EOSINOPHILS

BONUS BIOLOGY Multiple Choice Which of the following pollinators is a member of the order Lepidoptera [lep-i-DOP-ter-uh]? W) Lesser long-nosed bats X) Fritillary butterflies [FRIT-l-er-ee] Y) Green bottle flies Z) Blue orchard bees

ANSWER: X) FRITILLARY BUTTERFLIES

Biology - Multiple Choice Which of the following organelles is the location of post-translation modifications to proteins? W) Smooth ER X) Golgi body Y) Glyoxysome [gligh-ox-ih-sohm] Z) Mitochondria

ANSWER: X) GOLGI BODY

BIOLOGY Multiple Choice Which of the following pairs includes a toothed whale and a baleen whale? W) Bowhead whale and blue whale X) Grey whale and beluga whale Y) Sperm whale and narwhal Z) Northern right whale and minke

ANSWER: X) GREY WHALE AND BELUGA WHALE

(General Science) Multiple Choice To remove this parasite that erupts from the skin, victims will wrap the end of the emerging worm around a stick and turn the stick over a period of days to slowly remove the worm: W) Ascaris X) guinea worm Y) hookworm Z) strongyloid

ANSWER: X) GUINEA WORM

BIOLOGY Multiple Choice The taxonomic class Myxini [MIK-suh-nee] contains which of the following fish? W) Wolf eels X) Hagfish Y) Chimaeras [ki-MEER-uhs] Z) Bowfin

ANSWER: X) HAGFISH

BIOLOGY Multiple Choice Which of the following molecules would have the most peptide bonds: W) glycogen X) hemoglobin Y) lecithin Z) DNA

ANSWER: X) HEMOGLOBIN

BONUS BIOLOGY Multiple Choice Which of the following is an example of a protein with quaternary structure? W) Myoglobin [my-uh-GLOH-bin] X) Hemoglobin Y) Cytochrome C [SY-tuh-krohm C] Z) Chymotrypsin [ky-mo-TRIP-sin]

ANSWER: X) HEMOGLOBIN

Multiple Choice: Robert Hooke coined the term 'cell' mostly because of: W) his observations of red blood cells X) his observations of cork cells Y) his analysis of hundreds of different protozoans Z) his readings of the works of other early microscopists

ANSWER: X) HIS OBSERVATIONS OF CORK CELLS

BONUS BIOLOGY Multiple Choice Which of the following insect orders is characterized by the beak or stylet arising from the posterior part of the head and extending along the insect's sternum? W) Hymenoptera [hy-muh-NOP-ter-uh] X) Homoptera [hoh-MOHP-tuh-ruh] Y) Coleoptera [koh-lee-OP-ter-uh] Z) Hemiptera [hi-MIP-ter-uh]

ANSWER: X) HOMOPTERA

Multiple Choice: Which of the following is a homologous structure to a whale flipper: W) insect wing X) human arm Y) a fish operculum Z) shark fin

ANSWER: X) HUMAN ARM

Biology - Multiple Choice Which of the following would be the most likely explanation for hypothyroidism in a patient whose iodine level is normal? W) Disproportionate production of T3 to T4 X) Hyposecretion of TSH Y) Hypersecretion of TSH Z) Hypersecretion of MSH

ANSWER: X) HYPOSECRETION OF TSH

BONUS BIOLOGY Multiple Choice In contrast to the lysogenic [ly-suh-JEN-ik] cycle of a bacteriophage [bak-TEER-ee-uh-fayj], the lytic cycle does which of the following? W) Integrates phage DNA into the host chromosome X) Involves the dissolution of host cells to release new phages Y) Features prophages that can be excised from the host chromosome Z) Copies phage DNA when the cell divides

ANSWER: X) INVOLVES THE DISSOLUTION OF HOST CELLS TO RELEASE NEW PHAGES

BONUS BIOLOGY Multiple Choice Transfer RNA has all of the following characteristics, EXCEPT which of the following? W) It travels from the nucleus to the cytoplasm in eukaryotes X) It is attached to an amino acid at the 5 prime end Y) Its three dimensional structure is roughly L-shaped Z) It can be used repeatedly

ANSWER: X) IT IS ATTACHED TO AN AMINO ACID AT THE 5 PRIME END

BIOLOGY Multiple Choice Which of the following statements is NOT true about lichen [LYkuhn]? W) It is a composite organism X) It is composed of algae and moss Y) It is hardy and slow-growing Z) It usually reproduces asexually

ANSWER: X) IT IS COMPOSED OF ALGAE AND MOSS

[BONUS] Multiple Choice: Which of the following is NOT a blood vessel located in the human head or neck: W) jugular vein X) jugular artery Y) internal carotid artery Z) temporal vein

ANSWER: X) JUGULAR ARTERY

BONUS BIOLOGY Multiple Choice What is the joint that is formed by the articulation of the tibia, fibula, and femur: W) wrist X) knee Y) elbow Z) ankle

ANSWER: X) KNEE

BONUS BIOLOGY Multiple Choice Which of the following processes is MOST under the influence of prolactin? W) Growth and maturation of the uterine lining [YOO-ter-in] X) Lactation Y) Follicular maturation in the ovaries Z) Stimulation of uterine contractions

ANSWER: X) LACTATION

BIOLOGY Multiple Choice Which of the following is NOT a tool to develop homeothermy [hoh-mee-uh-THERM-ee] in baby birds? W) Large brood size X) Large toes Y) Down Z) Large leg muscles

ANSWER: X) LARGE TOES

[Bonus] Multiple Choice Which of the following is an insertion of the human biceps femoris muscle: W) ventral portion of the head of the femur X) lateral sides of the head of the tibia Y) ileac crest Z) pubic symphisis

ANSWER: X) LATERAL SIDES OF THE HEAD OF THE TIBIA

BIOLOGY Multiple Choice Which of the following sea turtle species is phylogenetically distinct from the others and can maintain body temperature elevated above ambient levels? Y) Kemp's Ridley X) Leatherback Y) Green Z) Loggerhead

ANSWER: X) LEATHERBACK

BONUS BIOLOGY Multiple Choice Cloning a gene into a plasmid often requires you to amplify the target DNA sequence, then digest and paste the target sequence into the plasmid. Several purified enzymes are important in this process. Which of the following enzymes would act LAST in the cloning process? W) Polymerase [POL-uh-muh-rays] X) Ligase [LY-gays] Y) Exonuclease [eks-uh-NOO-klee-ays] Z) Restriction endonuclease [en-duh-NOO-klee-ays]

ANSWER: X) LIGASE

BONUS BIOLOGY Multiple Choice Which of the following scenarios most closely resembles HardyWeinberg equilibrium W) Lizards on a large, isolated island with scarce, scattered food resources X) Lizards on a large, isolated island with plentiful food resources Y) Birds on a small island that have highly selective mating behavior Z) Birds on a small island that fly frequently back and forth to the mainland

ANSWER: X) LIZARDS ON A LARGE, ISOLATED ISLAND WITH PLENTIFUL FOOD RESOURCES

BONUS BIOLOGY Multiple Choice All of the following factors increase the release of oxygen from hemoglobin in systemic capillaries, EXCEPT: W) High temperature X) Low temperature Y) Acidic pH levels Z) Low partial pressure of oxygen

ANSWER: X) LOW TEMPERATURE

BONUS Biology - Multiple Choice Conifers often have needlelike leaves, which are an adaptation to arid conditions. The slopes of mountains, however, often have dense conifer populations and experience a large amount of rainfall each year. Which of the following correctly explains why trees with adaptations to arid climates flourish near mountains? W) Albedo near mountains is high, desiccating normal leaves X) Most soil water is frozen, rendering it unavailable to the tree Y) The high winds near mountains increase the rate of desiccation Z) The soil around mountains drains poorly

ANSWER: X) MOST SOIL WATER IS FROZEN, RENDERING IT UNAVAILABLE TO THE TREE

BIOLOGY Multiple Choice Different cell junctions are used to connect neighboring cells as needed for their functions. Which of the following options describes a situation that is utilizing desmosomes [DEZ-muh-sohms]? W) In developing embryos, ions can travel through a pore connecting the two membranes of adjacent cells to permit rapid cell communication X) Muscle cells are joined to form a sturdy sheet Y) Skin cells are joined to make a barrier impervious to even water Z) In a plant cell, small solutes in the cytoplasm will travel from cell to celld

ANSWER: X) MUSCLE CELLS ARE JOINED TO FORM A STURDY SHEET

BIOLOGY Multiple Choice Which of the following is typically derived from ectoderm [EK-tuhdurm]? W) Areolar connective tissue X) Nervous tissue Y) Blood Z) Simple squamous epithelium [SKWAY-muhs ep-uh-THEE-lee-uhm]

ANSWER: X) NERVOUS TISSUE

BONUS Biology - Multiple Choice Which of the following is true regarding the differences between New World and Old World monkeys? W) New World monkeys evolved from prosimians, while Old World monkeys evolved from anthropoids X) New World monkeys are all arboreal, while Old World monkeys include some ground dwelling species Y) New World monkeys lack prehensile tails, while Old World monkeys have prehensile tails Z) New World monkeys have downward pointed noses, while Old World monkeys have flat, spreading noses

ANSWER: X) NEW WORLD MONKEYS ARE ALL ARBOREAL, WHILE OLD WORLD MONKEYS INCLUDE SOME GROUND-DWELLING SPECIES

BIOLOGY Multiple Choice During meiosis, when a cell has lost both members of a homologous pair of chromosomes, which of the following types of aneuploidy [an-yoo-PLOI-dee] would apply to the cell? W) Tetrasomy X) Nullisomy Y) Monosomy Z) Trisomy

ANSWER: X) NULLISOMY

BONUS BIOLOGY Multiple Choice Carbohydrates are best defined as which of the following? W) Inorganic compounds containing carbon, hydrogen, and oxygen in a ratio of 1:2:1 X) Organic compounds containing carbon, hydrogen, and oxygen in a ratio of 1:2:1 Y) Organic compounds containing carbon, hydrogen, and oxygen in a ratio of 2:1:2 Z) Inorganic compounds containing carbon, hydrogen, and oxygen in a ratio of 2:1:2

ANSWER: X) ORGANIC COMPOUNDS CONTAINING CARBON, HYDROGEN, AND OXYGEN IN A RATIO OF 1:2:1

BONUS BIOLOGY Multiple Choice Which of the following biological molecules most directly enables the structural compartmentalization required for cellular respiration and photosynthesis [fohtuh-SIN-thuh-sis]? W) Amino acids X) Phospholipids [fos-foh-LIP-ids] Y) Glycogen [GLY-kuh-juhn]term-634 Z) Cholesterol

ANSWER: X) PHOSPHOLIPIDS

BONUS BIOLOGY Multiple Choice Which of the following statements is true regarding the mechanism of infection in prion diseases? W) Prions invade the cell nucleus, altering DNA transcription processes X) Prions induce abnormal folding in normal protein molecules Y) Prions bind to specific sites on a ribosome, rendering them useless Z) Prions bind to mRNA, leaving the nucleus, inducing post-transcriptional changes before translation

ANSWER: X) PRIONS INDUCE ABNORMAL FOLDING IN NORMAL PROTEIN MOLECULES

Multiple Choice Which of the following BEST meets the premises of the HardyWeinberg equilibrium: W) random mating, small populations, and no genetic load X) random mating, large populations, and no genetic load Y) non-random mating, small populations, and no net mutations Z) non-random mating, large populations, and no net mutations

ANSWER: X) RANDOM MATING, LARGE POPULATIONS, AND NO GENETIC LOAD

BONUS BIOLOGY Multiple Choice With which of the following are cyclin-dependent kinases most directly involved: W) repairing damaged DNA X) regulating cell cycle Y) digesting plasmids Z) insulin's action on fat cells

ANSWER: X) REGULATING CELL CYCLE

[bonus] Multiple Choice Which of the following MOST likely occurs during speciation (read as: spee-cee-A-tion): W) increased mutation rates X) reproductive isolation Y) elimination of recessive alleles Z) increase in genetic load

ANSWER: X) REPRODUCTIVE ISOLATION

BONUS BIOLOGY Multiple Choice Which of the following molecules catalyzes the formation of peptide bonds between amino acids? W) Protease X) Ribosome Y) Pepsin Z) Kinase

ANSWER: X) RIBOSOME

BIOLOGY Multiple Choice Which of the following animals is considered a keystone species of the kelp forests of the Aleutian [uh-LOO-shuhn] Islands ecosystem? W) Sea urchin X) Sea otter Y) Killer whale Z) Alaskan fur seals

ANSWER: X) SEA OTTER

Biology - Multiple Choice Which of the following types of epithelial tissue would most likely be found lining the lungs and blood vessels? W) Stratified columnar X) Simple squamous Y) Simple cuboidal Z) Simple columnar

ANSWER: X) SIMPLE SQUAMOUS

BIOLOGY Multiple Choice Which of the following mutations would most likely affect a gene the LEAST: W) single base deletion X) single base substitution Y) single base insertion Z) quadruple base deletion

ANSWER: X) SINGLE BASE SUBSTITUTION

BONUS Biology - Multiple Choice Which of the following is the most superficial layer of the epidermis? W) Stratum [STRAAT-um] lucidum [LOO-sih-dum] X) Stratum corneum [KOR-nee-um] Y) Stratum spinosum [spih-NOH-sum] Z) Stratum granulosum [gran-yoo-LOH-sum]

ANSWER: X) STRATUM CORNEUM

BIOLOGY Multiple Choice Which of the following is NOT part of initiation during translation? W) Binding of a small ribosomal subunit to mRNA and initiator tRNA X) TATA box Y) One GTP molecule Z) Protein initiation factors

ANSWER: X) TATA BOX

BONUS BIOLOGY Multiple Choice When actin reaches its steady state in a cell, which of the following is occurring? W) The actin filament is no longer adding new subunits or losing existing subunits X) The actin filament is constantly adding new and losing existing subunits Y) The actin filament forms new disulfide bonds within itself Z) The actin filament forms new alpha helix structures to maintain its shape

ANSWER: X) THE ACTIN FILAMENT IS CONSTANTLY ADDING NEW AND LOSING EXISTING SUBUNITS

BIOLOGY Multiple Choice Which of the following statements best describes the thermodynamic effect of an enzyme on the reaction it catalyzes? W) The energy level of the reactants is decreased in the presence of the enzyme X) The activation energy of the reaction is decreased Y) The activation energy of the reaction is increased Z) The ΔG [delta-G] between products and reactants is decreased

ANSWER: X) THE ACTIVATION ENERGY OF THE REACTION IS DECREASED

Multiple Choice If the human SRY gene is inactivated in the first week of gestation, which of the following will occur: W) the embryo will die X) the embryo will remain female Y) the embryo will continue as a male Z) the embryo will remain in a quiescent state

ANSWER: X) THE EMBRYO WILL REMAIN FEMALE

BONUS BIOLOGY Multiple Choice A Paramecium [par-uh-MEE-see-uhm] lives in fresh water and has a contractile vacuole [kuhn-TRAK-tl VAK-yoo-ohl] that allows it to regularly expel excess water that enters its cell. Which of the following is correct? W) The inside of the cell is hypertonic compared to the outside of the cell, so water enters the cell through osmosis X) The inside of the cell is hypotonic to the outside of the cell, so water enters the cell through osmosis Y) The inside and outside of the cell are isotonic, but water enters the cell as the Paramecium swims around Z) The cellular processes of the Paramecium produce a lot of excess water, so it needs to get rid of the extra water

ANSWER: X) THE INSIDE OF THE CELL IS HYPOTONIC TO THE OUTSIDE OF THE CELL, SO WATER ENTERS THE CELL THROUGH OSMOSIS

BIOLOGY Multiple Choice The amino acid sequence of a protein represents the translation of which of the following? W) The template DNA strand X) The nontemplate DNA strand Y) The tRNA nucleotide sequence [NOO-klee-uh-tyd] Z) The reverse-complement of the mRNA strand

ANSWER: X) THE NONTEMPLATE DNA STRAND

BONUS BIOLOGY Multiple Choice Kearns-Sayre Syndrome is caused by a mutation on a gene in the mitochondrial [my-tuh-KON-dree-uhl] DNA. Both males and females can have the disease, but the disease can only be transmitted to a child by the mother, never the father. Why is this so? W) The gene is on the X chromosome, and females are XX X) The sperm that fertilizes the egg contributes only its chromosomes and none of its mitochondria Y) The mutant protein is only active in the egg, not the sperm Z) The disease is sex-limited

ANSWER: X) THE SPERM THAT FERTILIZES THE EGG ONLY CONTRIBUTES ITS CHROMOSOMES AND NONE OF ITS MITOCHONDRIA

BIOLOGY Multiple Choice Which of the following statements is NOT true about spike moss? W) They do not have true roots X) They have a well-developed vascular system Y) They are widely distributed in all parts of the world Z) Their leaves are spore-bearing

ANSWER: X) THEY HAVE A WELL-DEVELOPED VASCULAR SYSTEM

BIOLOGY Multiple Choice Which of the following is true of enzymes? W) They increase the activation energy of metabolic reactions X) They have active sites that bind to specific substrate molecules Y) They are made of chitin [KY-tin] protein Z) They generally act on a variety of substrate molecules

ANSWER: X) THEY HAVE ACTIVE SITES THAT BIND TO SPECIFIC SUBSTRATE MOLECULES

BONUS BIOLOGY Multiple Choice Which of the following statements is NOT correct regarding prions? W) They can cause scrapie and Creutzfeldt-Jakob diseases [KROYTS-felt Jakob] X) They infect the same taxa [TAK-suh] as viroids [VY-roids] Y) They can cause degenerative brain diseases Z) They are an infectious agent comprised entirely of protein

ANSWER: X) THEY INFECT THE SAME TAXA AS VIROIDS

[bonus] Multiple Choice Which of the following is NOT characteristic of retroviruses: W) they have single-stranded RNA as their genetic material X) they make mRNA from their original RNA Y) they incorporate their genome into the DNA of the host cell using reverse transcriptase Z) the host cell polymerase transcribes viral genes

ANSWER: X) THEY MAKE mRNA FROM THEIR ORIGINAL RNA

BONUS BIOLOGY Multiple Choice In which of the following glands do hematopoietic [hi-mat-ohpoi-EE-tik] stem cells develop into T cells? W) Pineal [PIN-ee-uh] X) Thymus [THY-muhs] Y) Pituitary [pi-TOO-i-ter-ee] Z) Adrenal

ANSWER: X) THYMUS

BIOLOGY Multiple Choice Which of the following bacterial gene transfer mechanisms requires the involvement of a bacteriophage [bak-TEER-ee-uh-fayj]? W) Conjugation X) Transduction Y) Horizontal gene transfer Z) Bacterial transformation

ANSWER: X) TRANSDUCTION

BONUS BIOLOGY Multiple Choice At a 4th of July picnic, Uncle Vernon was munching down on his ear of yellow corn when he exclaimed, "Why do some of these corn kernels have purple spots? Is the corn toxic?" You quickly explain that the corn is not toxic and that the spots were caused by what? W) Transcription start variation X) Transposition of a jumping gene Y) Random X-inactivation Z) Genetic modifications

ANSWER: X) TRANSPOSITION OF A JUMPING GENE

BIOLOGY Multiple Choice If a plant had a taproot, it would also most likely have: W) parallel leaf venation X) two cotyledons in its seedling stage Y) diffusely arranged vascular bundles in its stem Z) no stomata on the upper surfaces of its leaves

ANSWER: X) TWO COTYLEDONS IN ITS SEEDLING STAGE

BONUS GENERAL SCIENCE Multiple Choice Which of the following is a mixing incompatibility of common household products that can produce toxic products: W) vinegar and baking soda X) vinegar and bleach Y) vinegar and borax Z) castile soap and salt

ANSWER: X) VINEGAR AND BLEACH (Solution: small amounts of vinegar can be added to bleach to acidify the bleach, but it's generally not a good idea to mix these two solutions)

BIOLOGY Multiple Choice Which of the following infectious agents should a human be least concerned about contracting? W) Prion X) Viroid Y) Retrovirus Z) RNA virus

ANSWER: X) VIROID

BIOLOGY Multiple Choice After a volcanic eruption wipes out the organisms on an island, which of the following would be the first to recolonize? W) Large carnivore X) Wind-dispersed grass Y) Frugivorous bird [froo-JIV-er-uhs] Z) Forest tree

ANSWER: X) WIND-DISPERSED GRASS

Biology - Multiple Choice Which of the following methods was used to determine the 3D structure of hemoglobin? W) Circular dichroism [dye-croh-i sm] X) X-ray crystallography Y) Protein NMR Z) Electrolysis

ANSWER: X) X-RAY CRYSTALLOGRAPHY

BONUS BIOLOGY Short Answer Hemophilia A and Duchenne [duh-SHAYN] muscular dystrophy are examples of inherited genetic disorders that show which specific pattern of inheritance?

ANSWER: X-LINKED RECESSIVE (ACCEPT: SEX-LINKED RECESSIVE; DO NOT ACCEPT ALONE: X-LINKED, RECESSIVE)

BONUS BIOLOGY Multiple Choice The mRNA codon for methionine [meh-THY-uh-neen] is 5'-AU-G-3'. From 5 prime to 3 prime, which of the following would be the anticodon present in the cognate tRNA? W) 5'-UAC-3' X) 5'-GUA-3' Y) 5'-CAU-3' Z) 5'-AUG-3' [NOTE: 3' = THREE PRIME and 5' = FIVE PRIME]

ANSWER: Y) 5'-CAU-3'

BIOLOGY Multiple Choice An animal's respiratory tidal volume is 500 milliliters and its cardiac output is 250 milliliters per minute with a stroke volume of 5 milliliters. Which of the following is its heart rate in beats per minute? W) 10 X) 20 Y) 50 Z) 100

ANSWER: Y) 50

(General Science) Multiple Choice Which of the following BEST describes the size of the average amoeba: W) 1 angstrom X) 2 micrometers Y) 600 micrometers Z) 1 centimeter

ANSWER: Y) 600 MICROMETERS

BIOLOGY Multiple Choice G-protein coupled receptors regulate various cell signaling events in a cell. How many transmembrane segments are typically found in these receptors? W) 5 X) 6 Y) 7 Z) 9

ANSWER: Y) 7

BONUS BIOLOGY Multiple Choice Which of the following is LEAST accurate regarding human genetics: W) genes from gametes are typically unchanged during their passage to the zygote X) the zygote gets 1 allele from each parental gene pair Y) all genes independently segregate Z) the number of copies of a particular gene can vary among individuals

ANSWER: Y) ALL GENES INDEPENDENTLY SEGREGATE (Solution: genes on the same chromosome do not independently segregate)

BIOLOGY Multiple Choice A man with an X-linked dominant trait has children with a female who is homozygous [hoh-muh-ZY-guhs] recessive for the trait. Which of the following proportions of his offspring will be affected by the trait? W) All of his children, male or female X) All his male children Y) All his female children Z) Half of his female children

ANSWER: Y) ALL HIS FEMALE CHILDREN

BIOLOGY Multiple Choice As could be detected clinically during a functional magnetic resonance image, which of the following parts of the brain would be activated when a person is faced with a fearful situation? W) Neocortex X) Hippocampus [hip-uh-KAM-puhs] Y) Amygdala [uh-MIG-duh-luh] Z) Superior colliculi [ko-LI-kyoo-lee]

ANSWER: Y) AMYGDALA

BONUS BIOLOGY Multiple Choice Which of the following types of fatty acids would alpha linolenic or all cis-9,12,15-Octadecatrienoic (read as: octa-deca-tri-en-oh-ick) acid most commonly be classified: W) an alpha 9 fatty acid X) an alpha 9,12,15 fatty acid Y) an omega 3 fatty acid Z) an omega 6 fatty acid

ANSWER: Y) AN OMEGA 3 FATTY ACID

BONUS BIOLOGY Multiple Choice Neurons can communicate with each another at synapses via synaptic vesicles. Which of the following does NOT participate in communication at these sites? W) Neurotransmitter X) Gated channels Y) ATP dependent channels Z) Acetylcholine [uh-seet-l-KOH-leen]

ANSWER: Y) ATP DEPENDENT CHANNELS

BONUS BIOLOGY Multiple Choice Through which of the following mechanisms do allosteric inhibitors typically operate: W) binding the substrate X) binding to an enzyme's active site Y) binding to a site other than the active site and changing the shape of an enzyme Z) acting as a competitive inhibitor and overpowering the active sites of enzymes

ANSWER: Y) BINDING TO A SITE OTHER THAN THE ACTIVE SITE AND CHANGING THE SHAPE OF AN ENZYME

Biology - Multiple Choice A deletion mutation that results in loss of the final 3 nucleotides of an organism's histidine tRNA gene will impair what function of the tRNA? W) Folding into functional tertiary structure X) Codon-anticodon base pairing with mRNA Y) Binding to histidine Z) Transcription of the gene encoding histidine

ANSWER: Y) BINDING TO HISTIDINE

BONUS BIOLOGY Multiple Choice Which of the following BEST describes how Pasteur attenuated the rabies virus to create an effective vaccine: W) by heat-treating the purified virus X) by treating the live virus with a weak solution of formaldehyde Y) by passing the rabies infection through a successive series of rabbits Z) by purifying the protein from the viral capsids

ANSWER: Y) BY PASSING THE RABIES INFECTION THROUGH A SUCCESSIVE SERIES OF RABBITS

BIOLOGY Multiple Choice Of which of the following is endosperm in wheat primarily composed: W) protein X) nucleic acid Y) carbohydrate Z) lipid

ANSWER: Y) CARBOHYDRATE

BIOLOGY Multiple Choice Which of the following is the main target of bacteria by the antibiotic penicillin: W) energy production X) DNA synthesis Y) cell wall synthesis Z) translation

ANSWER: Y) CELL WALL SYNTHESIS

BIOLOGY Multiple Choice Which of the following does the enzyme aminoacyl-tRNA synthetase [uh-mee-noh-AS-il T-R-N-A SIN-thuh-tays] catalyze? W) Base-pairing between the tRNA and the mRNA X) Transfer of the amino acid from the tRNA to form a peptide bond Y) Charging of a tRNA with an amino acid Z) Synthesis of tRNA molecules

ANSWER: Y) CHARGING OF A tRNA WITH AN AMINO ACID

BIOLOGY Multiple Choice Which of the following marine microbe groups has a cell membrane mainly comprised of lipids? W) Foraminiferans [for-uh-MIN-uh-fer-uhns] X) Radiolarians [ray-dee-oh-LAIR-ee-uhns] Y) Ciliates [SIL-ee-ayts] Z) Tritaxis

ANSWER: Y) CILIATES

BIOLOGY Multiple Choice What osteichthyes [os-tee-IK-thee-eez] was known from the fossil record for nearly 100 years before a living specimen was discovered? W) Australian lungfish X) Beluga sturgeon Y) Coelacanth [SEE-luh-kanth] Z) Bowfin

ANSWER: Y) COELACANTH

BONUS BIOLOGY Multiple Choice Which of the following is true regarding osmoregulation in a freshwater fish? W) Osmotic water loss occurs over body surface and via gills X) Nitrogenous wastes are excreted primarily in the form of uric acid Y) Copious amount of urine hypotonic to the blood is excreted Z) Excess salt is excreted from gills

ANSWER: Y) COPIOUS AMOUNT OF URINE HYPOTONIC TO THE BLOOD IS EXCRETED

BONUS BIOLOGY Multiple Choice Which of the following chemical bonds does NOT correctly characterize the interaction between two molecules? W) Ester linkage: ribose to phosphate X) Peptide bond: amino acid to amino acid Y) Covalent bond: anticodon to codon Z) Hydrogen bond: adenine to thymine

ANSWER: Y) COVALENT BOND: ANTICODON TO CODON

Multiple Choice: Which of the following occurs in meiosis but NOT in mitosis: W) spindle apparatus attaches to kinetochores X) nuclear membrane is not visible Y) crossing over Z) chromosomes line up at central area of cell

ANSWER: Y) CROSSING OVER

BONUS BIOLOGY Multiple Choice Microtubules, intermediate filaments, and microfilaments are integral components of what basic infrastructure of a eukaryotic cell? W) flagella X) cillia Y) cytoskeleton Z) cell wall

ANSWER: Y) CYTOSKELETON

BONUS Biology - Multiple Choice Which of the following is not a medically relevant difference between eukaryotic cells and bacterial cells? W) Differences in ribosome structure X) Presence of peptidoglycan layer in bacteria Y) Differences in genetic code Z) Presence of lipopolysaccharides [l ye-p oh-p oly-s ak-kar-i des]in bacteria

ANSWER: Y) DIFFERENCES IN GENETIC CODE

BIOLOGY Multiple Choice In elephant seals, sexual selection favors competition among males for female mates. Which of the following is a likely result of this phenomenon? W) Sexual monomorphism [mon-uh-MOHR-fis-uhm] X) Evolution of female handicap structures Y) Directional selection toward large male size Z) Conspicuous coloration of females

ANSWER: Y) DIRECTIONAL SELECTION TOWARD LARGE MALE SIZE

BIOLOGY Multiple Choice The helix-turn-helix motif is characteristic of a large group of proteins. Which of the following is a common function of the helix-turn-helix motif? W) Insertion into the cell membrane X) Lipid hydrolysis Y) DNA binding Z) Glycosidic bond formation

ANSWER: Y) DNA BINDING

BONUS Biology - Multiple Choice Which of the following roles does creatine play in the human body? W) Intermediate in protein synthesis X) Coenzyme associated with purine breakdown Y) Energy source in muscles Z) Enzyme in the urea cycle

ANSWER: Y) ENERGY SOURCE IN MUSCLES

BIOLOGY Multiple Choice The proboscis [proh-BOS-kis] is used primarily by invertebrates to perform which of the following functions? W) Egg fertilization X) Excavation Y) Feeding Z) Defense

ANSWER: Y) FEEDING

BONUS BIOLOGY Multiple Choice Which of the following microscopy methods would be most appropriate for observing the distribution of GFP in a cell? W) Simple light microscope X) Atomic force microscope Y) Fluorescence light microscope Z) Transmission electron microscope

ANSWER: Y) FLUORESCENCE LIGHT MICROSCOPE

BIOLOGY Multiple Choice Female insects usually lay their eggs near which of the following? W) Light X) Shade Y) Food Z) Water

ANSWER: Y) FOOD

BONUS BIOLOGY Multiple Choice Which of the following will most likely result from a single base deletion in DNA: W) nonsense mutation X) stop codon Y) frame shift mutation Z) inversion mutation

ANSWER: Y) FRAME SHIFT MUTATION

BONUS BIOLOGY Multiple Choice The basilar membrane in the cochlea [KOK-lee-uh] is differentially sensitive to which of the following aspects of sound? W) Amplitude X) Direction Y) Frequency Z) Bandwidth

ANSWER: Y) FREQUENCY

BIOLOGY Multiple Choice Which of the following is necessary for the evolution of pesticide resistance? W) A population that is highly susceptible to the pesticide X) An extremely lethal pesticide Y) Genetic variation in susceptibility to the pesticide Z) Heterogeneous [het-er-uh-JEE-nee-uhs] application of pesticide

ANSWER: Y) GENETIC VARIATION IN SUSCEPTIBILITY TO THE PESTICIDE

BONUS BIOLOGY Multiple Choice Which of the following is NOT true regarding glycogen: W) glycogen synthesis is endergonic (read as: en-der-GONE-ick) X) glycogen is a polysaccharide Y) glycogen side branches have beta 1,4 linkages Z) glycogen in the human liver often has more than 10,000 glucose monomers

ANSWER: Y) GLYCOGEN SIDE BRANCHES HAVE BETA 1,4 LINKAGES (Solution: beta-1,4 linkages are indigestible to humans and are found in cellulose. Glycogen has alpha 1-4 and alpha 1-6 linkages)

BONUS BIOLOGY Multiple Choice In which of the following areas of the human body is the zygomatic (read as: ZYE-go-MAT-ick) arch found: W) shoulder X) pelvis Y) head Z) knee

ANSWER: Y) HEAD

Multiple Choice Which of the following is the form of highly condensed chromatin that is not transcribable in its present form and typical of the type of chromatin visible in the light microscope when the cell is in interphase: W) Z DNA X) euchromatin Y) heterochromatin Z) histochromatin

ANSWER: Y) HETEROCHROMATIN

BIOLOGY Multiple Choice Which of the following best exemplifies secondary endosymbiosis [en-doh-sim-bee-OH-sis]? W) Human white blood cell phagocytosis [fag-uh-sy-TOH-sis] of a pathogenic bacterium X) Cyanobacterium [sy-uh-noh-bak-TEER-ee-uhm] is engulfed by a heterotrophic eukaryote [het-er-uh-TROF-ik yoo-KAR-ee-oht] Y) A heterotrophic eukaryote engulfs a green algal eukaryote Z) Receptor mediated endocytosis [en-doh-sy-TOH-sis]

ANSWER: Y) HETEROTROPHIC EUKARYOTE ENGULFS A GREEN ALGAL EUKARYOTE

Multiple Choice: What is the most appropriate term for the genetic character of an organism having a pair of identical alleles for a certain trait: W) dominant X) recessive Y) homozygous Z) heterozygous

ANSWER: Y) HOMOZYGOUS

BIOLOGY Multiple Choice A type of albinism in the Hopi Native American population is caused by mutation of the OCA2 gene. If the dominant form of the gene gives a person normal pigmentation, then which of the following genotypes would result in albinism? W) Heterozygous co-dominance [het-er-uh-ZY-guhs] X) Homozygous dominant [hoh-muh-ZY-guhs] Y) Homozygous recessive Z) Heterozygous incomplete dominance

ANSWER: Y) HOMOZYGOUS RECESSIVE

BIOLOGY Multiple Choice Which of the following is TRUE: W) chimpanzees are an ancestor of Homo sapiens X) bipedal motion dominated hominoid ancestors by about 30 million years ago Y) humans and modern apes have a common ancestor Z) Neanderthals were ancestors to modern humans

ANSWER: Y) HUMANS AND MODERN APES HAVE A COMMON ANCESTOR

BIOLOGY Multiple Choice What chemicals do peroxisomes use to oxidize other compounds? W) Mono-diglyceride [dy-GLIS-uh-ryd] X) Nitrogen dioxide Y) Hydrogen peroxide Z) Dihydrogen monoxide

ANSWER: Y) HYDROGEN PEROXIDE

BIOLOGY Multiple Choice In the lipid membranes of cells, there are multipass proteins that are partly within the lipid bilayer and partly outside of the lipid bilayer. Through which of the following forces do these proteins achieve this position relative to the membrane? W) Covalent linkages to lipid molecules X) Disulfide bonds to lipid molecules Y) Hydrophobic and hydrophilic interactions Z) Physical connections to the cytosol

ANSWER: Y) HYDROPHOBIC AND HYDROPHILIC INTERACTIONS

BONUS BIOLOGY Multiple Choice Brine shrimp are able to survive in wide ranges of salinity. Which of the following phrases is NOT true regarding their osmoregulation when placed in a tank of saline? W) Osmoconformation, body fluid osmolarity is equivalent to osmolarity of saline in which they are swimming X) Osmoregulation, body fluid osmolarity is kept at an osmolarity different from the osmolarity of saline in which they are swimming Y) Hyper-osmoregulation, body fluid osmolarity is kept higher than osmolarity of saline in which they are swimming Z) Hypo-osmoregulation, body fluid osmolarity is kept lower than osmolarity of saline in which they are swimming

ANSWER: Y) HYPER-OSMOREGULATION, BODY FLUID OSMOLARITY IS KEPT HIGHER THAN OSMOLARITY OF SALINE IN WHICH THEY ARE SWIMMING

BIOLOGY Which of the following is an example of a negative chemotactic response in bacteria? W) Marine bacteria that use carbon dioxide as a nutrient are able to sense the concentration and move toward more highly-concentrated areas X) Bacteria being viewed under a microscope move away from the light Y) In a petri dish, E. coli will move away from concentrated areas of acetate Z) Photosynthetic bacteria will move towards the light in a gradient

ANSWER: Y) IN A PETRI DISH, E. COLI WILL MOVE AWAY FROM CONCENTRATED AREAS OF ACETATE

BONUS- Multiple Choice Mendel succeeded in forming the basis of modern hereditary theory in part because of his training W) as a printer of rare and scientific documents X) in calligraphy of scientific manuscripts for the Royal Austrian Science Society Y) in probability and mathematics Z) as a scientific apostolic nuncio

ANSWER: Y) IN PROBABILITY AND MATHEMATICS

[bonus] Multiple Choice The conversion of pyruvate to Acetyl Co-A occurs: W) in the cytosol X) in the ER Y) in the mitochondrion Z) in the lysosome

ANSWER: Y) IN THE MITOCHONDRION

BIOLOGY Multiple Choice If you have a white horse mate with a white horse that has many spots called leopard spots and half of their offspring have a little bit of leopard spotting, which of the following is your likely conclusion about the leopard-spotting allele? W) Co-dominant X) Low penetrance Y) Incompletely dominant Z) Low expressivity

ANSWER: Y) INCOMPLETELY DOMINANT

BIOLOGY Multiple Choice Which of the following is NOT a point mutation? W) Missense X) Transversion Y) Inversion Z) Transition

ANSWER: Y) INVERSION

BIOLOGY Multiple Choice Which of the following molecules are NOT covalently attached to proteins as post-translational modifications? W) Ubiquitins X) Phosphates Y) Ions such as Ca⁺² and Mg⁺² Z) Carbohydrate groups

ANSWER: Y) IONS SUCH AS Ca⁺² AND Mg⁺²

BIOLOGY Multiple Choice Which of the following BEST explains how cyclic AMP can cause so many different intracellular responses in a mammalian cell: W) it is produced in large quantities X) it migrates to the DNA where it binds to a variety of histones Y) it activates a wide variety of specific protein kinases Z) it prevents the activation of other hormonal signals

ANSWER: Y) IT ACTIVATES A WIDE VARIETY OF SPECIFIC PROTEIN KINASES

BONUS BIOLOGY Multiple Choice Which of the following is NOT true regarding human myoglobin [my-uh-GLOH-bin]? W) It is a monomer X) It exhibits higher affinity for oxygen than hemoglobin Y) It exhibits a sigmoidal [SIG-moid-uhl] saturation curve Z) It is primarily an oxygen storage protein

ANSWER: Y) IT EXHIBITS A SIGMOIDAL SATURATION CURVE

BIOLOGY Multiple Choice Which of the following statements about the lone star tick is true? W) It only lives in Texas X) It transmits Lyme disease Y) It feeds on humans, dogs, and cats Z) It is relatively passive

ANSWER: Y) IT FEEDS ON HUMANS, DOGS, AND CATS

Multiple Choice During embryogenesis, which of the following is MOST accurate concerning the blastopore: W) it does not exist in protostomes X) it originates before the blastula stage Y) it forms the anus in deuterostomes (read as: DOO-ter-oh-stomes) Z) it is the hollow center of the gastrula

ANSWER: Y) IT FORMS THE ANUS IN DEUTEROSTOMES

BIOLOGY Multiple Choice Which of the following best describes blood? W) It is a specialized type of cell found in blood vessels X) It is a type of cell formed in the bone marrow Y) It is a kind of connective tissue in which red and white blood cells are suspended in plasma Z) It is a kind of epithelial tissue [ep-uh-THEE-lee-al]

ANSWER: Y) IT IS A KIND OF CONNECTIVE TISSUE IN WHICH RED AND WHITE BLOOD CELLS ARE SUSPENDED IN PLASMA

BIOLOGY Multiple Choice Which of the following BEST describes the syndrome that manifests itself in starving children as swollen distended abdomens, spindly arms, and edema in the face, hands, and feet resulting from the catabolism of muscle and blood proteins? W) Malnutrition primus X) Beri Beri Y) Kwashiorkor (read as; kwah-shee-OHR-kohr) Z) Emaciation

ANSWER: Y) KWASHIORKOR

BIOLOGY Multiple Choice Which of the following processes is NOT correctly matched with the role it plays in the global nitrogen cycle? W) Nitrogen fixation with the Haber-Bosch process X) Nitrogen-fixing bacteria with ammonia produced from nitrogen gas Y) Legume root nodules with nitrate produced from ammonia Z) Denitrification with nitrogen gas produced from nitrates

ANSWER: Y) LEGUME ROOT NODULES WITH NITRATE PRODUCED FROM AMMONIA

Biology - Multiple Choice Which of the following amino acids is non-polar? W) Glutamine [gloo-tuh-meen] X) Histidine Y) Leucine [loo-seen] Z) Lysine

ANSWER: Y) LEUCINE

BONUS Biology - Multiple Choice Which of the following statements is true regarding fungal mutualism? W) Most endophytes are basidiomycetes [buh-s id-ee-o h-mye-seets] X) Many species of ants culture fungi inside of their bodies Y) Lichens are capable of nitrogen fixation Z) Plants can generally grow normally without endophytes

ANSWER: Y) LICHENS ARE CAPABLE OF NITROGEN FIXATION

Biology - Multiple Choice Which of the following genes would initially establish the axes of the Drosophila body? W) Homeotic genes X) Hox genes Y) Maternal effect genes Z) Embryonic lethal genes

ANSWER: Y) MATERNAL EFFECT GENES

BIOLOGY Multiple Choice Which of the following senses is most important to the behavior of most spiders? W) Audition X) Olfaction Y) Mechanoreception Z) Polarized light vision

ANSWER: Y) MECHANORECEPTION

BONUS Biology - Multiple Choice Which of the following amino acids is encoded by a single codon? W) Leucine [LOO-seen] X) Glycine [GLYE-seen] Y) Methionine [meth-IOH-neen] Z) Phenylalanine [Fennel-AL-ahneen]

ANSWER: Y) METHIONINE

BONUS BIOLOGY Multiple Choice Which of the following is NOT true regarding ribosomes: W) they are about 20 nanometers in diameter X) eukaryotic ribosomes are slightly larger than prokaryotic ribosomes Y) mitochondrial ribosomes have 40S (read as: forty S) small subunits Z) they are found in chloroplasts

ANSWER: Y) MITOCHONDRIAL RIBOSOMES HAVE 40S SMALL SUBUNITS

BIOLOGY Multiple Choice Which of the following BEST describes the function of the golgi (read as: GOAL-gee) complex: W) metabolism of carbohydrates X) production of peroxides Y) modification and packaging of proteins for exocytosis Z) storage of lipids

ANSWER: Y) MODIFICATION AND PACKAGING OF PROTEINS FOR EXOCYTOSIS

Biology Multiple Choice: Which of the following BEST describes the function of the golgi (read as: GOAL-gee) complex: W) metabolism of carbohydrates X) production of peroxides Y) modification and packaging of proteins for exocytosis Z) storage of lipids

ANSWER: Y) MODIFICATION AND PACKAGING OF PROTEINS FOR EXOCYTOSIS

BONUS Biology - Multiple Choice Which of the following is not true regarding the development of multicellular organisms? W) Homeotic genes control pattern formation in the late embryo X) Proto-oncogenes are often involved in regulation of the cell cycle Y) Mutations in proto-oncogenes prevent cancer Z) Mutations in tumor suppressor genes often result in cancer

ANSWER: Y) MUTATIONS IN PROTO-ONCOGENES PREVENT CANCER

Multiple Choice TSH causes the thyroid to produce more T3. When T3 rises, it flows through the blood to the brain to decrease the release of TSH. Which of the following is the MOST accurate and common term for this physiological phenomenon: W) metabolic reaction X) hormonal balance Y) negative feedback Z) endocrine action

ANSWER: Y) NEGATIVE FEEDBACK

BONUS BIOLOGY Multiple Choice Which of the following is a carnivorous [kahr-NIV-or-uhs] plant? W) Mushroom X) Nightshade Y) Nepenthes [ni-PEN-theez] Z) Datura [duh-TOOR-uh]

ANSWER: Y) NEPENTHES

BONUS BIOLOGY Multiple Choice Which of the following are NOT correctly matched to their function? W) Denitrifying bacteria: convert nitrates to gaseous nitrogen X) Nitrogen fixing bacteria: convert gaseous nitrogen to ammonia Y) Nitrosofying bacteria: convert ammonia to nitrates Z) Nitrifying bacteria: convert nitrites to nitrates

ANSWER: Y) NITROSOFYING BACTERIA: CONVERT AMMONIA TO NITRATES

BIOLOGY Multiple Choice Aposematic coloration allows an organism to signal which of the following? W) Readiness to mate X) Territoriality Y) Noxious taste Z) Darwinian fitness

ANSWER: Y) NOXIOUS TASTE

BIOLOGY Multiple Choice Which of the following best describes actin treadmilling? W) New actin polymers attach to the minus end of the actin filament X) The actin filament remains static with no addition or loss of subunits Y) One end of an actin filament has a net addition of actin polymers while the other end has a net loss of actin polymers Z) One end of the actin filament adds and loses polymers at an equal rate

ANSWER: Y) ONE END OF THE ACTIN FILAMENT HAS A NET ADDITION OF ACTIN POLYMERS WHILE THE OTHER END HAS A NET LOSS OF ACTIN POLYMERS

BONUS Biology - Multiple Choice In which of the following organisms would you most likely be able to find contractile vacuoles? W) Cytomegalovirus X) E. coli Y) Paramecium Z) Xenopus [zee-n oh-p us]

ANSWER: Y) PARAMECIUM

BIOLOGY Multiple Choice Which of the following is NOT true of globular proteins in mammals: W) most are soluble in water X) some serve as transport molecules in the plasma Y) plasma enzymes typically have an optimal operating pH of about 6.8 to 7.0 Z) most protein hormones are globular

ANSWER: Y) PLASMA ENZYMES TYPICALLY HAVE AN OPTIMAL OPERATING PH OF ABOUT 6.8 TO 7.0

Multiple Choice Which of the following are MOST often used to introduce foreign genes into mammalian genomes: W) sliceosomes (read as: sly-see-oh-somz) X) zinc fingers Y) plasmids Z) telomeres

ANSWER: Y) PLASMIDS

BIOLOGY Multiple Choice Which of the following describes the bonds found in one molecule of water? W) Hydrogen X) Nonpolar covalent Y) Polar covalent Z) Ionic

ANSWER: Y) POLAR COVALENT

BONUS 7) BIOLOGY Multiple Choice Which of the following is most likely to cause sympatric speciation? W) Mountain building X) Habitat fragmentation Y) Polyploidy [pol-ee-PLOI-dy] Z) Urbanization

ANSWER: Y) POLYPLOIDY

BONUS BIOLOGY Multiple Choice Which of the following best describes the type of RNA that codes for a complete gene on the sense strand of a DNA molecule? W) Ribosomal RNA X) Mitochondrial RNA Y) Pre-messenger RNA Z) Transfer RNA

ANSWER: Y) PRE-MESSENGER RNA

BONUS BIOLOGY Multiple Choice Quorum [KWOHR-uhm] sensing in bacteria occurs when bacteria "sense" other bacteria. Which of the following triggers this phenomenon to occur? W) Number of bacteria X) Density of bacteria Y) Production and detection of autoinducers Z) Bacterial fission

ANSWER: Y) PRODUCTION AND DETECTION OF AUTOINDUCERS

BIOLOGY Multiple Choice In active meiosis, new genetic combinations can arise in two different ways. Which of the following is the first stage in which there could be genetic recombination? W) Anaphase I X) Anaphase II Y) Prophase I Z) Prophase II

ANSWER: Y) PROPHASE I

BIOLOGY Multiple Choice If you are doing a western blot, what kind of macromolecule are you trying to detect? W) DNA X) RNA Y) Protein Z) Carbohydrate

ANSWER: Y) PROTEIN

Multiple Choice Which of the following is NOT true: W) comb jellies or ctenophores (read as: ten-OH-fours) have radial symmetry X) lobsters have bilateral symmetry Y) protostomes have no true coeloms (read as: SEE-loams) Z) flatworms are acoelomates (read as: ah-SEA-low-mates)

ANSWER: Y) PROTOSTOMES HAVE NO TRUE COELOMS

BONUS BIOLOGY Multiple Choice Koch's (read as: Coke's) Postulates have to do mainly with which of the following: W) culturing of viruses X) causative agents of viral diseases Y) proving what bacterial agents cause what diseases Z) how to prove that a certain gene causes a certain disease

ANSWER: Y) PROVING WHAT BACTERIAL AGENTS CAUSE WHAT DISEASES

Biology - Multiple Choice Which of the following vitamins is not fat soluble? W) Tocopherol [toh-c of-uh-r ol] X) Phylloquinone [fy-loh-k wih-nohn] Y) Pyridoxine [pie-rih-dox-een] Z) Calciferol [cal-s ihf-u h-r ol]

ANSWER: Y) PYRIDOXINE

BONUS BIOLOGY Multiple Choice Which of the following molecules does NOT act as an electron shuttle? W) NAD+ X) Cytochrome C Y) Pyruvate [py-ROO-vayt] Z) Coenzyme Q

ANSWER: Y) PYRUVATE

Biology - Multiple Choice Which of the following is NOT a characteristic of chordates? W) Notochord X) Pharyngeal slits [fahr-in-jee-u hl] Y) Radial symmetry Z) Post-anal tail

ANSWER: Y) RADIAL SYMMETRY

BONUS BIOLOGY Multiple Choice Which of the following is NOT an energy source for active transport across a membrane? W) Electrochemical gradient of a different molecule X) Light Y) Ras proteins Z) ATP

ANSWER: Y) RAS PROTEINS

BIOLOGY Multiple Choice Which of the following best describes the repressor protein in the lac (read as: LACK) operon: W) uncompetitive inhibitor X) structural protein Y) regulatory protein Z) transcriptional factor

ANSWER: Y) REGULATORY PROTEIN

BONUS Biology - Multiple Choice The mechanisms involved in aligning homologous chromosomes in meiosis are also useful for which of the following? W) Repairing nicks X) Repairing single-stranded breaks Y) Repairing double-stranded breaks Z) Repairing point mutations

ANSWER: Y) REPAIRING DOUBLE STRANDED BREAKS

BONUS Biology - Multiple Choice Which of the following enzymes would be required for the formation of cDNA, a genetic transcript from eukaryotic cells that has had the introns removed? W) Excinuclease [ex-ih-noo-clee-ayse] X) AP Endonuclease Y) Reverse transcriptase Z) DNase I

ANSWER: Y) REVERSE TRANSCRIPTASE

[bonus] Multiple Choice Which of the following functions primarily by cutting RNA strands at specific sites: W) micro RNA X) episome Y) ribozyme Z) ligase

ANSWER: Y) RIBOZYME

BIOLOGY Multiple Choice Which of the following anatomical adjectives is NOT correctly described? W) Celiac-abdominal X) Anterior-front end Y) Rostral-near the tail Z) Pedal-relating to the foot

ANSWER: Y) ROSTRAL-NEAR THE TAIL

Biology - Multiple Choice Which of the following techniques would one use to separate proteins based on size? W) Polymerase chain reaction X) E. coli transduction Y) SDS-polyacrylamide gel electrophoresis [el-e k-tr oh-fer-ee-s ihs] Z) Enzyme-linked immunoprecipitation

ANSWER: Y) SDS-POLYACRYLAMIDE GEL ELECTROPHORESIS

BONUS BIOLOGY Multiple Choice The margin of an ash tree leaf would most generally be called: W) divided X) entire Y) serrate Z) lacerate

ANSWER: Y) SERRATE

Biology - Multiple Choice What human body cells contain H bands, Z disks and I bands? W) Neurons X) Adipocytes [u h-d ip-o h-s ights] Y) Skeletal muscle Z) Osteocytes [os-t ee-o h-sights

ANSWER: Y) SKELETAL MUSCLE

Multiple Choice: The lambdoid suture is found in which of the following parts of the human body: W) scapula X) pelvis Y) skull Z) knee

ANSWER: Y) SKULL

BONUS BIOLOGY Multiple Choice Which of the following would NOT be a part of protein synthesis in prokaryotes? W) Translation and transcription occurring simultaneously X) Ribosomes Y) Spliceosomes Z) RNA polymerases

ANSWER: Y) SPLICEOSOMES

Biology - Multiple Choice Helicobacter pylori [HEEL-ik-oh-bak-tur pie-LOR-ee] is associated with which of the following disorders? W) SARS X) Tuberculosis Y) Stomach ulcers Z) Anemia

ANSWER: Y) STOMACH ULCERS

BIOLOGY Multiple Choice Which of the following is NOT an evolutionary adaptation of plants to arid climates? W) Leaves have a waxy coating X) Carbon incorporation and the Calvin cycle are separated Y) Stomata are closed at night Z) Leaves are absent

ANSWER: Y) STOMATA ARE CLOSED AT NIGHT

BONUS BIOLOGY Multiple Choice Which of the following terms indicates centromere placement at the end of the chromosome? W) Metacentric X) Acrocentric Y) Telocentric Z) Submetacentric

ANSWER: Y) TELOCENTRIC

BIOLOGY Multiple Choice Which of the following is the highest level of structural organization in myoglobin: W) primary X) secondary Y) tertiary Z) quaternary

ANSWER: Y) TERTIARY

Biology - Multiple Choice Which of the following hormones is derived from cholesterol? W) Substance P X) Indoleacetic acid Y) Testosterone Z) Prolactin

ANSWER: Y) TESTOSTERONE

BONUS BIOLOGY Multiple Choice Which of the following BEST explains why a single base substitution in a gene-coding region for amino acids will sometimes lead to no change in amino acid sequence: W) all nucleotides are subject to the same mutation rate X) some nucleotide bases are covalently joined by stronger bonds Y) the code is degenerate with more than one triplet coding for the same amino acid Z) some amino acids act in the same fashion in determining the tertiary structure of a protein

ANSWER: Y) THE CODE IS DEGENERATE WITH MORE THAN ONE TRIPLET CODING FOR THE SAME AMINO ACID

BONUS BIOLOGY Multiple Choice Which of the following statements about the Calvin cycle is NOT true? W) It is an important step during photosynthesis X) During the Calvin cycle, the simple inorganic molecule of carbon dioxide is used to make a complex organic molecule Y) The compound that begins the Calvin cycle is glyceraldehyde-3-phosphate Z) The energy needed for the Calvin cycle is provided by the light reaction

ANSWER: Y) THE COMPOUND THAT BEGINS THE CALVIN CYCLE IS GLYCERALDEHYDE 3-PHOSPHATE

Multiple Choice During the electron transport system, a store of potential energy is created that will be converted into high energy phosphates. The potential energy is best described as: W) ADP X) ATP Y) the hydrogen ion gradient Z) the sodium differential

ANSWER: Y) THE HYDROGEN ION GRADIENT

BONUS Biology - Multiple Choice Which of the following is not true regarding mitosis? W) Sister chromatids are held together by cohesion [coh-ee-sin]complexes X) M-Cdk drives entry of the cell into mitosis Y) The mitotic spindle is built from actin filaments Z) The Golgi apparatus becomes less prominent during metaphase

ANSWER: Y) THE MITOTIC SPINDLE IS BUILT FROM ACTIN FILAMENTS

BONUS BIOLOGY Multiple Choice Some virus species can infect an individual host more than once because they change the shape of molecules exposed on the surface of their virions [VY-reeons] so that they are no longer recognized by the host's immunological memory. The influenza virus can make sudden and drastic changes in these molecules due to a phenomenon called reassortment. Which of the following contributes to the process of reassortment? W) Widely varying strains of influenza X) The diversity of animals infected by influenza Y) The segmented genome found in influenza Z) The lack of effective antivirals against influenza until recently

ANSWER: Y) THE SEGMENTED GENOME FOUND IN INFLUENZA

BONUS BIOLOGY Multiple Choice You're investigating guinea pig coat genetics, and you've found an interesting new phenotype: spiral guinea pig fur. If you take a purebred wavy-hair guinea pig and cross it with a purebred kinked-hair guinea pig, you get all spiral-hair progeny. If you mate spiral-hair guinea pigs with each other, the progeny have a phenotypic ratio of 9 spiral: 3 wavy: 3 kinked: 1 straight. Which of the following is most likely to be true given your experimental results? W) There are two different genes that result in the wavy and kinked phenotypes, and there is an epistatic interaction between them X) There are two different genes that result in the wavy and kinked phenotypes, and they are linked Y) There are two different genes that result in the wavy and kinked phenotypes, and they are independently assorting Z) There is only one gene with a number of different alleles

ANSWER: Y) THERE ARE TWO DIFFERENT GENES THAT RESULT IN THE WAVY AND KINKED PHENOTYPES, AND THEY ARE INDEPENDENTLY ASSORTING

BONUS BIOLOGY Multiple Choice Which of the following is true of the twenty common amino acids? W) They contain a primary amine and secondary alcohol X) They contain a primary amide and a primary alcohol Y) They contain a primary amine and a primary carboxyl group Z) They contain a secondary amine and a primary carboxyl group

ANSWER: Y) THEY CONTAIN A PRIMARY AMINE AND A PRIMARY CARBOXYL GROUP

BONUS BIOLOGY Multiple Choice Muscle fibers can be classified as slow, intermediate, or fast. Which of the following is NOT true regarding fast muscle fibers? W) They are found in higher relative abundance in world class sprinters versus world class marathoners X) They fatigue most rapidly Y) They have the highest mitochondrial density Z) The primary pathway for ATP production is fermentation

ANSWER: Y) THEY HAVE THE HIGHEST MITOCHONDRIAL DENSITY

BONUS Biology - Multiple Choice Which of the following is true regarding the neural crest cells in the amphibian embryo? W) They induce formation of the notochord X) They develop into the main sections of the brain Y) They produce cells that migrate to form teeth and skull bones in the embryo Z) They roll up and form the neural tube

ANSWER: Y) THEY PRODUCE CELLS THAT MIGRATE TO FORM TEETH AND SKULL BONES IN THE EMBRYO

Multiple Choice Which of the following is NOT true about vaccinations: W) they were first developed against viral infections such as small pox X) they allow the immune system to respond more quickly and effectively on exposure to the disease causing agent Y) they will typically protect the person from infection within 24 hours after vaccination Z) they often require adjuvants which are nonspecific irritants for best results

ANSWER: Y) THEY WILL TYPICALLY PROTECT THE PERSON FROM INFECTION WITHIN 24 HOURS AFTER VACCINATION

BIOLOGY Multiple Choice When two pyrimidine bases are adjacent on a DNA strand, exposure to UV light will commonly lead to formation of a cyclobutane ring between them, resulting in a bulky lesion in the DNA. The normal replicative polymerases cannot use this as a template, and the lesion often results in mutation if it cannot be repaired. Which of the following is an example of this lesion formed by UV exposure of neighboring pyrimidines? W) Transversion X) Cytosine deamination Y) Thymine dimer Z) Intercalation

ANSWER: Y) THYMINE DIMER

[bonus]Multiple Choice Which of the following is NOT properly matched with its function or operation: W) conjugation and transfer of genetic material between bacteria X) F plasmids and reversible incorporation into bacterial genome Y) transposons and plasmids interchanging genes Z) R plasmids and antibiotic resistance

ANSWER: Y) TRANSPOSONS AND PLASMIDS INTERCHANGING GENES

Multiple Choice Which of the following tissues is MOST directly responsible for the increased diameter of a tree trunk from year to year: W) apical meristem X) pericycle Y) vascular cambium Z) pith

ANSWER: Y) VASCULAR CAMBIUM

(General Science) [Bonus] Multiple Choice Which of the following is the BEST description of atherosclerosis: W) inflammation of the brain cortical tissues X) paralysis of extremities and loss of sensation Y) walls of blood vessels become thicker and less elastic Z) loss of ability to hear certain sounds

ANSWER: Y) WALLS OF BLOOD VESSELS BECOME THICKER AND LESS ELASTIC

Multiple Choice During photosynthesis, from what substance does the oxygen that is liberated originate: W) glucose X) air Y) water Z) carbon dioxide

ANSWER: Y) WATER

BIOLOGY Multiple Choice Which of the following animals would have the highest levels of myoglobin [my-uh-GLOH-bin] in their muscle? W) American pronghorn X) Human being Y) Weddell seal Z) Kangaroo rat

ANSWER: Y) WEDDELL SEAL

Multiple Choice Which of the following is closest to the maximum ATP molecules generated from the complete oxidation of 1 mole of glucose in a typical eukaryotic cell: W) 8 moles X) 16 moles Y) 24 moles Z) 38 moles

ANSWER: Z) 38 MOLES

Biology - Multiple Choice MRSA is an infection by which of the following? W) A rod shaped bacteria X) A rigid helical bacteria Y) A non-rigid helical bacteria Z) A spherical bacteria

ANSWER: Z) A SPHERICAL BACTERIA

Biology - Multiple Choice The Edinger-Westphal nucleus consists of parasympathetic neurons that control pupillary reflexes. Which of the following neurotransmitters do Edinger-Westphal neurons most likely release? W) Epinephrine X) Norepinephrine Y) Glutamine Z) Acetylcholine

ANSWER: Z) ACETYLCHOLINE

Biology - Multiple Choice The Edinger-Westphal nucleus consists of parasympathetic neurons that control pupillary reflexes. Which of the following neurotransmitters do Edinger-Westphal neurons most likely release? W) Epinephrine X) Norepinephrine Y) Glutamine Z) Acetylcholine

ANSWER: Z) ACETYLCHOLINE

[Bonus] Multiple Choice Which of the following is NOT true of nucleated mammalian cells: W) mast cells contain large amounts of histamine X) the golgi apparatus processes proteins for exocytosis Y) microtubules are components in cilia Z) all the DNA in a nucleated cell is in the nucleus

ANSWER: Z) ALL THE DNA IN A HUMAN NUCLEATED CELL IS IN THE NUCLEUS (Solution: mitochondrial DNA is present)

Biology - Multiple Choice Which of the following characteristics of plants is absent in charophyte algae? W) Presence of chlorophyll b X) Cellulose in the cell walls Y) Sexual reproduction Z) Alternation of multicellular generations

ANSWER: Z) ALTERNATION OF MULTICELLULAR GENERATIONS

BIOLOGY Multiple Choice In plants, which of the following groups of cells is responsible for the growth of shoots? W) Secondary meristem X) Parenchyma (read as: pah-REHN-ki-mah) cells Y) Guard cells Z) Apical (read as: AY-pi-kahl) meristem

ANSWER: Z) APICAL MERISTEM

BIOLOGY Multiple Choice Bacteria use which of the following mechanisms during quorum [KWOHR-uhm] sensing? W) Hetero-induction X) Apoptosis Y) Density independent gene regulation Z) Autoinducers

ANSWER: Z) AUTOINDUCERS

Multiple Choice: Which of the following human cell types is MOST directly responsible for the secretion of antibodies: W) monocytes X) CD4+ cells Y) T activator lymphocytes Z) B cells

ANSWER: Z) B CELLS

Multiple Choice In the famous experiment by Hershey and Chase that showed DNA was the genetic material, what did they use to transmit the DNA into E. coli: W) other E. coli X) electric shocks Y ) a blender Z) bacteriophage

ANSWER: Z) BACTERIOPHAGE

BONUS Biology - Multiple Choice Vitamin K is important to which of the following processes? W) Metabolism X) Calcium absorption Y) Night vision Z) Blood clotting

ANSWER: Z) BLOOD CLOTTING

Multiple Choice Which of the following is LEAST accurate regarding Avian migratory patterns: W) migration of birds from the tropics to the northern latitudes is typically endogenously controlled X) the migratory timing of many birds is being affected by global change Y) both large and very small birds migrate long distances Z) breeding in most birds is timed to be when fewest predators are present

ANSWER: Z) BREEDING IN MOST BIRDS IS TIMED TO BE WHEN FEWEST PREDATORS ARE PRESENT

[bonus] Multiple Choice In which of the following does the RUBISCO enzyme play a central role: W) Photosystem One X) Electron Transport Y) Photosystem Two Z) Calvin Cycle

ANSWER: Z) CALVIN CYCLE

Multiple Choice: Which of the following is a function of the pollen tube in flowering plants: W) attaches pollen to the developing anther X) allows pollen to absorb nutrients as they develop in the anther Y) enables the ovule to breach through the micropyle Z) carries the sperm nuclei to the ovule

ANSWER: Z) CARRIES THE SPERM NUCLEI TO THE OVULE

BIOLOGY Multiple Choice The basal lamina is a long thin structure that is essential for the stability of body cells. Which of the following cell classes is NOT typically underlain or surrounded by the basal lamina? W) Epithelial X) Muscle Y) Nerve Z) Cartilage

ANSWER: Z) CARTILAGE

BONUS BIOLOGY Multiple Choice The gram stain can be used to distinguish between different prokaryotes [proh-KAR-ee-ohts] based on which of the following? W) Glucose metabolism X) Flagella Y) Lactose metabolism Z) Cell wall morphology

ANSWER: Z) CELL WALL MORPHOLOGY

BIOLOGY Multiple Choice Which of the following Orders does NOT belong to the Class Insecta: W) Lepidoptera X) Diptera Y) Isoptera Z) Chelicerata

ANSWER: Z) CHELICERATA

BONUS Biology - Multiple Choice Which of the following is not true regarding arthropods? W) Arthropods have an open circulatory system X) Molting occurs because arthropods cannot grow without shedding their exoskeleton Y) Book lungs are found in arachnids Z) Cheliceriformes [kuh-liss-er-uh-forms] are characterized by 5 pairs of legs and two pairs of antennae

ANSWER: Z) CHELICERIFORMES ARE CHARACTERIZED BY 5 PAIRS OF LEGS AND TWO PAIRS OF ANTENNAE

BONUS Biology - Multiple Choice A deficiency in which of the following vitamins causes pernicious anemia? W) Thiamine [thigh-u h-meen] X) Retinol Y) Ascorbic Acid Z) Cobalamin [co-bahl-u h-min]

ANSWER: Z) COBALAMIN

Biology - Multiple Choice Which of the following reactions does NOT require biotin? W) Conversion of pyruvate to oxaloacetate [ox-aa-loh-ASS-ah-tate] X) Conversion of acetyl-CoA to malonyl [MAL-uh-nihl]-CoA Y) Conversion of propionyl [PRO-pee-on-il]-CoA to methylmalonyl[meh-thil-MAL-uh-nil]- CoA Z) Conversion of oxaloacetate [ox-aa-loh-ASS-ah-tate] to succinate

ANSWER: Z) CONVERSION OF OXALOACETATE TO SUCCINATE

Multiple Choice In Barbara McClintock's famous experiments revealing the existence of transposable genes, she primarily used which of the following as her experimental organism: W) drosophila X) E. coli Y) lambda phage Z) corn plants

ANSWER: Z) CORN PLANTS

Biology - Multiple Choice Which of the following does not play a role in protein digestion? W) Trypsin X) Pepsin Y) Carboxypeptidase [car-b ox-ee-p ep-tih-d ays] Z) Dextrinase [dex-trih-nays]

ANSWER: Z) DEXTRINASE

BONUS BIOLOGY Multiple Choice Which of the following is NOT true of the most common form of DNA: W) it is a right-handed helix X) it has 3.4 nanometers between each successive nucleotide base of the same strand Y) the strands are considered anti-parallel Z) each strand has a 3-prime phosphate and a 5-prime hydroxyl at opposite ends

ANSWER: Z) EACH STRAND HAS A 3-PRIME PHOSPHATE AND A 5-PRIME HYDROXYL AT OPPOSITE ENDS

Multiple Choice The genetic material of different viruses can be: W) only double-stranded DNA X) only single-stranded DNA or double-stranded DNA Y) only double-stranded DNA or double-stranded RNA Z) either single-stranded DNA and RNA or double-stranded DNA and RNA

ANSWER: Z) EITHER SINGLE-STRANDED DNA AND RNA OR DOUBLE-STRANDED DNA AND RNA

(General Science) Multiple Choice Which of the following is NOT true: W) nanometer-thick films that roll themselves into tubes and other useful shapes have been recently discovered X) counting annual growth lines in dinosaur bones can be used to estimate their age at death Y) amphibian population declines and extinctions are global and rapid Z) epiphytic plants such as many orchids are considered parasitic plants

ANSWER: Z) EPIPHYTIC PLANTS SUCH AS MANY ORCHIDS ARE CONSIDERED PARASITIC PLANTS

BONUS Biology - Multiple Choice Which of the following cell types is responsible for generating the extracellular matrix? W) Macrophages X) Monocytes Y) Megakaryocytes Z) Fibroblasts

ANSWER: Z) FIBROBLASTS

Biology - Multiple Choice Which of the following types of animal viruses is responsible for hepatitis C? W) Papovirus X) Adenovirus[add-en-o -virus] Y) Retrovirus Z) Flavivirus [flay-vee-virus]

ANSWER: Z) FLAVIVIRUS

Biology - Multiple Choice Which of the following lengths of DNA would be least likely to be unwound by a passive helicase? W) ATATAT X) AAATTT Y) TTTGGG Z) GCGCGC

ANSWER: Z) GCGCGC

BONUS BIOLOGY Multiple Choice In the human body, copper plays a role in which of the following processes? W) Muscle contraction X) pH balance Y) Protein synthesis Z) Hemoglobin synthesis

ANSWER: Z) HEMOGLOBIN SYNTHESIS

(General Science) Multiple Choice: The process in which fat globules in milk are broken up into smaller, more evenly dispersed particles is BEST described as: W) lactation X) pasteurization Y) fermentation Z) homogenization

ANSWER: Z) HOMOGENIZATION

BONUS BIOLOGY Multiple Choice Symmetry in animal taxonomy describes which of the following? W) How the animals are related X) Similarities in anatomy of different species Y) Patterns of animal behavior Z) How the parts of an animal are arranged around a point or an axis

ANSWER: Z) HOW THE PARTS OF AN ANIMAL ARE ARRANGED AROUND A POINT OR AN AXIS

BONUS BIOLOGY Multiple Choice Which of the following diseases is NOT caused by bacteria: W) strep throat X) toxic shock syndrome Y) scarlet fever Z) influenza

ANSWER: Z) INFLUENZA

BONUS BIOLOGY Multiple Choice Which of the following is NOT true regarding human hemoglobin? W) It is a tetramer X) It exhibits binding cooperativity Y) It exhibits a sigmoidal saturation curve Z) It exhibits higher affinity for oxygen than myoglobin

ANSWER: Z) IT EXHIBITS HIGHER AFFINITY FOR OXYGEN THAN MYOGLOBIN

BONUS BIOLOGY Multiple Choice Which of the following best describes adaptive immunity? W) It includes external barriers such as secretions from mucous membranes X) It is found in both vertebrates and invertebrates Y) It involves the lymphatic system Z) It includes an interaction of pathogens, antibodies, and lymphocytes [LIM-fuh-syts]

ANSWER: Z) IT INCLUDES AN INTERACTION OF PATHOGENS, ANTIBODIES, AND LYMPHOCYTES

(General Science) [Bonus] Multiple Choice Which of the following is NOT true: W) the epilimnion is the upper warm layer of water of a lake in summer X) in cameras, the larger the f-number, the greater the depth of field Y) crab grass is better adapted to hot dry weather than Kentucky blue grass Z) Listeria is a waterborne parasite

ANSWER: Z) LISTERIA IS A WATERBORNE PARASITE (Solution: it is food-borne)

BIOLOGY Multiple Choice What component of the nephron establishes and maintains a salt gradient used to concentrate urine? W) Bowman's capsule X) Proximal convoluted tubule Y) Distal convoluted tubule Z) Loop of Henle

ANSWER: Z) LOOP OF HENLE

BONUS BIOLOGY Multiple Choice What component of the nephron establishes and maintains a salt gradient used to concentrate urine? W) Bowman's capsule X) Proximal convoluted tubule Y) Distal convoluted tubule Z) Loop of Henle

ANSWER: Z) LOOP OF HENLE

BONUS BIOLOGY Multiple Choice Which of the following is an example of an ectoparasite [ek-tohPAR-uh-syt]? W) Tapeworm X) Hookworm Y) Nematode Z) Louse

ANSWER: Z) LOUSE

(General Science) [bonus] Multiple Choice Shock is often a life-threatening health condition that is typically and most directly caused by: W) low blood pH X) loss of brain function Y) extremely high cardiac output Z) low blood pressure

ANSWER: Z) LOW BLOOD PRESSURE

BONUS BIOLOGY Multiple Choice Enzyme A has a high Km value and low Vmax value relative to Enzyme B. This means Enzyme A, relative to Enzyme B has which of the following? W) Higher substrate affinity and a lower reaction rate X) Lower substrate affinity and a higher reaction rate Y) Higher substrate affinity and higher reaction rate Z) Lower substrate affinity and lower reaction rate

ANSWER: Z) LOWER SUBSTRATE AFFINITY AND LOWER REACTION RATE

BONUS BIOLOGY Multiple Choice Stomata will open in response to internal and external cues. Which of the following is NOT one of these cues? W) Uptake of potassium ions X) Decrease of carbon dioxide in leaf air spaces due to photosynthesis in the mesophyll Y) Changes in light levels Z) Mesophyll production of abscisic acid in response to water deficiency

ANSWER: Z) MESOPHYLL PRODUCTION OF ABSCISIC ACID IN RESPONSE TO WATER DEFICIENCY

BONUS5) Biology - Multiple Choice Which of the following structures does dynein [dye-n een] travel along? W) Actin filaments X) Keratin filaments Y) Myosin filaments Z) Microtubules

ANSWER: Z) MICROTUBULES

[BONUS] Multiple Choice: Which of the following does NOT belong to the Arthropoda: W) Arachnida X) Diplopoda Y) Chilopoda Z) Nematoda

ANSWER: Z) NEMATODA

BIOLOGY Multiple Choice Which of the following organic groups is NEVER bound to the central carbon atom of an amino acid? W) Carboxyl X) Methyl Y) Amine Z) Nitrile

ANSWER: Z) NITRILE

BONUS BIOLOGY Multiple Choice Plasmids are autonomously replicating circles of DNA containing one or a few genes. How many plasmids may a single bacterium carry? W) 0 X) 1 Y) 2 Z) No fixed limit

ANSWER: Z) NO FIXED LIMIT

Multiple Choice According to the 3 domain system of classification with the 3 domains being, Archaea, Bacteria and Eukarya, which of the following would be LEAST likely to belong to the domain Bacteria: W) chlamydia X) spirochetes (read as: spy-row-keets) Y) gram positive bacteria Z) non-nucleated thermophiles

ANSWER: Z) NON-NUCLEATED THERMOPHILES

BIOLOGY Multiple Choice Which of the following monomers CANNOT be converted into an intermediate that can be used for cellular respiration? W) Fatty acids X) Glycerol Y) Amino acids Z) Nucleic acids

ANSWER: Z) NUCLEIC ACIDS

BONUS BIOLOGY Multiple Choice Which of the following is NOT characteristic of eudicots [yoo-DYkots]? W) Flower parts in multiples of 4 or 5 X) Taproot Y) Two cotyledons [kot-l-EED-ns] Z) Pollen granules with one opening

ANSWER: Z) POLLEN GRANULES WITH ONE OPENING

BIOLOGY Multiple Choice From which of the following is the cell wall of plants mostly composed: W) protein X) lipid monolayer Y) lipid bilayer Z) polysaccharide

ANSWER: Z) POLYSACCHARIDE

Biology - Multiple Choice Hybrid breakdown and lack of hybrid robustness are examples of which type of reproductive isolation? W) Mechanical isolation Y) Allopatric speciation X) Prezygotic barriers Y) Postzygotic barriers

ANSWER: Z) POSTZYGOTIC BARRIERS

BIOLOGY Multiple Choice In the lab, you engineer a gene so that it is specifically expressed in pancreatic cells. Which of the following aspects of the gene did you likely adjust? W) Intron sequences X) 5' [5 prime] UTR sequences Y) Translational start sequences Z) Promoter sequences

ANSWER: Z) PROMOTER SEQUENCES

BIOLOGY Multiple Choice With which of the following does human interstitial fluid differ in concentration the most when compared with plasma: W) sodium ions X) potassium ions Y) magnesium ions Z) proteins

ANSWER: Z) PROTEINS

Biology - Multiple Choice Which of the following cleavage patterns is characteristic of deuterostomes [doo-t uhr-o h-s tohms]? W) Spiral cleavage X) Determinate cleavage Y) Bilateral cleavage Z) Radial cleavage

ANSWER: Z) RADIAL CLEAVAGE

[Bonus] (General Science) Multiple Choice Which of the following statements is LEAST accurate: W) systemic autoimmunity often targets a person's own nucleic acids X) the protonated hydrogen molecule is the most abundant molecular ion in the universe Y) carbenes are highly reactive intermediates in certain organic reactions Z) recent studies show that the female brain releases up to 3-times more dopamine as compared to males which may account for higher addiction rates in females

ANSWER: Z) RECENT STUDIES SHOW THAT THE FEMALE BRAIN RELEASES UP TO 3- TIMES MORE DOPAMINE AS COMPARED TO MALES WHICH MAY ACCOUNT FOR HIGHER ADDICTION RATES IN FEMALES (Solution: males have higher rates of addiction)

BONUS BIOLOGY Multiple Choice Which of the following BEST describes the function of a spliceosome (read as: SPLY-see-oh-zome): W) splicing DNA together during replication X) splicing out mutations Y) joining multiple copies of structural genes Z) removing introns

ANSWER: Z) REMOVING INTRONS

Multiple Choice Which of the following is an incorrect descriptor of DNA replication? W) Semi-conservative X) Discontinuous on the lagging strand and continuous on the leading strand Y) High-fidelity Z) Repetitive

ANSWER: Z) REPETITIVE

BONUS BIOLOGY Multiple Choice An ecologist uses seven foot tall mist nets to capture birds. In which of these habitats would the number of captures most likely underestimate the actual number of birds present? W) Tundra X) Temperate grassland Y) Chaparral Z) Temperate forest

ANSWER: Z) TEMPERATE FOREST

BONUS BIOLOGY Multiple Choice Which of the following BEST describes what the term virulence means: W) the specificity of a virus to cause disease in a certain host X) the ability of a microbe to cause cellular damage Y) any transmissible disease caused by a microbe Z) the degree to which a microbe causes disease or harm to its host

ANSWER: Z) THE DEGREE TO WHICH A MICROBE CAUSES DISEASE OR HARM TO ITS HOST

[BONUS] Biology Multiple Choice: Which of the following BEST describes what the term virulence means: W) the specificity of a virus to cause disease in a certain host X) the ability of a microbe to cause cellular damage Y) any transmissible disease caused by a microbe Z) the degree to which a microbe causes disease or harm to its host

ANSWER: Z) THE DEGREE TO WHICH A MICROBE CAUSES DISEASE OR HARM TO ITS HOST

BONUS BIOLOGY Multiple Choice Which of the following is true about reverse phase chromatography? W) The stationary phase is hydrophilic [hy-druh-FIL-ik] X) The stationary phase is acidic Y) It is typically used for separation of proteins Z) The stationary phase is hydrophobic

ANSWER: Z) THE STATIONARY PHASE IS HYDROPHOBIC

Multiple Choice Which of the following is NOT true of chloroplasts: W) they are relatively large organelles and can exceed 3 microns long X) thay have smaller ribosomes than are found in the plant cytosol Y) they contain thylakoid (read as: THY-la-koid) stacks Z) their numbers are controlled primarily by the cell's nuclear genes that control division

ANSWER: Z) THEIR NUMBERS ARE CONTROLLED PRIMARILY BY THE CELL'S NUCLEAR GENES THAT CONTROL DIVISION

Biology - Multiple Choice Which of the following instruments would be used to perform a polymerase chain reaction in a molecular biology lab? W) Microtome X) Autoclave Y) Spectrometer Z) Thermocycler

ANSWER: Z) THERMOCYCLER

BONUS Biology - Multiple Choice Which of the following is not a behavior generally associated with cancerous cells? W) They induce angiogenesis [an-gee-oh-gen-uh-sis] X) They produce telomerase [tel-aw-mer-ayse] Y) They cannot stop cell division Z) They cannot proliferate when not adherent to substrate

ANSWER: Z) THEY CANNOT PROLIFERATE WHEN NOT ADHERENT TO SUBSTRATE

BONUS Biology - Multiple Choice Which of the following is not a behavior generally associated with cancerous cells? W) They induce angiogenesis [an-gee-oh-g en-u h-s is] X) They produce telomerase [tel-aw-mer-ayse] Y) They cannot stop cell division Z) They cannot proliferate when not adherent to substrate

ANSWER: Z) THEY CANNOT PROLIFERATE WHEN NOT ADHERENT TO SUBSTRATE

BIOLOGY Multiple Choice In a gel electrophoresis [i-lek-troh-fuh-REE-sis] experiment, in which of the following directions will a DNA sample move, and why? W) Toward the negative electrode because DNA has a positive charge X) Toward the negative electrode because DNA has a negative charge Y) Toward the positive electrode because DNA has a positive charge Z) Toward the positive electrode because DNA has a negative charge

ANSWER: Z) TOWARD THE POSITIVE ELECTRODE BECAUSE DNA HAS A NEGATIVE CHARGE

BIOLOGY Multiple Choice Which of the following methods of dissemination of bacterial genetic material is mismatched to its description? W) Transduction: viral transfer of one cell's DNA to another X) Binary fission: cellular division Y) Conjugation: through physical interaction, a donor cell transfers genetic material to a recipient cell Z) Transformation: one bacteria engulfs another to integrate its DNA

ANSWER: Z) TRANSFORMATION: ONE BACTERIA ENGULFS ANOTHER TO INTEGRATE ITS DNA

BIOLOGY Multiple Choice Which of the following examples describes a postzygotic reproductive barrier that creates or enforces reproductive isolation? W) Two closely-related species of garter snakes live in the same area, but one lives mainly in water while the other lives mainly on land X) Two species of closely-related skunks have different mating seasons Y) Two species of closely-related snails have morphological differences that prevent successful inter-species mating Z) Two species of closely related equines may successfully mate to produce offspring, but the offspring is sterile

ANSWER: Z) TWO SPECIES OF CLOSELY RELATED EQUINES MAY SUCCESSFULLY MATE TO PRODUCE OFFSPRING, BUT THE OFFSPRING IS STERILE

BONUS Biology - Multiple Choice Which of the following traits does NOT make Eukarya distinct from Archaea and Bacteria? W) Presence of a nucleosome structure X) Presence of 80S rather than 70S ribosomes Y) Meiotic [my-o h-tic] chromosome segregation Z) Use of methionine [meth-eye-o h-n een] as initiator tRNA

ANSWER: Z) USE OF METHIONINE AS INITIATOR tRNA

(General Science) [Bonus] Multiple Choice: Which of the following vitamins is NOT properly matched with its function in humans: W) vitamin E and antioxidant action X) vitamin C and collagen production Y) vitamin D and calcium absorption Z) vitamin A and absorption of iron

ANSWER: Z) VITAMIN A AND ABSORPTION OF IRON

(General Science) Multiple Choice Rooted plants that grow below the surface of the water are called submerged plants. Which of the following is BEST classified as a freshwater submerged plant: W) common reed X) sedges Y) water lilies Z) water milfoil

ANSWER: Z) WATER MILFOIL

BIOLOGY Multiple Choice Eukaryotic [yoo-KAR-ee-oh-tik] genes are composed of introns and exons. Which of the following eukaryotes [yoo-KAR-ee-ohts] would have the shortest intron size? W) Mice X) Fruit flies Y) Humans Z) Yeast

ANSWER: Z) YEAST

BONUS BIOLOGY Short Answer Name four of the five structural characteristics shared by all Chordata at some point in their life cycle?

ANSWER: any 4 of DORSAL HOLLOW NERVE CORD (ACCEPT: DORSAL NERVE CORD), NOTOCHORD, GILL-SLITS, POST ANAL TAIL (ACCEPT: TAIL), ENDOSTYLE

BIOLOGY Short Answer What is the MOST common anatomical synonym for ventral, when locating a part of the human body in relation to another part?

ANTERIOR

What is the most common term for the substance recognized as foreign and able to elicit antibody production?

ANTIGEN

The webbed hand of a human embryo develops into five separate fingers by a process of programmed cell death. What is the term for this process?

APOPTOSIS

Ced-3 and ced-4 are genes likely involved in what cell process?

APOPTOSIS (ACCEPT: PROGRAMMED CELL DEATH)

BONUS BIOLOGY Short Answer Name all of the following 4 plants that produce seeds with a fleshy covering that are eaten by animals and dispersed in their droppings: apple; orchid; cherry; sphagnum moss

APPLE; CHERRY

BIOLOGY Short Answer: What is the most common term for the method Louis Pasteur used to create the first rabies vaccine?

ATTENUATION

What plant hormone mediates phototropism?

AUXIN

The Bowman's capsule and the glomerulus [gloh-MER-yuh-luhs] compose what part of the mammalian kidney?

MALPIGHIAN BODY (ACCEPT: MALPIGHIAN CORPUSCLE, RENAL CORPUSCLE, RENAL CORTEX; DO NOT ACCEPT: MALPIGHIAN)

BONUS Short Answer What is the name of the major excretory organ found in insects?

MALPIGHIAN TUBULE

Liverworts are a part of which nonvascular plant phylum?

MARCHANTIOPHYTA (ACCEPT: HEPATOPHYTA or HEPATICOPHYTA)

BIOLOGY Short Answer Into what specific mitochondrial compartment are hydrogen ions pumped out during chemiosmosis (read as: chem-ee-oz-MOE-sis)?

MATRIX

BONUS BIOLOGY Short Answer: Name all of the following 4 diseases that are caused by viruses: measles; AIDS; tuberculosis; leprosy

MEASLES; AIDS

What region of the human brainstem is commonly thought to be responsible for the control of cardiac, respiratory, and other autonomic functions and is connected to the thalamus via the pons?

MEDULLA (ACCEPT: MEDULLA OBLONGATA)

Short Answer Which specific region of the human brainstem regulates heart rate?

MEDULLA OBLONGATA

What hormone, secreted from pinealocytes [pin-ee-A-loh-syt], is thought to be responsible for the regulation of the body's circadian [sur-KAY-ee-uhn] rhythms?

MELATONIN

Pluripotent [ploor-uh-POHT-nt] cells develop from what type of connective tissue?

MESENCHYME (ACCEPT: MESENCHYMAL)

BONUS BIOLOGY Short Answer: What is the layer typically found between 2 plant cell walls of adjoining plant cells which is composed of pectin?

MIDDLE LAMELLA

BONUS BIOLOGY Short Answer: In what specific part of a eukaryotic cell will pyruvate be converted into acetyl Co-enzyme A?

MITOCHONDRION (ACCEPT: MITOCHONDRIA)

What term best describes a plant that has parallel leaf veins, fibrous roots, and flowers with parts in threes?

MONOCOT (ACCEPT: MONOCOTYLEDON)

BIOLOGY Short Answer: What cell in the peripheral blood is the immediate precursor of the macrophage?

MONOCYTE

BONUS Short Answer A plant has both megasporangiate (read as: meh-gah-spoh-RAHNjeet) and microsporangiate (read as: my-krōe-spoh-RAHN-jeet) flowers on the same plant. Indicate the term that describes this type of plant.

MONOECIOUS

[Bonus] In humans, backflow of blood from ventricles into atria during contraction is prevented primarily by what structures?

Answer: VALVES

Biology - Multiple Choice Meiosis I differs from mitosis in that: W) Meiosis I separates sister chromatids whereas mitosis does not X) Meiosis I introduces genetic diversity via crossing over whereas mitosis does not Y) Meiosis I lines up homologous chromosomes at the equator whereas mitosis does not Z) Meiosis I is much shorter than mitosis

Answer: X) MEIOSIS I INTRODUCES GENETIC DIVERSITY VIA CROSSING OVER WHEREAS MITOSIS DOES NOT

BIOLOGY Short Answer: What is the main respiratory pigment of skeletal muscle cells?

MYOGLOBIN

Bonus: In humans,the mucosa of the stomach produces intrinsic factor, which is required for absorption of what vitamin?

B12 (ACCEPT: COBALAMIN)

BIOLOGY Short Answer What is the scientific term given to the shape of the E. coli bacterium?

BACILLUS (ACCEPT: ROD or BACILLI or BACILLIFORM)

Chemoautotrophs [kee-moh-AW-tuh-trofs] produce energy by oxidizing inorganic substances in their surroundings. Name all of the domains that include organisms with chemoautotrophic metabolisms.

BACTERIA AND ARCHAEA

In female mammalian cells, the second X chromosome is generally silenced. What term is used to describe the condensed inactive X chromosome?

BARR BODY

BONUS BIOLOGY Short Answer: What is the most common term for the form of protection when a non-poisonous frog has the same appearance as a poisonous frog?

BATESIAN MIMICRY (ACCEPT: MIMICRY or WARNING MIMICRY) (DO NOT ACCEPT: MULLERIAN MIMICRY)

Identify the food test used to detect the presence of a reducing sugar by the addition of a solution containing sodium carbonate, sodium citrate, and copper sulphate

BENEDICT'S TEST

What early developmental stage of an animal follows the morula stage and consists of a single, spherical layer of cells enclosing a hollow, central cavity?

BLASTULA

BIOLOGY Short Answer Intestinal absorption of what vitamin is most directly dependent on intrinsic factor?

B₁₂

BIOLOGY Short Answer What is the primary calcium-binding protein in eukaryotic cells, that is about 148 amino acids long, and has a wide variety of functions often through kinases and phosphatases (read as: foss-fah-TAY-sis)?

CALMODULIN

BONUS BIOLOGY Short Answer: What is the most common term for the stage of photosynthesis (read as: fōe-tōe-SIN-teh-sis) during which carbon fixation occurs?

CALVIN CYCLE

BIOLOGY Short Answer: In what part of photosynthesis, named after its discoverers, is NADPH used to produce 3-carbon sugar phosphate molecules?

CALVIN CYCLE (ACCEPT: CALVIN-BENSON CYCLE or CALVIN-BENSONBASSHAM CYCLE)

Bonus: The term capnophile, used to describe a microbe, refers to the microbe's preference for environments high in what?

CARBON DIOXIDE (ACCEPT: CARBON DIOXIDE GAS or CO2 or CO2 GAS)

Short Answer What is the term for the process of securing carbon from the environment by incorporating it into a stable organic compound

CARBON FIXATION

BIOLOGY Short Answer: If one end of a single polypeptide chain has an amino group exposed, what functional group is typically exposed at the opposite end?

CARBOXYL

What group of orange unsaturated hydrocarbons is primarily synthesized by plants for use as photosynthetic pigments and also contains a biologically inactive form of vitamin A?

CAROTENES

What fluid, produced by ependymal cells, fills the space between the arachnoid [uh-RAK-noid] and the pia mater as well as the ventricles of the brain?

CEREBROSPINAL FLUID

BONUS BIOLOGY What terrestrial biome occurs in mid-latitude coastal regions and is found in Mediterranean climates such as California?

CHAPARRAL

[BONUS] BIOLOGY Short Answer Name two cellular organelles besides the nucleus that contain a genome.

CHLOROPLAST, MITOCHONDRIA

BIOLOGY Short Answer What is the common name for the 27-carbon lipid whose synthesis is in part regulated by HMG CoA (read as: H, M, G, koh, A) reductase and is the basic starting point for estrogen synthesis?

CHOLESTEROL

Bonus: Tetanus is a disease caused by what genus of endospore-forming bacteria?

CLOSTRIDIUM

BONUS BIOLOGY Short Answer: Which one of the basic 3 plant cell types has irregularly thickened cell walls, retains its cytoplasm, and functions in structural support?

COLLENCHYMA

What general class of animal tissue is characterized by the presence of collagen?

CONNECTIVE

BONUS BIOLOGY Short Answer By name or number, identify all of the following four plants that are C4 plants: 1) corn, 2) rice, 3) soybeans, 4) wheat.

CORN (ACCEPT: 1)

BONUS BIOLOGY Short Answer: Name all of the following 4 plants that produce seeds: cycads; ferns; mosses; conifers

CYCADS; CONIFERS

BIOLOGY Short Answer What is the most common name for the first membrane-associated molecule discovered to function in a wide variety of cells in what is commonly referred to as a second messenger?

CYCLIC AMP (ACCEPT: cAMP)

BIOLOGY Short Answer: What is the MOST common term for the division of a somatic cell into 2 nucleated cells following of mitosis?

CYTOKINESIS

BONUS BIOLOGY Short Answer: What are the names for the pyrimidine bases found in DNA?

CYTOSINE; THYMINE

BIOLOGY Short Answer What is the process by which particles move from an area of higher concentration to an area of lower concentration, not requiring cellular energy?

DIFFUSION

BIOLOGY Short Answer From what specific biological molecule are Barbara McClintock's transposons (read as: trans-POE-zons) made?

DNA

BIOLOGY Short Answer What enzyme in E. coli functions as the primary proof-reading molecule during replication?

DNA POLYMERASE (ACCEPT: DNA POLYMERASE 3)

BONUS BIOLOGY Short Answer: What is the most common name of the protein that synthesizes DNA?

DNA POLYMERASE (ACCEPT: POLYMERASE)

BONUS BIOLOGY Short Answer: What section of the small intestine is the first to receive gastric contents?

DUODENUM

What form of transport across membranes is mediated by channel proteins and requires no input of energy?

FACILITATED DIFFUSION (ACCEPT: FACILITATED TRANSPORT or FACILITATED PASSIVE TRANSPORT ; DO NOT ACCEPT: SIMPLE DIFFUSION)

BONUS BIOLOGY Short Answer Cells that become quiescent enter G-zero from what stage of the cell cycle?

G-1

BIOLOGY Short Answer From the FIRST to the LAST, what are the stages into which interphase is most commonly divided?

G-1; S; G-2 (ACCEPT: GAP ONE, DNA SYNTHESIS, GAP 2)

BONUS Short Answer The energy investment phase of glycolysis (read as: gly-KOH-lisis) results in the formation of which triose sugar?

GADP (ACCEPT: DHAP, GLYCERALDEHYDE 3-PHOSPHATE)

BIOLOGY Short Answer Yersinia, Drosophila and Homo are all what taxonomical category?

GENUS (ACCEPT: GENERA)

BONUS BIOLOGY Short Answer: From what monosaccharide is cellulose primarily composed?

GLUCOSE (ACCEPT: Β-D-GLUCOPYRANOSYL or D-GLUCOSE)

BONUS BIOLOGY Short Answer: Name all of the following 4 choices that are characteristic of E. coli: gram negative; an enteric organism; non-motile; bacillus

GRAM NEGATIVE; AN ENTERIC ORGANISM; BACILLUS (ACCEPT: ALL BUT NON-MOTILE)

BIOLOGY Short Answer: What is the most common color for the calyx (read as: KAL-IX) of angiosperms?

GREEN

What specialized plant cells are responsible for the opening and closing of stomatal airways in response to water stress?

GUARD CELLS

BONUS BIOLOGY Short Answer: What specific cells and how many of them border each stomal pore?

GUARD CELLS; 2

Short Answer By name or number, identify all of the following five choices that are considered embryophyta (read as: ehm-bree-ah-FY-tah): 1) gymnosperms (read as: JIMnoh-spehrms), 2) diatoms, 3) mosses, 4) dinoflagellates, 5) ferns.

GYNOSPERMS, MOSSES, FERNS (ACCEPT: 1, 3, AND 5)

BIOLOGY Short Answer: The sinoatrial (read as: sine-oh-AYE-tree-al) node is located in what human organ?

HEART

In prokaryotic [proh-Kr-ee-OT-ik] DNA replication, what enzyme is responsible for unwinding the DNA double helix?

HELICASE

What type of perennials die back each year and survive as tubers until the next growing season?

HERBACEOUS

BIOLOGY Short Answer From what amino acid is the vasodilator histamine primarily synthesized?

HISTIDINE

BIOLOGY Short Answer: Name all of the following 4 animals for which L-ascorbate is an essential nutrient: humans; dogs; cats; horses

HUMANS

What is the primary force that holds the antiparallel stands of a DNA helix together?

HYDROGEN BOND

Trans fats are produced from unsaturated fats in what chemical process?

HYDROGENATION (ACCEPT: ISOMERIZATION)

BIOLOGY Multiple Choice Which of the following is the target tissue of the hormone leptin where it binds with LepRB (read as: L, E, P, R, B) receptors: W) hypothalamus X) pancreas Y) adipose tissue Z) kidney

HYPOTHALAMUS

BIOLOGY Short Answer: What class of immunoglobulins is most directly responsible for hay fever and common allergies in humans?

IGE (ACCEPT: E or IMMUNOGLOBULIN E)

Huntington disease is caused by the sequence CAG [C-A-G] repeated between 37-100 times, rather than the normal 9-35 times. What type of mutation is this?

INSERTION

BIOLOGY Short Answer: What specific class of antibodies is most commonly responsible for degranulation of basophils or mast cells?

IgE (ACCEPT: IMMUNOLGLOBULIN E)

BONUS BIOLOGY Short Answer: What is the most plentiful class of immunoglobulins in human plasma?

IgG (ACCEPT: G or IMMUNOGLOBULIN G)

Which of the following is NOT a major site of lymphoid tissue? W) Thymus gland [THY-muhs] X) Spleen Y) Tonsils Z) Kidney

KIDNEY

BONUS BIOLOGY Short Answer: What human organ produces the majority of immune complement proteins?

LIVER

BIOLOGY Short Answer What reduced coenzyme donates its electrons to complex-one in the electron transport system?

NADH

BONUS BIOLOGY Short Answer: Beadle and Tatum primarily used what organism to show that mutations in a single gene caused disruptions in metabolic pathways because of the lack of a single enzyme?

NEUROSPORA (ACCEPT: BREAD MOLD or NEUROSPORA CRASSA)

BONUS BIOLOGY Short Answer: Name all of the following 4 cells that are leukocytes: neutrophil; basophil; melanocyte; macrophage

NEUTROPHIL; BASOPHIL

You perform an experiment where you isolate chromatin from sea urchin sperm cells, then partially digest with micrococcal nuclease [my-kruh-KOK-kuhl NOOklee-ays]. When you purify the DNA away from the chromatin proteins and run your DNA on a gel, you find that your DNA fragments have lengths that are multiples of 260 base pairs. What organizational element of chromatin leads to this digestion pattern?

NUCLEOSOME

Bonus: What type of learning involves an animal associating a behavioral response with punishment or reward?

OPERANT LEARNING (ACCEPT: OPERANT CONDITIONING OR INSTRUMENTAL CONDITIONING; DO NOT ACCEPT: CLASSICAL CONDITIONING OR CONDITIONING)

The high sugar content of honey gives it the ability to suppress growth of nearly all microbes via what kind of pressure on potentially colonizing cells?

OSMOTIC PRESSURE

BIOLOGY Short Answer What cell type is MOST directly responsible for secreting the bone matrix in developing humans?

OSTEOBLASTS (DO NOT ACCEPT: OSTEOCYTE)

BONUS BIOLOGY Short Answer What specific human bone cell is antagonistic to osteoblasts and is known to cause apoptosis of osetoblasts in vitro?

OSTEOCLASTS

Name the type of reproduction in which the eggs hatch inside the female body and the embryo is retained for protection?

OVOVIVIPARITY (ACCEPT: OVOVIVIPAROUS; DO NOT ACCEPT: VIVIPARY)

In photorespiration, ribulose bisphosphate carboxylase [RY-byoolays bis-fos-FAYT kahr-BOK-suh-layz] binds what molecule instead of carbon dioxide?

OXYGEN

BONUS BIOLOGY Short Answer During photorespiration, the plant enzyme RuBisCO (read as: rueBIS-ko) will bind what molecular substance rather than carbon dioxide?

OXYGEN (ACCEPT: O₂)

BONUS BIOLOGY Short Answer What protein, that takes its name from its apparent molecular weight, can arrest cells in the G1-S checkpoint, initiate apoptosis (read as: aye-pop-TOE-sis) and, when deactivated, is implicated in a wide variety of cancers?

P53

In the human body, what two hormones regulate the level of calcium in the blood?

PARATHYROID HORMONE AND CALCITONIN (ACCEPT: THYROCALCITONIN)

BIOLOGY Short Answer From what basic plant cell type are the majority of cells in a corm (read as: KORM) composed?

PARENCHYMA

In plants, what class of ground tissue is responsible for most of the plants' photosynthesis?

PARENCHYMA

What specific cell type in the stomach is responsible for production of hydrochloric acid in response to stimulation by gastrin?

PARIETAL CELLS

What is a form of asexual reproduction in which young develop from unfertilized eggs?

PARTHENOGENESIS

BONUS BIOLOGY Short Answer: Name all of the following 4 substances that would typically be considered exocrine products: insulin; pepsin; hemoglobin; epinephrine

PEPSIN

BONUS Short Answer In eudicots (read as: yuh-DY-kohts), lateral roots arise from the mitotic activity of which tissue layer?

PERICYCLE

BONUS BIOLOGY Short Answer: Sieve cells and sieve tube members are types of what specific conductive plant cell type?

PHLOEM

BONUS BIOLOGY Short Answer: What is the general biological term for organisms that synthesize organic molecules from inorganic molecules using light as an energy source?

PHOTOAUTOTROPHS (ACCEPT: PHOTOTROPHS)

BIOLOGY Short Answer: What specific biological process is believed to have produced most of the diatomic oxygen in the Earth's atmosphere?

PHOTOSYNTHESIS

BONUS BIOLOGY Short Answer: Order the following 5 taxonomic group with the MOST members to the one with the FEWEST members: genus; order; phylum; class; family

PHYLUM; CLASS; ORDER; FAMILY; GENUS

BONUS BIOLOGY Short Answer: Name all of the following 4 cells that typically possess mitochondria: erythrocytes; plasma cells; lymphocytes; neurons

PLASMA CELLS; LYMPHOCYTES; NEURONS (ACCEPT: ALL BUT ERYTHROCYTES)

BIOLOGY Short Answer: What is the most common name for the micro-gametophyte in flowering plants?

POLLEN (ACCEPT: POLLEN GRAINS)

Spongiform encephalopathies [en-sef-uh-LOP-uh-thees], such as scrapie [SKRAY-pee], mad cow disease, and kuru [KOO-roo], are produced after exposure to what general group of infectious agents?

PRIONS

BIOLOGY Short Answer: During what phase of meiosis-one do homologous chromosomes exchange genetic information?

PROPHASE-ONE (ACCEPT: PROPHASE)

Which class of organic biomolecules is the major component of animal bodies?

PROTEIN

Sodium dodecyl [duh-DES-uhl] sulfate is a detergent often used to dissolve molecules. In SDS-PAGE, what type of molecule is SDS used to dissolve?

PROTEINS

What is the term for a fluid-filled cavity formed within the mesoderm in triploblastic animals?

PSEUDOCOELOM

Place the following four animals in order of lowest to highest energy cost per unit body weight for regulation of body temperature: python, canary, human, poodle.

PYTHON, HUMAN, POODLE, CANARY

Bonus: What name is used to describe the ability of a microbe to respond to changes in local microbial population density?

QUORUM SENSING

BONUS BIOLOGY Short Answer;: Name all of the following 4 choices that are typically considered human autoimmune dysfunctions: Rheumatoid arthritis; Lupus; shingles; Down's syndrome

RHEUMATOID ARTHRITIS; LUPUS

BIOLOGY Short Answer: What is the common name for the units in DNA to which phosphates are covalently linked?

RIBOSE SUGARS (ACCEPT: PENTOSE SUGARS or SUGARS or DEOXYRIBOSE SUGARS)

BONUS BIOLOGY Short Answer: Order the following 4 objects found in the human body from the SMALLEST to the LARGEST: mitochondrion; ribosome; erythrocyte; average monocyte

RIBOSOME; MITOCHONDRION; ERYTHROCYTE; AVERAGE MONOCYTE

Rough endoplasmic reticulum is named rough because it possesses what cellular structures?

RIBOSOMES

BONUS BIOLOGY Short Answer: In what chamber of the human heart is the sinoatrial (read as: sineoh-AYE-tree-al) node located?

RIGHT ATRIUM

BIOLOGY Short Answer From what chamber of the human heart does blood carried by the left pulmonary artery leave?

RIGHT VENTRICLE

BIOLOGY Short Answer What is the specific intermediate informational molecule for human retrotransposons (read as: rhet-tro-trans-POE-zons):

RNA (ACCEPT: RIBONUCLEIC ACID or mRNA or MESSENGER RNA)

BONUS BIOLOGY Short Answer What specific molecule binds immediately upstream of the structural genes in the lac (read as: LACK) operon and functions to transcribe these genes?

RNA POLYMERASE

Bonus: What system, widely dispersed in eukaryotes, is sometimes used for a similar immune function as the CRISPR [crisper]/Cas9 system, and also used for endogenous gene regulation?

RNAI (ACCEPT: RNA INTERFERENCE, RISC-SIRNA,SIRNA,MICRO RNA)

BONUS BIOLOGY Short Answer: What is the common biological name for the product of human sublingual glands?

SALIVA

Early in the evolution of modern day fish, the superclass Osteichthyes [os-tee-IK-thee-eez] split into two groups, one of which includes fish such as the Coelocanth [see-LUH-kahnth] and lungfish. What is the name for this group of fish?

SARCOPTERYGII (ACCEPT: LOBE FINNED FISH)

BIOLOGY Short Answer: Which one of basic plant cell types has a primary and secondary cell wall that is dead when mature?

SCLERENCHYMA

BONUS BIOLOGY Short Answer: Choosing from primary through quaternary, what level of protein structure is conferred by a beta pleated sheet?

SECONDARY

BONUS BIOLOGY Short Answer: Order the following 3 choices from the EARLIEST stage to the LATEST stage of ovarian follicle development, and identify which of the structures is typically the LARGEST: secondary follicle; corpus luteum; corpus albicans

SECONDARY FOLLICLE; CORPUS LUTEUM; CORPUS ALBICANS; LARGEST = CORPUS LUTEUM

BIOLOGY Short Answer: In what specific part of the testes does spermatogenesis occur in mammals?

SEMINIFEROUS TUBULES

BONUS BIOLOGY Short Answer: What 2 ions are most directly involved with the conduction of a nerve cell message down an axon?

SODIUM AND POTASSIUM (ACCEPT: Na⁺ AND K⁺ )

A group of butterflies occurs in many color patterns. The butterflies can interbreed and the offspring are fertile. What is the most appropriate taxonomical rank for this group of butterflies?

SPECIES

BONUS BIOLOGY Short Answer: In photosynthesis II (read as: two), what is light energy most notably used to initially accomplish?

SPLITTING OF WATER

What is the only multicellular animal without a permanent gut cavity?

SPONGE (ACCEPT: PORIFERAN, PORIFERA, or PLACOZOA

What form of active transport involves moving two or more different molecules across a membrane in the same direction, coupling the favorable transport of one molecule to the unfavorable transport of the other?

SYMPORT

Bonus: What is the name forskeletal muscle cells containing multiple nuclei?

SYNCYTIUM (ACCEPT:SYMPLASM)

What is the defining functional structure of the kidney?

THE NEPHRON

Name the growth response to generalized mechanical disturbances in plants.

THIGMOMORPHOGENESIS (ACCEPT: THIGMOTROPISM)

Bonus: 5-methylcytosine [methil-SYE-toh-seen] is particularly susceptible to mutations because upon deamination [dee-am-ih-NAY-shun], it becomes what other nucleobase?

THYMINE

BONUS BIOLOGY Short Answer: What human gland, which is mostly lymphoid tissue, is critical to the normal development of cell mediated immunity and, if removed after young adulthood, seems to have no ill effects on the immune system?

THYMUS

BONUS BIOLOGY Short Answer: What human gland or organ produces tri-iodothyronine (read as: TRY- I-OH-doh-THY-row-neen)?

THYROID

BONUS BIOLOGY Short Answer Name all of the following 4 hormones that are formed primarily from the amino acid tyrosine: glucagon; thyroxine; epinephrine; erythropoietin (read as: EE-rithroe-POE-ih-tin)

THYROXINE; EPINEPHRINE

BONUS BIOLOGY Short Answer: Name all of the following 4 diseases that are caused by protists: botulism; rabies; typhoid fever; toxoplasmosis

TOXOPLASMOSIS

BONUS BIOLOGY Short Answer Order the following 4 processes from the one to typically occur the EARLIEST to the one to occur the LATEST in the production of collagen: (read slowly) spliceosome (read as: SPLY-see-oh-zome); transcription; translation; golgi modification

TRANSCRIPTION; SPLICEOSOME; TRANSLATION; GOLGI MODIFICATION

BIOLOGY Short Answer: What specific type of microscopy passes a beam of electrons through an ultra-thin specimen that is usually stained with an electron opaque substance, producing an image which can be captured by a charged coupled device?

TRANSMISSION ELECTRON MICROSCOPY (ACCEPT: TEM) (DO NOT ACCEPT: SCANNING ELECTRON MICROSCOPY OR SEM)

BIOLOGY Short Answer: What is the specific type of protein from which spindle fibers are composed?

TUBULIN (ACCEPT: ALPHA AND/OR BETA TUBULIN)

Luke Skywalker is looking for evidence about whether Darth Vader was his father. Luke has blood type A, and medical records show that his mother had blood type AB. What are all the possible blood types that Luke's father could be?

TYPES A, B, O, AB (SAYING "TYPES" IS NOT NECESSARY)

Dopamine, norepinephrine [nohr-ep-uh-NEF-rin], and epinephrine [e-puh-NE-frin] are all derivatives of what amino acid?

TYROSINE

BONUS BIOLOGY Short Answer: What is the anticodon on a tRNA for AAA (read as: A, A, A)?

UUU

BIOLOGY Multiple Choice: At which of the following pH levels will human pepsin be most active: W) 2 X) 4 Y) 6 Z) 8

W) 2

BONUS BIOLOGY Multiple Choice Which of the following is the RNA complementary sequence of the following sequence: 5'CCGCGA 3' (read as: 5-prime, CCGCGS, 3-prime) W) 5' UCGCGG 3' (read as: 5-prime, UCGCGG, 3-prime) X) 5' GGCGCU 3' Y) 5' GGCGCT 3' Z) 5' TCGCGG 3'

W) 5' UCGCGG 3'

Plant cells are distinguished from animal cells by having: W) A large central vacuole, chloroplasts, and a cell wall made of cellulose X) A large central vacuole, mitochondria, and a nucleus Y) A cell wall made of cellulose,Golgi apparatus, and chloroplasts Z) Chloroplasts, mitochondria, and a nucleus

W) A LARGE CENTRAL VACUOLE, CHLOROPLASTS, AND A CELL WALL MADE OF CELLULOSE

In a human, breathing air with too much carbon dioxide would lead to all of the following except: W) A reduction in oxygen carrying capacity X) A decrease in oxygen affinity for hemoglobin Y) Facilitation of oxygen to tissues Z) A rightward shift of the oxygen equilibrium curve

W) A REDUCTION IN OXYGEN CARRYING CAPACITY

Tunicamycin [too-ni-kuh-MY-sin] is a drug produced by some bacteria. It is used as a tool by molecular biologists because it induces eukaryotic [yoo-KARee-oh-tik] cells to arrest at the G1 checkpoint of the cell cycle. Using this information about tunicamycin and your knowledge of the cell cycle, which of the following would you expect to observe in a tunicamycin-arrested cell? W) Activated APC protein X) Chiasmata [ky-AZ-muh-tuh] Y) Replication bubbles Z) Spindle fibers

W) ACTIVATED APC PROTEIN

The citric acid cycle is a component of what kind of metabolic process? W) Aerobic respiration X) Anaerobic respiration Y) Fermentation Z) Photosynthesis

W) AEROBIC RESPIRATION

BIOLOGY Multiple Choice: Which of the following tissues is most directly responsible for the growth in height of a tree: W) apical meristem X) shoot terminus Y) terminal cambial Z) cambium terminus

W) APICAL MERISTEM

Which of the following glial cell types makes up the blood-brain barrier? W) Astrocytes X) Oligodendrocytes [OL-i-goh-DEN-ruh-syts] Y) Radial glia Z) Schwann cells

W) ASTROCYTES

Which of the following plant hormones is manufactured primarily in the shoot tips, in embryos, and in parts of developing flowers and seeds? W) Auxins [AWK-sins] X) Cytokinins [sy-tuh-KY-nins] Y) Gibberellins [jib-uh-REL-ins] Z) Ethylene

W) AUXINS

Which of the following vitamins is NOT synthesized by plants or animals? W) B12 X) B6 Y) B2 Z) C

W) B12

Which of the following molecules is most similar in structure to vitamin A? W) Beta carotene X) Tocopherol [toh-KOF-ur-all] Y) Dopamine Z) Estradiol [es-trah-DYE-awl]

W) BETA CAROTENE

Some plants are able to adapt to normally toxic levels of metals in soil by absorbing and storing high concentrations of the metals within themselves. By doing this, these plants are able to detoxify the surrounding soil. Restoration ecologists use these plants as a form of: W) Bioremediation, to remove harmful substances from the environment X) Bioremediation, to add helpful plant life to the environment Y) Biological augmentation, to add helpful plant life to the environment Z) Biological augmentation, to remove harmful substances from the environment

W) BIOREMEDIATION, TO REMOVE HARMFUL SUBSTANCES FROM THE ENVIRONMENT

BONUS BIOLOGY Multiple Choice: The black fur allele B (read as: big b), is dominant over the white fur allele b (read as: little b). Two mice with black fur mate, producing four offspring: one is white and three are black. What are the likely genotypes of the two parents? W) Bb and Bb X) BB and Bb Y) BB and BB Z) BB and bb

W) Bb AND Bb

Which of the following was NOT a component of the Miller-Urey [MILL-er YOO-ree] experiment test conditions for the chemical origins of life? W) Carbon dioxide X) Hydrogen Y) Ammonia Z) Water

W) CARBON DIOXIDE

What is the name of the enzyme that catalyzes the formation of bicarbonate and hydronium ions from CO2 and H2O [C-O two and H two O]? W) Carbonic anhydrase [an-HY-drays] X) Carbonic carboxylase [kahr-BOK-suh-layz] Y) Carbonic decarboxylase Z) Carbonic dehydrogenase [dee-HY-druh-juh-nays]

W) CARBONIC ANHYDRASE

The suprachiasmatic nucleus is involved in controlling which of the following? W) Circadian rhythms X) Body temperature Y) Heart rate Z) Respiration rate

W) CIRCADIAN RHYTHMS

Which of the following processes is used to produce additional ATP for thelight-independent reactions of photosynthesis? W) Cyclic electron flow X) Lactic acid fermentation Y) Glycolysis [glye-KAWL-eh-sis] Z) Beta oxidation

W) CYCLIC ELECTRON FLOW

If the average fur thickness of wild foxes in the northern hemisphere increased with increasing latitude, this variation would be an example of which of the following. W) Directional selection X) Genetic drift Y) Balancing selection Z) Polymorphism

W) DIRECTIONAL SELECTION

In the human digestive system, nutrient absorption predominantly occurs in which of the following? W) Duodenum [doo-uh-DEE-nuhm] and jejunum [ji-JOO-nuhm] X) Stomach and duodenum Y) Jejunum and ileum [IL-ee-uhm] Z) Ileum and cecum [SEE-kuhm]

W) DUODENUM AND JEJUNUM

Bonus: Which of the following is NOT a biogenic amine? W) Dynorphin [dye-NOR-fin] X) Norepinephrine [NOR-eh-pih-NEH-frin] Y) Dopamine Z) Histamine

W) DYNORPHIN

BIOLOGY Multiple Choice Which of the following would BEST describe the quaternary structure of collagen: W) fibrous X) globular Y) planar Z) transport

W) FIBROUS

Which of the following animal cell junctions is most similar in function to plant cell plasmodesmata [plaz-muh-des-MUH-tuh]? W) Gap junction X) Tight junction Y) Desmosome Z) Adherens junction

W) GAP JUNCTION

Which of the following visual disorders results in large pressure increases within the eye? W) Glaucoma X) Astigmatism Y) Macular degeneration Z) Cataracts

W) GLAUCOMA

If you lower the intracellular levels of fructose 2, 6 biphosphate, what will happen to the rate of gluconeogenesis [gloo-koh-nee-uh-JEN-uh-sis] and glycolysis [gly-KOL-uh-sis], respectively? W) Gluconeogenesis speeds up, glycolysis slows down X) Gluconeogenesis slows down, glycolysis speeds up Y) Gluconeogenesis and glycolysis both slow down Z) Glycolysis speeds up and gluconeogenesis halts

W) GLUCONEOGENESIS SPEEDS UP, GLYCOLYSIS SLOWS DOWN

Which of the following animals transfers oxygen to blood via a process called countercurrent exchange? W) Goldfish X) Tarantula Y) Hummingbird Z) Mole

W) GOLDFISH

Bonus: Which of the following types of cells could one find surrounding a stomatal pore? W) Guard cells X) Mesophyll [MEH-zoh-fill] cells Y) Palisade cells Z) Trace cells

W) GUARD CELLS

Moths are nocturnal, have a good sense of smell, and are also pollinators of a variety of plant species, many of which have evolved to better suit the needs of their primary pollinators. Which of the following traits would you expect to find in flowers of moth-pollinated plants? W) Heavy, musky odors X) Dangling flowers Y) Large, red flowers Z) Blue or purple flowers

W) HEAVY, MUSKY ODORS

BIOLOGY Multiple Choice Glucose is an example of what class of sugars? W) Hexose X) Pentose Y) Tetrose Z) Dextrose

W) HEXOSE

Compared to a seawater acclimated fish, a freshwater acclimated fish would be expected to have which of the following? W) Higher glomerular filtration rates [gloh-MER-you-lur] X) Increased drinking rates Y) Decreased water gain Z) Increased ion loss

W) HIGHER GLOMERULAR FILTRATION RATES

BONUS Multiple Choice Which of the following is the primary determinant of secondary protein structure? W) Hydrogen bonding X) Amino acid sequence Y) Protein free energy Z) Global energy minimum

W) HYDROGEN BONDING

Multiple Choice Which of the following is a model of enzyme action? W) Induced fit X) Lipid bilayer Y) Activator action Z) Active site

W) INDUCED FIT

Which of the following structures is referred to as our body's internal gyroscope? W) Inner ears X) Eyes Y) Outer ears Z) Nose

W) INNER EARS

BIOLOGY Multiple Choice From which of the following does the seed coat of angiosperm seeds typically develop: W) integuments X) polar nuclei Y) nucleus Z) cotyledons

W) INTEGUMENTS

Bonus: SDS is a detergent. Which of the following explains how SDS denatures proteins? W) Interferes with the hydrophobic interactions that normally stabilize the proteins X) Interferes with the hydrophillic interactions that normally stabilize the proteins Y) Interferes with the amphiphillic [amfih-FILL-ic]interactions that normally stabilize the proteins Z) Breaks the peptide bonds

W) INTERFERES WITH THE HYDROPHOBIC INTERACTIONS THAT NORMALLY STABILIZE THE PROTEINS

Bonus: Which of the following best describes a lysogenic cell? W) It contains a prophage X) It is immune to viral infection Y) It is actively producing viral particles Z) It is about to lyse

W) IT CONTAINS A PROPHAGE

BONUS BIOLOGY Multiple Choice Which of the following is NOT true of DNA replication: W) it is initiated by a short DNA primer X) the growing strand is polymerized in a 5-prime to 3-prime direction Y) DNA polymerase helps form the phosphodiesterase (read as: FOSS-foe-die-ESS-ter-ase) bonds Z) synthesis of the lagging strand is discontinuous

W) IT IS INITIATED BY A SHORT DNA PRIMER

BONUS BIOLOGY If you needed to find a sample of simple cuboidal epithelium [eh-pih-THEEL-ee-um] in the human body, you would want tissue from which of the following organs? W) Kidneys X) Lungs Y) Esophagus Z) Stomach

W) KIDNEYS

All except which of the following can act as direct substrates for the production of acetyl CoA? W) Lactate X) Amino acids Y) Fatty acids Z) Ketone bodies

W) LACTATE

Which of the following receptors could NOT be found on the surface of a cell? W) Laminar receptor X) Ion-channel coupled receptor Y) Tyrosine [TIE-row-seen] kinase receptor Z) G-protein coupled receptor

W) LAMINAR RECEPTOR

BIOLOGY Multiple Choice Which of the following is MOST characteristic of human blast cells in on observation with light microscopy: W) large nuclear to cytoplasmic ratio X) small nucleolus and mostly heterochromatin in the nucleus Y) lack of cytoplasmic granules and small overall size compared to mature cells Z) absence of nucleolus (read as: NEW-clee-oh-luss) and endoplasmic reticulum

W) LARGE NUCLEAR TO CYTOPLASMIC RATIO

BIOLOGY Multiple Choice: Which of the following is the most common term for an area of primitive cells in plants where active cell division occurs: W) meristem X) ground tissue Y) primary cells Z) germinal tissue

W) MERISTEM

BONUS BIOLOGY Multiple Choice: How does neural transmission across a synaptic gap occur? W) Neural impulses cause the release of neurotransmitters that diffuse across the gap X) Neural impulses open potassium-gated channels, allowing sodium to move across the gap Y) Neural impulses travel across the gap as unidirectional electrical currents Z) Neural impulses travel across the gap as multidirectional electrical currents

W) NEURAL IMPULSES CAUSE THE RELEASE OF NEUROTRANSMITTERS THAT DIFFUSE ACROSS THE GAP

All codon combinations specify a particular amino acid, with the EXCEPTION of which of the following? W) Nonsense codons X) Missense codons Y) Promoters Z) Silent codons

W) NONSENSE CODONS

Which of the following electrophoresis methods is used to analyze RNA? W) Northern Blot X) Southern Blot Y) Eastern Blot Z) Western Blot

W) NORTHERN BLOT

Which of the following does albumin [al-BYOO-muhn] help to regulate? W) Osmotic pressure X) Electron transport Y) DNA repair Z) Heme maturation [HEEM

W) OSMOTIC PRESSURE

Photosynthesis requires water as a reactant. In which of the following product molecules do the oxygen atoms from the water end up? W) O₂ X) H₂O Y) C₆H₁₂O₆ [C- six H- twelve O- six] Z) ATP

W) O₂

Which of the following enzymes is unique to gluconeogenesis [gloo-koh-nee-uh-JEN-uh-sis]? W) PEP carboxykinase [kahr-bok-si-KY-nays] X) Pyruvate kinase [py-ROO-vayt] Y) Hexokinase [hek-soh-KY-nays] Z) Phosphofructokinase [fohs-foh-frook-toh-KY-nays]

W) PEP CARBOXYKINASE

In photosynthesis, where do electrons derived from water first enter the process? W) Photosystem II reaction center X) Photosystem I reaction center Y) Both light-harvesting complexes Z) P700 chlorophylls

W) PHOTOSYSTEM II REACTION CENTER

Electroporation of a cell enables which of the following? W) Polar molecules to pass through the plasma membrane X) Glucose to escape the plasma membrane Y) Polar molecules to intercalate into the plasma membrane [in-TUR-kuh-layt] Z) Non-polar molecules to escape the plasma membrane

W) POLAR MOLECULES TO PASS THROUGH THE PLASMA MEMBRANE

Which of the following groups of organisms would contain the most biomass productivity in a trophic pyramid? W) Primary Producers X) Primary Consumers Y) Detritivores [de-TRY-tuh-vohrs] Z) Tertiary Consumers

W) PRIMARY PRODUCERS

Which of the following phases of meiosis [my-OH-sis] is longest? W) Prophase I X) Prophase II Y) Metaphase I Z) Anaphase II

W) PROPHASE I

BIOLOGY Multiple Choice: Which of the following types of biological molecules is digested by trypsin: W) proteins X) nucleic acids Y) carbohydrates Z) lipids

W) PROTEINS

Bonus: Which of the following human cell types do NOT perform aerobic respiration? W) Red blood cells X) Heart muscle cells Y) Hair cells Z) Outer skin cells

W) RED BLOOD CELLS

In which of the following female cervids [SUR-vids] would you find antlers? W) Reindeer X) Okapi [oh-KAH-pee] Y) Mule deer Z) Moose

W) REINDEER

Which of the following animals possess two circuits of blood flow, the pulmocutaneous [POOL-moh-kyoo-tay-NEE-uhs] and systemic? W) Salamanders X) Snakes Y) Lizards Z) Birds

W) SALAMANDERS

In which of the following habitats would you expect to find the highest productivity AND lowest diversity of species? W) Salt marsh X) Desert Y) Deciduous forest [di-SIJ-oo-uhs] Z) Grassland

W) SALT MARSH

Which of the following is a characteristic of monocots, but NOT eudicots [yoo-dy-KOTS]? W) Scattered stem vascular bundles X) Taproot Y) Flower parts in multiples of four or five Z) Netted leaf venation

W) SCATTERED STEM VASCULAR BUNDLES

What substance composes the frustule [FRUH-stool] of a diatom? W) Silica X) Cellulose Y) Keratin Z) Calcium carbonate

W) SILICA

In what organ are dietary fats primarily absorbed? W) Small intestine X) Large intestine Y) Stomach Z) Liver

W) SMALL INTESTINE

Multiple Choice The drug Digoxin from the Digitalis plant slows the reuptake of calcium by the sarcoplasmic reticulum (read as: ri-TI-kyuh-luhm). What is the effect of this on the strength and rate of heart contractions? W) Strengthens and slows the heartbeat X) Strengthens and speeds the heartbeat Y) Weakens and slows the heartbeat Z) Weakens and speeds the heartbeat

W) STRENGTHENS AND SLOWS THE HEARTBEAT

Which of the following would be least likely to be a life history trait of an r-selected organism? W) Strong competitive ability X) Rapid development Y) Early reproductive age Z) Found in disturbed habitats

W) STRONG COMPETITIVE ABILITY

Bonus: Which of the following describes the relationship between glucose and fructose? W) Structural isomers X) Diastereomers [dye-ah-STARE-ee-oh-mers] Y) Epimers Z) Enantiomers [en-AN-tee-oh-mirz]

W) STRUCTURAL ISOMERS

In regard to human anatomy, with which of the following is Wolff's law concerned? W) The shape of bone being determined by mechanical and gravitational stresses X) The negative feedback loops that control production of thyroid hormone Y) The forces generated by isotonic contractions of skeletal muscle Z) The electrical potential of resting cardiac cells

W) THE SHAPE OF BONE BEING DETERMINED BY MECHANICAL AND GRAVITATIONAL STRESSES

BONUS Multiple Choice Which of the following BEST explains why water molds are NOT classified as fungi? W) They have cell walls of cellulose and produce motile spores X) They are decomposers Y) They are multinucleate Z) They have cell walls of chitin (read as: KY-tin) and when aggregated function like a multicellular organism

W) THEY HAVE CELL WALLS OF CELLULOSE AND PRODUCE MOTILE SPORES

Beriberi is caused by deficiency in which of the following vitamins? W) Thiamine [THY-uh-min] X) Retinol Y) Ascorbic Acid Z) Cobalamin [koh-BAL-uh-min]

W) THIAMINE

BIOLOGY Multiple Choice: Which of the following is the term for the transfer of genes by viruses from one prokaryote to another: W) transduction X) conjugation Y) transformation Z) relaxation

W) TRANSDUCTION

Which of the following is the most similar to the ecosystem type that would be found at 4000m elevation in the Andes? W) Tundra X) Broadleaf forest Y) Chaparral [shap-uh-RAL] Z) Temperate grassland

W) TUNDRA

BIOLOGY Multiple Choice: Some plants spread their pollen via the wind while others use animals to spread their pollen. When a plant uses an animal to spread its pollen, this type of pollination is called: W) Vector pollination X) Seed dispersal Y) Evolutionary adaptation Z) Reproductive guarantee

W) VECTOR POLLINATION

Which of the following vitamin deficiencies causes the condition known as "Pellagra"? W) Vitamin B3 X) Vitamin B6 Y) Vitamin B2 Z) Vitamin B1

W) VITAMIN B3

Which of the following is an organic dietary requirement that is only needed in small amounts? W) Vitamins X) Minerals Y) Enzymes Z) Coenzymes

W) VITAMINS

Maple syrup is extracted from maple trees by tapping what part of the plant? W) Xylem [ZYE-lum] X) Phloem [FLOW-em] Y) Cambium [KAM-bee-um] Z) Bark

W) XYLEM

Assume that genes A and B are not linked. If an organism is heterozygous [het-er-uh-ZY-guhs] for both genes, what is the probability that the organism will produce a gamete [GAM-eet] with both the dominant A and B alleles [uh-LEELS]? W) ¼ X) ½ Y) 1 Z) 0

W) ¼

The size of most mammalian cells is closest to which of the following? W) 1 picometer X) 1 micrometer Y) 1 millimeter Z) 1 centimeter

X) 1 MICROMETER

A plant has a somatic chromosome number of 2n = 22. After a cell in this plant undergoes mitosis, how many chromosomes will the daughter cells each have? W) 11 X) 22 Y) 33 Z) 44

X) 22

BIOLOGY Multiple Choice In which of the following wavelengths does DNA absorb the best: W) 3,900 angstroms X) 260 nanometers Y) 340 nanometers Z) 540 nanometers

X) 260 NANOMETERS

BONUS BIOLOGY Multiple Choice Which of the following is closest to the diameter of a plasmodesmata (read as: PLAS-mo-dez-MAH-tah): W) 2 angstroms X) 60 nanometers Y) 80 microns Z) 180 microns

X) 60 NANOMETERS

If a cell has 8 chromosomes during mitotic prophase, how many chromosomes will it have during metaphase and anaphase? W) 16 in metaphase, 8 in anaphase X) 8 in metaphase, 8 in anaphase Y) 8 in metaphase, 4 in anaphase Z) 4 in metaphase, 8 in anaphase

X) 8 IN METAPHASE, 8 IN ANAPHASE

Which of the following neurotransmitters typically has a fast excitation response? W) Aminobutyric acid [uh-MEE-noh-byoo-TIR-ik] X) Acetylcholine [uh-seet-l-KOH-leen] Y) Glycine [GLY-seen] Z) Histamine

X) ACETYLCHOLINE

Certain plants cannot convert nitrate to ammonia because they lack the enzyme nitrate reductase. What would you expect for the pH of the environments where these plants are found? W) Basic X) Acidic Y) Neutral Z) Highly basic

X) ACIDIC

BIOLOGY Multiple Choice Which of the following BEST explains why leaves fall off trees in the autumn: W) the leaves die and passively fall off X) an abscission layer forms at the base of the leaf stem causing it to fall off Y) the leaves slowly dry out from the bottom up because of abscisic acid that prevents phloem from delivering nutrients Z) the vascular tissue of the leaves fill up with resin increasing their stiffness and causing them to become brittle

X) AN ABSCISSION LAYER FORMS AT THE BASE OF THE LEAF STEM CAUSING IT TO FALL OFF

BIOLOGY Multiple Choice: When sound waves travelling through the air reach the human ear, the cochlea (read as: KŌE-klee-ah) converts the sound wave into: W) A chemical response transmitted through the blood stream X) An electrical impulse sent to the brain Y) A lower frequency wave, which resonates within the ear Z) Many small wave vectors that are separately communicated to the brain

X) AN ELECTRICAL IMPULSE SENT TO THE BRAIN

Which of the following instruments would be used to sterilize glassware for use in a biology lab? W) Microtome X) Autoclave Y) Spectrometer Z) Thermocycler

X) AUTOCLAVE

Which of the following hormones causes plants to grow toward light? W) Cytokinin [sy-tuh-KY-nin] X) Auxin [AWK-sin] Y) Gibberellin [jib-uh-REL-in] Z) Abscisic

X) AUXIN

What are the most important sensory receptors for the monitoring and regulation of blood pressure in mammals? W) Statocysts X) Baroreceptors Y) Nocireceptors [NOH-si-SEP-tuhrs] Z) Chemoreceptors

X) BARORECEPTORS

If the only method of introducing genetic variation into a sample population of E. coliis by spontaneous mutation, which of the following is the best way to increase the prevalence of beneficial mutations over many generations? W) Conjugation X) Binary fission Y) Transduction Z) Horizontal gene transfer

X) BINARY FISSION

Which of the following correctly describes why antifungal medications are often harder to design than antibacterials? W) Antifungals must target peptidoglycan [pep-ti-doh-GLY-kan] X) Both fungi and humans are eukaryotes [yoo-KAR-ee-ohts] Y) Bacteria and humans have similar cellular structures Z) Most antifungals target chitin building enzymes

X) BOTH FUNGI AND HUMANS ARE EUKARYOTES

Amniotes became very well adapted to life on land. Which of the following is not an adaptation of amniotes to a terrestrial existence? W) Eggs resistant to desiccation X) Buccal pumping Y) Water-conserving kidneys Z) Thoracic breathing

X) BUCCAL PUMPING

BIOLOGY Multiple Choice: Which of the following molecules do methanogens (read as: methAN-oh-gens) most often use as their final electron acceptor in their generation of energy: W) hydrogen X) carbon dioxide Y) methane Z) oxygen

X) CARBON DIOXIDE

BONUS BIOLOGY Multiple Choice In a double reciprocal plot of Michaelis-Menton kinetics, what is the effect of a competitive inhibitor: W) change in the vertical axis intercept X) change in the slope only Y) change in the horizontal axis intercept only Z) change in both vertical and horizontal axes intercepts

X) CHANGE IN THE SLOPE ONLY

BIOLOGY Multiple Choice: Which of the following was first proposed by Peter Mitchell as a potential energy ion gradient across a selectively permeable membrane that is responsible for the production of ATP: W) osmosis X) chemiosmosis (read as: chem-ee-oz-MOE-sis): Y) electro-phosphorylation Z) photosynthesis

X) CHEMIOSMOSIS

Over time, a large mountain range forms, separating two populations of squirrels for multiple generations. Which of the following pairs of terms best describes this speciation event? W) Anagenesis and allopatric speciation [an-uh-JEN-uh-sis] X) Cladogenesis and allopatric speciation [klad-uh-JEN-uh-sis] Y) Anagenesis and sympatric speciation Z) Cladogenesis and sympatric speciation

X) CLADOGENESIS AND ALLOPATRIC SPECIATION

BIOLOGY Multiple Choice: Eggs exit a chicken's body through what specific structure: W) rectal plexus X) cloaca (read as: KLO-aye-ka) Y) rostrum Z) spiculum

X) CLOACA

BONUS BIOLOGY Multiple Choice: Which of the following is a group of about 20 proteins that aid in the attraction and phagocytosis of microorganisms by macrophages: W) alpha interferons X) complement system Y) TNF Z) hemopoietins

X) COMPLEMENT SYSTEM

What location in a seed is equivalent to the embryonic leaf? W) Radicle X) Cotyledon [kat-uhl-EED-uhn] Y) Hypocotyl [hy-puh-KOT-l] Z) Epicotyl [ep-i-KOT-l]

X) COTYLEDON

Which of the following would you NOT expect to experience during fasting or starvation? W) Increase in ketone body formation X) Decrease in plasma fatty acid levels Y) Depletion of glycogen [GLY-kuh-juhn] Z) Increased use of ketones for ATP production in the brain

X) DECREASE IN PLASMA FATTY ACID LEVELS

Which of the following functions is least important in the open circulatory system of insects? W) Delivery of nutrients X) Delivery of oxygen Y) Circulation of immune cells Z) Movement of signaling molecules

X) DELIVERY OF OXYGEN

A decrease in which of the following would NOT result in a decrease in blood pressure? W) Heart rate X) Diameter of arterioles [ahr-TEER-ee-ohls] Y) Cardiac output Z) Stroke volume

X) DIAMETER OF ARTERIOLES

BIOLOGY Multiple Choice: In which of the following organ systems of the human body do chief cells primarily function: W) excretory X) digestive Y) nervous Z) reproductive

X) DIGESTIVE

Topoisomerases [toh-poh-eye-SAW-mer-aces]are involved in which of the following processes? W) DNA transcription X) DNA replication Y) RNA translation Z) RNA processing

X) DNA REPLICATION

In 1937, a medicinal elixir produced by a pharmaceutical company killed 71 adults and 34 children because the solvent for the drug was diethylene [dyE-thi-leen] glycol, a chemical used now as antifreeze. Diethylene glycol is toxic because it is converted to oxalic acid by alcohol dehydrogenase [dee-HY-druh-juh-nays] and other enzymes, which damages the kidneys and brain. If given early enough, ethanol is a treatment for this poisoning. Which of the following reasons is the most likely explanation for why ethanol is an effective treatment? W) Ethanol is an allosteric inhibitor of alcohol dehydrogenase X) Ethanol is a more competitive substrate for alcohol dehydrogenase than diethylene glycol Y) Diethylene glycol has a stronger affinity for ethanol than for alcohol dehydrogenase Z) Ethanol cleaves the diethylene glycol molecule through an electrophilic attack on the alphacarbon of diethylene glycol

X) ETHANOL IS A MORE COMPETITIVE SUBSTRATE FOR ALCOHOL DEHYDROGENASE THAN DIETHYLENE GLYCOL

BONUS Multiple Choice Which of the following plants is NOT a succulent? W) Cactus X) Eucalyptus Y) Aloe Z) Yucca

X) EUCALYPTUS

During which of the following phases of an action potential do voltage-gated sodium channels begin to close? W) Rising phase X) Falling phase Y) After hyperpolarization Z) Refractory perioda

X) FALLING PHASE

Which of the following terms is used to describe the center of the field of vision? W) Optic nerve X) Fovea [FOH-vee-uh] Y) Pupil Z) Blind spot

X) FOVEA

Which of the following terms is used to describe the movement of leaves and flowers toward the sun? W) Thigmomorphogenesis [THIG-moh-mawr-fuh-JEN-uh-sis] X) Heliotropism [hee-lee-O-truh-piz-uhm] Y) Nastic movements Z) Siesmonastic movements [sys-moh-NUH-stik]

X) HELIOTROPISM

BIOLOGY Multiple Choice: Which of the following molecules would have the most peptide bonds: W) glycogen X) hemoglobin Y) lecithin Z) DNA

X) HEMOGLOBIN

Konrad Lorenz, a noted behavioral psychologist, was famous for conditioning a gaggle of geese at a particular age in their development to adopt his rubber boots as their mother. This kind of animal behavior results from which of the following? W) Habituation X) Imprinting Y) Instinct Z) Nurturing

X) IMPRINTING

If the ambient temperature drops 10 degrees Celsius, what would happen to the body temperature of a rabbit and a snake, respectively? W) It would drop linearly for the rabbit and exponentially for the snake X) It would stay the same for the rabbit and drop linearly for the snake Y) It would stay the same for the rabbit and drop exponentially for the snake Z) It would drop linearly for the rabbit and for the snake

X) IT WOULD STAY THE SAME FOR THE RABBIT AND DROP LINEARLY FOR THE SNAKE

Bonus: Based on the formation of a phragmoplast [FRAG-moh-plast], which of the following pairs of organisms are most closely related? W) Cyanobacteria and ameobas X) Land plants and green algae Y) Red algae and brown algae Z) Land plants and cyanobacteria

X) LAND PLANTS AND GREEN ALGAE

Which of the following mammalian groups does NOT have a placenta? W) Marsupials [mahr-SOO-pee-uhls] X) Monotremes [MON-uh-treems] Y) Cetacea [si-TAY-shuh] Z) Chiroptera [kee-ROHP-tuh-ruh]

X) MONOTREMES

Duchenne [duh-SHEEN] muscular dystrophy is the result of a recessive disease allele [uh-LEEL] for a gene on the X-chromosome. Which of the following is true regarding inheritance of the disease? W) Females have twice the chance of developing it, because they have two X chromosomes X) Mothers with the disease allele can pass it with equal probability to sons or daughters Y) Inbreeding has no effect on the likelihood of inheriting this disease Z) An affected son is always produced if his father has the disease allele

X) MOTHERS WITH THE DISEASE ALLELE CAN PASS IT WITH EQUAL PROBABILITY TO SONS OR DAUGHTERS

If you were to selectively extract protein molecules from the H zone of a sarcomere [SAHR-kuh-meer], which of the following would be found in the extract? W) Actin X) Myosin [MY-uh-sin] Y) Titin Z) Tropomyosin [troh-puh-mee-UH-sin]

X) MYOSIN

You are conducting an experiment that involves incubating a heat-insensitive enzyme with its substrate, while slowly increasing the temperature of the test tube. The concentration of the substrate is much higher than the concentration of the enzyme. Can you use the Michaelis-Menten [mi-KAY-lis MEN-tuhn] equation to model the enzyme kinetics of this reaction, and why or why not? W) Yes, because the conditions are changing X) No, because the conditions are changing Y) Yes, because the enzyme activity remains unchanged Z) No, because the enzyme activity remains unchanged

X) NO, BECAUSE THE CONDITIONS ARE CHANGING

What facial muscle functions to enable blinking? W) Zygomaticus [zy-guh-MA-ti-kuhs] X) Orbicularis oculi [or-bee-kyoo-LUH-ris] Y) Orbicularis oris Z) Corrugator supercillii

X) ORBICULARIS OCULI

Which of the following animals is most likely to be responsible for girdling a tree? W) Skunk X) Porcupine Y) Raccoon Z) Yellow bellied marmot

X) PORCUPINE

Animals excrete various forms of nitrogenous [ny-TROJ-uhnuhs] wastes that are mainly the result of the catabolism of which of the following? W) Cellulose and triglycerides [try-GLIS-uh-ryds] X) Proteins and nucleic acids Y) Glycolipids and glycerol Z) Starches and proteins

X) PROTEINS AND NUCLEIC ACIDS

BONUS BIOLOGY Multiple Choice With which of the following are cyclin-dependent kinases most directly involved: W) repairing damaged DNA X) regulating cell cycle Y) digesting plasmids Z) insulin's action on fat cells

X) REGULATING CELL CYCLE

Which of the following eye adaptations allows birds of prey like raptors to maintain high visual sensitivity when diving for prey? W) Presence of a tapetum lucidum [tuh-PEE-tuhm loo-SEE-duhm] X) Retina with two foveas [FOH-vee-uhs] Y) Ability to change relative amounts of cis and trans retinal Z) Monocular vision

X) RETINA WITH TWO FOVEAS

Bonus: Heartwood and sapwood consist of which of the following? W) Bark X) Secondary xylem Y) Periderm Z) Secondary phloem

X) SECONDARY XYLEM

Bonus: Which of the following proteins is involved with prokaryotic transcription? W) DNA polymerase [paw-LIM-er-ace] 3 X) Sigma factor Y) Elongation factor Z) Amino acyl tRNA synthetase [ahMEEno-ASS-il tee-R.N.A. SIN-thah-tase]

X) SIGMA FACTOR

BIOLOGY Multiple Choice Which of the following mutations would most likely affect a gene the LEAST: W) single base deletion X) single base substitution Y) single base insertion Z) quadruple base deletion

X) SINGLE BASE SUBSTITUTION

In which of the following populations would you expect genetic drift to show the strongest effect? W) Large island group of penguins X) Small colony of zoo penguins Y) All Emperor penguins on Antarctica Z) All penguins on Antarctica

X) SMALL COLONY OF ZOO PENGUINS

In a particular bird population, individuals that lay too many or too few eggs are less likely to survive and reproduce. Which of the following terms best describes this pattern of natural selection? W) Directional selection X) Stabilizing selection Y) Disruptive selection Z) Negative-frequency-dependent selection

X) STABILIZING SELECTION

If your parietal [puh-RAY-i-tl] cells were to stop functioning, which of the following would you need to stop consuming? W) Soda pop X) Steak Y) Bread Z) Salad greens

X) STEAK

All of the following pairs of structures are homologous [huhMOL-uh-guhs], EXCEPT: W) The wing of a bat and the wing of a bird X) The leg of a human and the leg of a centipede Y) The scales on a bird's legs and the scales on a lizard Z) The shell of a clam and the shell of a snail

X) THE LEG OF A HUMAN AND THE LEG OF A CENTIPEDE

Which of the following glands contributes to thermoregulation by adjusting the resting metabolic rate? W) Pancreas X) Thyroid Y) Anterior pituitary [pi-TOO-i-ter-ee] Z) Adrenal

X) THYROID

Which of the following types of microscopy require the specimen to be subjected to a vacuum? W) Fluorescence X) Transmission electron Y) Scanning tunneling Z) Dark field

X) TRANSMISSION ELECTRON

What is the probability of obtaining an F1 purple pea flower if purple is dominant, white is recessive, and a parental cross is made between a heterozygous purple and homozygous white flower? W) 1/8 X) 1/4 Y) 1/2 Z) 1

Y) 1/2

Someone has carelessly tossed several sets of human vertebrae into a single box. You are trying to sort out the mess and have counted 20 lumbar, 48 thoracic, and 26 cervical vertebrae. It's now clear you are missing some bones. How many bones are you missing, and what type of vertebrae are they? W) 4 lumbar X) 2 thoracic Y) 2 cervical Z) 4 thoracic

Y) 2 CERVICAL

In dragons, the alleles [uh-LEELS] for yellow flame color and blue flame color are incompletely dominant. If a homozygous [hoh-muh-ZY-guhs] yellow-flame dragon mates with a homozygous blue-flame dragon, all of the brood will have green flame. Which of the following choices represents an expected probability for a brood produced by two green-flame dragons? W) 75% green flame X) 75% blue flame Y) 25% yellow flame Z) 25% green flame

Y) 25% YELLOW FLAME

In the spring of 2013 a group of researchers from the University of Alberta were able to revive an organism that had been trapped in glacial ice for approximately 400 years. What was the organism? W) An insect X) An ameba Y) A bryophyte [BRY-uh-fyt] Z) A tardigrade [TAHR-di-grayd]

Y) A BRYOPHYTE

Multiple Choice In insects, larval imaginal discs typically produce all of the following adult structures, EXCEPT: W) Legs X) Wings Y) Abdomens Z) Antennae

Y) ABDOMENS

During starvation, muscles convert most of the amino acids released from muscle breakdown to what two specific amino acids for use in gluconeogenesis [gloo-koh-nee-uh-JEN-uh-sis]? W) Arginine and glycine [AHR-juh-neen] X) Arginine and glutamine Y) Alanine and glutamine Z) Glutamine and glycine [GLY-seen]

Y) ALANINE AND GLUTAMINE

BONUS BIOLOGY Multiple Choice: Which of the following is LEAST accurate regarding human genetics: W) genes from gametes are typically unchanged during their passage to the zygote X) the zygote gets 1 allele from each parental gene pair Y) all genes independently segregate Z) the number of copies of a particular gene can vary among individuals

Y) ALL GENES INDEPENDENTLY SEGREGATE

BONUS BIOLOGY Multiple Choice Which of the following types of fatty acids would alpha linolenic or all cis-9,12,15-Octadecatrienoic (read as: octa-deca-tri-en-oh-ick) acid most commonly be classified: W) an alpha 9 fatty acid X) an alpha 9,12,15 fatty acid Y) an omega 3 fatty acid Z) an omega 6 fatty acid

Y) AN OMEGA 3 FATTY ACID

What complex in oxidative phosphorylation [FOS-for-uh-layshuhn] moves protons into rather that out of the mitochondrial [my-tuh-KON-dree-uhl] matrix? W) Cytochrome oxidase X) Cytochrome b-c1 Y) ATP synthase Z) NAD dehydrogenase [dee-HY-druh-juh-nays]

Y) ATP SYNTHASE

What is the name of the genus of the early hominid Lucy? W) Ardipithecus [ahr-de-PITH-i-cuhs] X) Homo Y) Australopithecus [aw-stray-loh-PITH-i-kuhs] Z) Paranthropus [par-RUN-throh-puhs

Y) AUSTRALOPITHECUS

One species resembles another species that is poisonous. What is this phenomenon called? W) Commensalism [kah-MEN-sul-ism] X) Mutualism Y) Batesian mimicry [BAIT-see-in MIM-ih-kree] Z) Mullerian mimicry[muhl-AIR-ee-an MIM-ih-kree]

Y) BATESIAN MIMICRY

Bacteria are able to become resistantto antibiotics by increasing their genetic variability through all of the following mechanisms EXCEPT: W) Conjugation X) Transduction Y) Binary fission Z) Transformation

Y) BINARY FISSION

BONUS BIOLOGY Multiple Choice Through which of the following mechanisms do allosteric inhibitors typically operate: W) binding the substrate X) binding to an enzyme's active site Y) binding to a site other than the active site and changing the shape of an enzyme Z) acting as a competitive inhibitor and overpowering the active sites of enzymes

Y) BINDING TO A SITE OTHER THAN THE ACTIVE SITE AND CHANGING THE SHAPE OF AN ENZYME

Which of the following vitamins is NOT fat soluble? W) Tocopherol [toh-KOF-uh-rol] X) Phylloquinone [fil-oh-KWIN-ohn] Y) Biotin Z) Calciferol [kal-SIF-uh-rol]

Y) BIOTIN

BONUS BIOLOGY Multiple Choice: Which of the following BEST describes how Pasteur attenuated the rabies virus to create an effective vaccine: W) by heat-treating the purified virus X) by treating the live virus with a weak solution of formaldehyde Y) by passing the rabies infection through a successive series of rabbits Z) by purifying the protein from the viral capsids

Y) BY PASSING THE RABIES INFECTION THROUGH A SUCCESSIVE SERIES OF RABBITS

BIOLOGY Multiple Choice Of which of the following is endosperm in wheat primarily composed: W) protein X) nucleic acid Y) carbohydrate Z) lipid

Y) CARBOHYDRATE

The water-proof layer within the endodermis of the root that filters substances entering a plant is called the: W) Pericycle X) Hypodermis Y) Casparian strip Z) Vascular cambium

Y) CASPARIAN STRIP

BIOLOGY Multiple Choice: Which of the following is the main target of bacteria by the antibiotic penicillin: W) energy production X) DNA synthesis Y) cell wall synthesis Z) translation

Y) CELL WALL SYNTHESIS

Which of the following is true regarding the major difference between cellulose and glycogen [GLY-kuh-juhn]? W) Cellulose is a polymer of galactose [guh-LAK-thos], while glycogen is a polymer of glucose X) Cellulose is mostly found in animal cells, while glycogen is mostly found in plant cells Y) Cellulose consists of beta glycosidic [gly-kuh-SY-dik] linkages, while glycogen consists of alpha glycosidic linkages Z) Cellulose is highly branched, while glycogen is linear

Y) CELLULOSE CONSISTS OF BETA GLYCOSIDIC LINKAGES, WHILE GLYCOGEN CONSISTS OF ALPHA GLYCOSIDIC LINKAGES

What is the most immediate role for the protons produced from water photolysis [foh-TOL-uh-sis] during photosynthesis? W) React with oxygen to make water X) Acidify the mitochondrial matrix and provide fuel for chemiosmosis Y) Contribute to the proton gradient in the thylakoid and ultimately provide energy to make ATP Z) Combine with carbon dioxide during sugar synthesis

Y) CONTRIBUTE TO THE PROTON GRADIENT IN THE THYLAKOID AND ULTIMATELY PROVIDE ENERGY TO MAKE ATP

What is the organ, found in birds and some invertebrates, that is used to temporarily store food, but also aids in preliminary digestion? W) Liver X) Gizzard Y) Crop Z) Cloaca [kloh-AY-kuh]

Y) CROP

Multiple Choice Wolves are introduced to an island populated with white-tailed deer. After several generations, the average sprint speed of the deer increased significantly relative to the wolf introduction period. In evolutionary terms, of which of the following kinds of selection is this is an example? W) Disruptive X) Stabilizing Y) Directional Z) Kin

Y) DIRECTIONAL

Which of the following processes would be least similar in terms of ATP production between normal adipose tissue and brown adipose tissue? W) Glycolysis [gly-KOL-uh-sis] X) Glycogenolysis [gly-KUH-juh-NOL-i-sis] Y) Electron transport chain Z) Citric acid cycle

Y) ELECTRON TRANSPORT CHAIN

BONUS BIOLOGY Multiple Choice Which of the following will most likely result from a single base deletion in DNA: W) nonsense mutation X) stop codon Y) frame shift mutation Z) inversion mutation

Y) FRAME SHIFT MUTATION

Bonus: Which of the following plant hormones plays the largest role in the germination of a seed? W) Cytokinin [sye-toh-KYE-nin] X) Zeatin [ZEE-ah-tin] Y) Gibberellins [jib-er-ELL-ins] Z) Ethylene

Y) GIBBERELLINS

BONUS BIOLOGY Multiple Choice Which of the following is NOT true regarding glycogen: W) glycogen synthesis is endergonic (read as: en-der-GONE-ick) X) glycogen is a polysaccharide Y) glycogen side branches have beta 1,4 linkages Z) glycogen in the human liver often has more than 10,000 glucose monomers

Y) GLYCOGEN SIDE BRANCHES HAVE BETA 1,4 LINKAGES

BONUS BIOLOGY Multiple Choice: In which of the following areas of the human body is the zygomatic (read as: ZYE-go-MAT-ick) arch found: W) shoulder X) pelvis Y) head Z) knee

Y) HEAD

Which of the following plants belong to the phylum [FY-luhm] Pterophyta [ter-uh-FY-tuh]? W) Ferns and palm trees X) Bryophytes and Horsetails [BRY-uh-fyts] Y) Horsetails and ferns Z) Palm trees and ginkgo

Y) HORSETAILS AND FERNS

BIOLOGY Multiple Choice: Which of the following is TRUE: W) chimpanzees are an ancestor of Homo sapiens X) bipedal motion dominated hominoid ancestors by about 30 million years ago Y) humans and modern apes have a common ancestor Z) Neanderthals were ancestors to modern humans

Y) HUMANS AND MODERN APES HAVE A COMMON ANCESTOR

Within the human forebrain, which of the following structures does not belong to the telencephalon [tel-en-SEF-uh-lon] ? W) Basal nuclei X) Hippocampus [hip-uh-KAM-puhs] Y) Hypothalamus [hy-puh-THAL-uh-muhs] Z) Cerebral cortex

Y) HYPOTHALAMUS

Mendel succeeded in forming the basis of modern hereditary theory in part because of his training: W) as a printer of rare and scientific documents X) in calligraphy of scientific manuscripts for the Royal Austrian Science Society Y) in probability and mathematics Z) as a scientific apostolic nuncio

Y) IN PROBABILITY AND MATHEMATICS

Bonus: All of the following changes would result in an increase in blood pressure, except: W) Increased cardiac output X) Increased heart rate Y) Increased diameter of arterioles Z) Increased stroke volume

Y) INCREASED DIAMETER OF ARTERIOLES

The arrector pili [pee-LEE] muscle has what function when a mammal is cold? W) Inducing shivering X) Speeding metabolism Y) Increasing the depth of the air insulation barrier Z) Vasoconstriction

Y) INCREASING THE DEPTH OF THE AIR INSULATION BARRIER

You are surfing at the beach and see a shark nearby, activating your vertebrate stress response. Levels of which of the following would decrease during this response? W) Cortisol X) Glucagon [GLOO-kuh-gon] Y) Insulin Z) Epinephrine [e-puh-NE-frin]

Y) INSULIN

Chlorosis in plants can be caused by a deficiency in which of the following nutrients? W) Zinc X) Phosphorous Y) Iron Z) Calcium

Y) IRON

Which of the following trace nutrients is most important for proper function of cytochrome c [SY-tuh-krohm C]? W) Magnesium X) Copper Y) Iron Z) Chromium

Y) IRON

BIOLOGY Multiple Choice Which of the following BEST explains how cyclic AMP can cause so many different intracellular responses in a mammalian cell: W) it is produced in large quantities X) it migrates to the DNA where it binds to a variety of histones Y) it activates a wide variety of specific protein kinases Z) it prevents the activation of other hormonal signals

Y) IT ACTIVATES A WIDE VARIETY OF SPECIFIC PROTEIN KINASES

You are walking in July when you come across a hedgehog. The animal does not stir when you pick it up; it is breathing and is not injured. What is the best explanation for the animal's lack of responsiveness? W) It is hibernating X) It is in a state of torpor Y) It is estivating Z) It is in a state of suspended animation

Y) IT IS ESTIVATING

Which of the following fish groups is most likely to utilize tidal ventilation when feeding? W) Shark X) Tuna Y) Lamprey Z) Eel

Y) LAMPREY

Which of the following describes antibody-dependent cell-mediated cytotoxicity? W) An invertebrate host defense mechanism X) Achieved via the classical complement pathway Y) Mediated by Fc receptors on NK cells and myeloid leukocytes Z) The primary cytotoxic pathway mediated by T cells

Y) MEDIATED BY FC RECEPTORS ON NK CELLS AND MYELOID LEUKOCYTES

BONUS BIOLOGY Multiple Choice Which of the following is NOT true regarding ribosomes: W) they are about 20 nanometers in diameter X) eukaryotic ribosomes are slightly larger than prokaryotic ribosomes Y) mitochondrial ribosomes have 40S (read as: forty S) small subunits Z) they are found in chloroplasts

Y) MITOCHONDRIAL RIBOSOMES HAVE 40S SMALL SUBUNITS

BIOLOGY Multiple Choice Which of the following BEST describes the function of the golgi (read as: GOAL-gee) complex: W) metabolism of carbohydrates X) production of peroxides Y) modification and packaging of proteins for exocytosis Z) storage of lipids

Y) MODIFICATION AND PACKAGING OF PROTEINS FOR EXOCYTOSIS

Injury to cranial nerve III would result in an inability to do which of the following? W) Smell X) Chew food Y) Move eyes to the left Z) Feel pain in the eye area

Y) MOVE EYES TO THE LEFT

Which of the following forms of nitrogen cannot be directly utilized by plants? W) Ammonia X) Ammonium Y) Nitrogen gas Z) Nitrite

Y) NITROGEN GAS

Bonus: Which of the following thought to have been the beneficial effect of two entire genome duplication events in the vertebrate lineage? W) More copies of each gene per cell were insurance against DNA damage from increased solar radiation X) Polyploid animals were more likely to have one good copy of each gene Y) One copy of a duplicated gene was free to take on new functions Z) Four copies of each gene meant stronger gene responses

Y) ONE COPY OF A DUPLICATED GENE WAS FREE TO TAKE ON NEW FUNCTIONS

Bonus: Which of the following is NOT a location where bile is either stored or travels to after it is made? W) Gall bladder X) Cystic duct Y) Pancreatic duct Z) Duodenum

Y) PANCREATIC DUCT

Bonus: Which of the following statements best describes the difference in responses of plasma cells and cytotoxic T-cells? W) Plasma cells respond the first time the invader is present, cytotoxic T-cells respond subsequent times X) Plasma cells execute the cell-mediated response, cytotoxic T-cells execute the humoral response Y) Plasma cells secrete antibodies against a pathogen, cytotoxic T-cells kill virus-infected cells Z) Plasmacells confer active immunity, cytotoxic T-cells confer passive immunity

Y) PLASMA CELLS SECRETE ANTIBODIES AGAINST A PATHOGEN, CYTOTOXIC T-CELLS KILL VIRUS-INFECTED CELLS

BIOLOGY Multiple Choice Which of the following is NOT true of globular proteins in mammals: W) most are soluble in water X) some serve as transport molecules in the plasma Y) plasma enzymes typically have an optimal operating pH of about 6.8 to 7.0 Z) most protein hormones are globular

Y) PLASMA ENZYMES TYPICALLY HAVE AN OPTIMAL OPERATING PH OF ABOUT 6.8 TO 7.0

Which of the following steroid hormones is the precursor to all of the others listed in the answer choices? W) Estrone X) Testosterone Y) Progesterone Z) Androstenedione [an-druh-steen-DY-ohn]

Y) PROGESTERONE

BIOLOGY Multiple Choice Which of the following best describes the repressor protein in the lac (read as: LACK) operon: W) uncompetitive inhibitor X) structural protein Y) regulatory protein Z) transcriptional factor

Y) REGULATORY PROTEIN

Ribulose-1,5-bisphosphate carboxylase [kahr-BOK-suhlayz]/oxygenase[OK-si-juh-nays], also known as RuBisCO, is an enzyme present in photosynthetic plants and is thought to be the most abundant protein on earth. Which of the following is NOT a reason that plants produce such large quantities of RuBisCO? W) RuBisCO is a slow-acting enzyme X) RuBisCO lacks specificity for carbon Y) RuBisCO is inhibited by carbon dioxide Z) Plants build large aggregate complexes of RuBisCO

Y) RuBisCO IS INHIBITED BY CARBON DIOXIDE

Which of the following is true regarding plant ground tissue? W) Parenchyma [puh-RENG-kuh-muh] is mostly responsible for structural support X) Collenchyma [kuh-LENG-kuh-muh] consists of dead cells with thick cell walls Y) Sclerenchyma [skli-RENG-kuh-muh] is the most lignified type of ground tissue Z) Sclerenchyma cells can adapt to stress conditions at maturity

Y) SCLERENCHYMA IS THE MOST LIGNIFIED TYPE OF GROUND TISSUE

Bonus: Which of the following mutations would most likely result in a frame-shift mutation? W) Single base substitution X) Triple base substitution Y) Single base deletion Z) Triple base deletion

Y) SINGLE BASE DELETION

BIOLOGY Multiple Choice Which of the following is NOT a way that monarch butterflies manage energy on their migrations? W) Fat storage in the abdomen X) Flying with the wind Y) Switching to higher-energy food sources Z) Gliding instead of flapping

Y) SWITCHING TO HIGHER-ENERGY FOOD SOURCES

BIOLOGY Multiple Choice: Which of the following is the highest level of structural organization in myoglobin: W) primary X) secondary Y) tertiary Z) quaternary

Y) TERTIARY

BONUS BIOLOGY Multiple Choice Which of the following BEST explains why a single base substitution in a gene-coding region for amino acids will sometimes lead to no change in amino acid sequence: W) all nucleotides are subject to the same mutation rate X) some nucleotide bases are covalently joined by stronger bonds Y) the code is degenerate with more than one triplet coding for the same amino acid Z) some amino acids act in the same fashion in determining the tertiary structure of a protein

Y) THE CODE IS DEGENERATE WITH MORE THAN ONE TRIPLET CODING FOR THE SAME AMINO ACID

Which of the following statements is NOT true of an r-selected species? W) The mother produces a large number of eggs that suffer heavy predation X) The mother reaches sexual maturity early in life Y) The parents provide long-term care for the offspring Z) The species is more successful in unstable environments

Y) THE PARENTS PROVIDE LONG-TERM CARE FOR THE OFFSPRING

Which of the following is NOT true about plant physiology? W) Meristematic [meh-rih-stuh-MAT-ic} tissue located at the tips of stems and roots forms the apical [A-pik-ul] meristems [MEH-rih-stems] X) Some plants, like shrubs, have lateral meristems Y) Vascular and cork cambium [KAM-bee-um] are produced by apical meristems Z) Intercalary [In-TER-cah-lair-ee] meristems allow grass to grow even after it has been mowed

Y) VASCULAR AND CORK CAMBIUM ARE PRODUCED BY APICAL MERISTEMS

Which of the following techniques would you use to determine the 3-D structure of a protein? W) Mass spectrophotometry X) Nuclear magnetic resonance Y) X-ray crystallography [kris-tl-OG-ruh-fee] Z) Atomic absorption spectroscopy

Y) X-RAY CRYSTALLOGRAPHY

Which of the following examples is NOT evidence for evolution from a common ancestor? W) All living things have basic cell processes that are similar and use similar molecules X) Hox genes, or homeotic genes, control vertebrate body plans and are homologous in all vertebrates Y) Even though snakes don't have legs, some snake embryos have limb buds during development similar to other reptile embryos Z) Bats and birds both have wings that are adaptations for flying

Z ) BATS AND BIRDS BOTH HAVE WINGS THAT ARE ADAPTATIONS FOR FLYING

Which of the following best describes how oxygen is stored in the muscles? W) Spread throughout X) Bound to hemoglobin Y) Bound to calmodulin Z) Bound to myoglobin

Z ) BOUND TO MYOGLOBIN

Bungarotoxin [bung-garoh-TOK-sins] is a fast-acting lethal toxin that is the primary component of venom from the South Asian snake called a krait. It blocks nerve transmission at cholinergic [koh-luh-NUR-jik] synapses by competing with acetylcholine [uh-seet-l-KOH-leen] to bind irreversibly at the acetylcholine receptor. Which of the following would be LEAST LIKELY to also impact nerve transmission at the cholinergic synapses? W) A drug floods the synapse with a molecule that resembles acetylcholine and is able to bind at the acetylcholine receptor, but cannot be recognized by acetylcholinesterase [uh-seet-lKOH-leen-STUH-rays] X) A chemical covalently binds to the acetylcholine receptor and is not easily inactivated Y) A chemical covalently bonds acetylcholinesterase at the enzyme active site Z) A drug increases the level of acetylcholinesterase present at the synapse

Z) A DRUG INCREASES THE LEVEL OF ACETYLCHOLINESTERASE PRESENT AT THE SYNAPSE

Which of the following amniotic egg structures does NOT match its function? W) Amnion, protects developing embryo in fluid filled sac X) Yolk sac, stockpiles nutrients Y) Chorion [KOHR-ee-on], facilitates gas exchange Z) Allantois, connects yolk to embryo

Z) ALLANTOIS, CONNECTS YOLK TO EMBRYO

Bonus: Which of the following is NOT an example of a stone fruit? W) Almond X) Coconut Y) Cherry Z) Apple

Z) APPLE

Whathuman organ utilizes the most glucose in the body? W) Liver X) Heart Y) Lungs Z) Brain

Z) BRAIN

Which of the following is NOT a direct requirement for dinoflagellates [din-uh-FLAJ-uh-layts] to carry out bioluminescence? W) Luciferyl adenylate [loo-sif-UH-ril uh-de-ni-layt] X) Oxygen Y) Inorganic pyrophosphate Z) Carbon dioxide

Z) CARBON DIOXIDE

If a person has consumed too much alcohol and is showing signs of ataxia [uh-TAK-see-uh], what portion of the brain is being inhibited? W) Left hemisphere of the cerebrum X) Tectum Y) Thalamus Z) Cerebellum [ser-uh-BEL-uhm]

Z) CEREBELLUM

What material typically constitutes the cell wall in fungi? W) Cellulose X) Peptidoglycan Y) Pectin Z) Chitin

Z) CHITIN

Which of the following hormones is a precursor of both estrogen and testosterone? W) Cortisol X) Ergosterol [ur-GAWS-tur-awl] Y) Aldosterone [al-DOSS-ter-own] Z) DHEA

Z) DHEA

As the average temperature dropped during the last glacial period, a species of cave bears evolved a layer of thicker fur. What kind of selection is this an example of? W) Speciation X) Disruptive selection Y) Stabilizing selection Z) Directional selection

Z) DIRECTIONAL SELECTION

Which of the following is an example of commensalism? W) A hummingbird drinking from a flower X) A tapeworm living in a host intestine Y) A clown fish living in a sea anemone [uh-NEM-uh-nee] Z) Egrets foraging among cattle fields

Z) EGRETS FORAGING AMONG CATTLE FIELDS

Most cells are exposed to extracellular glucose concentrations higher than those inside the cell. Glucose is taken up by these cells via which of the following? W) Osmosis X) Active transport Y) Exocytosis Z) Facilitated diffusion

Z) FACILITATED DIFFUSION

Membrane depolarization of an action potential is able to pass from one cardiac cell to another through what type of connection between cells? W) Desmosomes X) Tight junction Y) Occluding junction Z) Gap junction

Z) GAP JUNCTION

BIOLOGY Multiple Choice The combined genetic information of all the members of a particular population is known as the: W) Allele (read as: ah-LEEL) frequency X) Genome Y) Directional selection Z) Gene pool

Z) GENE POOL

In humans, in addition to transferring antibodies, colostrum also stimulates the development of the neonatal gut through the transference of which of the following? W) Pheromones X) Non-heritable mutations Y) Beta-galactose Z) Growth factors

Z) GROWTH FACTORS

Which of the following animal respiratory pigments does not contain iron? W) Hemoglobin [HEE-muh-gloh-bin] X) Hemoerythrin [hee-muh- ih-RITH-rin] Y) Myoglobin [my-uh-GLOH-bin] Z) Hemocyanin [hee-muh-SAHY-uh-neen]

Z) HEMOCYANIN

Which of the following brain regions is incorrectly matched with its function? W) Medulla oblongata regulates breathing X) Pons regulates breath holding Y) Cerebellum integrates sensory systems [ser-uh-BEL-uhm] Z) Hippocampus [hip-uh-KAM-puhs] allows two brain hemispheres to communicate with each other

Z) HIPPOCAMPUS ALLOWS TWO BRAIN HEMISPHERES TO COMMUNICATE WITH EACH OTHER

All of the following distinguish actin filaments from microtubules and intermediate filaments except which characteristic? W) Promotion of muscle contraction X) Ability to create amoeboid [uh-MEE-boid] cell movement Y) Spiral structure Z) Intracellular movement of cargo

Z) INTRACELLULAR MOVEMENT OF CARGO

Which of the following proteins makes up intermediate filaments in the cytoskeleton? W) Actin X) Myosin [MY-uh-sin] Y) Tubulin Z) Keratin

Z) KERATIN

Bonus: In the human skull, which of the following bones does NOT contain a sinus? W) Frontal X) Sphenoid [SFEE-noyd] Y) Maxillae [MAX-il-eye] Z) Lacrimal

Z) LACRIMAL

BONUS Multiple Choice Members of the order Gymnophiona (read as: jim-noh-FYOHnah), otherwise known as Caecilians (read as: si-SIL-yahns), do NOT have which of the following? W) Poison glands X) Teeth Y) Elongated organs Z) Legs

Z) LEGS

Bonus: Mycobacterium [MY-koh-bacterium]is a genus of slow-growing and difficult-to-eradicate bacteria responsible for what two human diseases? W) Mononucleosis and pertussis [per-TUH-sis] X) Shigella [shih-GEHL-ah]and malaria Y) Gingivitis and impetigo [im-puh-tie-goh] Z) Leprosy and tuberculosis

Z) LEPROSY AND TUBERCULOSIS

Which of the following statements about cystic fibrosis is NOT true? W) It is an autosomal recessive disease X) It affects the lungs, sweat glands and digestive system Y) It is one of the most common fatal genetic disorders in the United States Z) Most infants born with this disorder are unlikely to live beyond their first year

Z) MOST INFANTS BORN WITH THIS DISORDER ARE UNLIKELY TO LIVE BEYOND THEIR FIRST YEAR

What disadvantage does the presence of introns and exons give eukaryotes [yoo-KAR-ee-ohts] when compared to other organisms? W) The ability to produce more proteins from the same primary transcript X) Greater stability during reproduction Y) Lower error rate during replication Z) Mutations are harder to detect

Z) MUTATIONS ARE HARDER TO DETECT

Which of the following drugs is mismatched to its action on neurotransmission? W) Benzodiazepines: Increases inhibitory neurotransmission [ben-zoh-dy-AZ-uh-pyns] X) Cocaine: inhibits release of dopamine and norepinephrine [nohr-ep-uh-NEF-rin] Y) LSD: Binds to serotonin receptors Z) Nicotine: decreases brain dopamine levels

Z) NICOTINE DECREASES BRAIN DOPAMINE LEVELS

Which of the following is NOT true when comparing the 4 steps of aerobic respiration to the 2 steps of anaerobic respiration? W) Both utilize compounds other than glucose, such as lipids, for energy X) Both recycle NADH [N-A-D-H] to NAD+ [N-A-D plus] Y) Both have steps occurring in the cytoplasm Z) Only one produces pyruvate [py-ROO-vayt]

Z) ONLY ONE PRODUCES PYRUVATE

Mistletoe is a vascular plant that imbeds root-like structures into the limbs of a tree, robbing it of moisture and nutrients. The mistletoe provides no resources to the tree on which it depends. The ecological relationship between the mistletoe and the tree is an example of which of the following? W) Commensalism X) Herbivory [hur-buh-VOH-re] Y) Mimicry Z) Parasitism

Z) PARASITISM

Bonus: Sepals, petals, and which of the following provide the most complete description for the floral parts that could be found in a single flower of a monoecious plant species? W) Stamens and carpels X) Filaments Y) Carpels Z) Pistils

Z) PISTILS

Bonus: Which of the following disorders is due to a dominant trait? W) Cystic fibrosis X) Hemophilia [HEE-moh-FEEL-ee-ah] Y) Albinism [AL-bah-nih-zim] Z) Polydactyly [polly-DAK-til-ee]

Z) POLYDACTYLY

BIOLOGY Multiple Choice: From which of the following is the cell wall of plants mostly composed: W) protein X) lipid monolayer Y) lipid bilayer Z) polysaccharide

Z) POLYSACCHARIDE

BIOLOGY Multiple Choice: With which of the following does human interstitial fluid differ in concentration the most when compared with plasma: W) sodium ions X) potassium ions Y) magnesium ions Z) proteins

Z) PROTEINS

Which of the following is NOT an example of negative feedback? W) Excess ATP inhibiting phosphofructokinase [fohs-foh-frook-toh-KY-nays] X) High blood glucose causing insulin release Y) Secretion of cortisol inhibiting ACTH release Z) Release of platelet activating factor during clotting

Z) RELEASE OF PLATELET ACTIVATING FACTOR DURING CLOTTING

BONUS BIOLOGY Multiple Choice Which of the following BEST describes the function of a spliceosome (read as: SPLY-see-oh-zome): W) splicing DNA together during replication X) splicing out mutations Y) joining multiple copies of structural genes Z) removing introns

Z) REMOVING INTRONS

Which of the following pairs of echinoderms [ih-KY-nuh-durms] have respiratory structures that could be characterized as lungs rather than gills, since they are formed from invaginations of the body surface? W) Sea star and sea urchin X) Sea urchin and brittle star Y) Sea star and brittle star Z) Sea cucumber and brittle star

Z) SEA CUCUMBER AND BRITTLE STAR

BONUS BIOLOGY Multiple Choice Which of the following substances is transported into cells via carrier-assisted transport?

Z) SODIUM ION

Which of the following structures would have been most likely absent from the last shared common ancestor of osteichthyans [os-tee-IK-thee-uhns] and chondrichthyans [kan-DRIK-thee-ez]? W) Teeth X) Gills Y) Scales Z) Swim bladder

Z) SWIM BLADDER

BONUS BIOLOGY Multiple Choice: Which of the following BEST describes what the term virulence means: W) the specificity of a virus to cause disease in a certain host X) the ability of a microbe to cause cellular damage Y) any transmissible disease caused by a microbe Z) the degree to which a microbe causes disease or harm to its host

Z) THE DEGREE TO WHICH A MICROBE CAUSES DISEASE OR HARM TO ITS HOST

51 Place filler

f

A man is born with webbed fingers and toes. His wife and their daughter do not have webbed fingers and toes. If webbed digits are a dominant trait, what fraction of this couple's children would be expected to have webbed digits?

½


Ensembles d'études connexes

Energi - konsekvenser af produktion og forbrug

View Set

Pediatric Growth and Development Quiz 2

View Set

(Praxis) Teaching Reading: Elementary 5205

View Set

Property/Casualty CH.6 EXAM QUESTIONS

View Set